21 22test11 EnvironmentExplanation

You might also like

Download as pdf or txt
Download as pdf or txt
You are on page 1of 139

PRESTORMINGTM 2022

TEST – 11 ENVIRIONMENT
EXPLANATION
QUESTION NO PAGE NO
Q.1) .............................................................................................................................................. 4
Q.2) .............................................................................................................................................. 5
Q.3) .............................................................................................................................................. 6
Q.4) .............................................................................................................................................. 7
Q.5) .............................................................................................................................................. 8
Q.6) .............................................................................................................................................. 9
Q.7) ............................................................................................................................................ 10
Q.8) ............................................................................................................................................ 11
Q.9) ............................................................................................................................................ 14
Q.10) ............................................................................................................................................ 16
Q.11) . ........................................................................................................................................... 17
Q.12) ............................................................................................................................................ 18
Q.13) ............................................................................................................................................ 20
Q.14) . ........................................................................................................................................... 20
Q.15) . ........................................................................................................................................... 22
Q.16) ............................................................................................................................................ 24
Q.17) ............................................................................................................................................ 24
Q.18) ............................................................................................................................................ 26
Q.19) ............................................................................................................................................ 27
Q.20) . ........................................................................................................................................... 28
Q.21) ............................................................................................................................................ 30
Q.22) ............................................................................................................................................ 32
Q.23) ............................................................................................................................................ 34
Q.24) ............................................................................................................................................ 36
Q.25) ............................................................................................................................................ 37
Q.26) ............................................................................................................................................ 38
Q.27) ............................................................................................................................................ 39
Q.28) ............................................................................................................................................ 40
Q.29) ............................................................................................................................................ 42
Q.30) ............................................................................................................................................ 43
Q.31) ............................................................................................................................................ 44
Q.32) ............................................................................................................................................ 45
Q.33) ............................................................................................................................................ 48
Q.34) ............................................................................................................................................ 49
Q.35) ............................................................................................................................................ 50
Q.37) ............................................................................................................................................ 53
Q.38) ............................................................................................................................................ 55
Q.39) ............................................................................................................................................ 56
Q.40) . ........................................................................................................................................... 57
Q.41) ............................................................................................................................................ 59
Q.42) ............................................................................................................................................ 60
Q.43) ............................................................................................................................................ 61
Q.44) ............................................................................................................................................ 63
Q.45) ............................................................................................................................................ 65
Q.46) ............................................................................................................................................ 66
Q.47) ............................................................................................................................................ 67
Q.48) ............................................................................................................................................ 70
Q.49) ............................................................................................................................................ 72
Q.50) ............................................................................................................................................ 74
Q.51) ............................................................................................................................................ 76
Q.52) ............................................................................................................................................ 77
Q.53) ............................................................................................................................................ 79
Q.54) ............................................................................................................................................ 81
Q.55) ............................................................................................................................................ 83
Q.56) ............................................................................................................................................ 84
Q.57) ............................................................................................................................................ 85
Q.58) . ........................................................................................................................................... 86
Q.59) ............................................................................................................................................ 88
Q.60) ............................................................................................................................................ 89
Q.61) . ........................................................................................................................................... 90
Q.62) . ........................................................................................................................................... 91
Q.63) ............................................................................................................................................ 92
Q.64) . ........................................................................................................................................... 93
Q.65) ............................................................................................................................................ 94
Q.66) ............................................................................................................................................ 95
Q.67) . ........................................................................................................................................... 96
Q.68) ............................................................................................................................................ 98
Q.69) . ........................................................................................................................................... 99
Q.70) . ........................................................................................................................................... 99
Q.71) .......................................................................................................................................... 100
Q.72) .......................................................................................................................................... 101
Q.73) .......................................................................................................................................... 103
Q.74) .......................................................................................................................................... 104
Q.75) .......................................................................................................................................... 104
Q.76) . ......................................................................................................................................... 105
Q.77) .......................................................................................................................................... 107
Q.78) .......................................................................................................................................... 107
Q.79) .......................................................................................................................................... 109
Q.80) .......................................................................................................................................... 110
Q.81) . ......................................................................................................................................... 112
Q.82) .......................................................................................................................................... 113
Q.83) .......................................................................................................................................... 115
Q.84) .......................................................................................................................................... 116
Q.85) .......................................................................................................................................... 118
Q.86) .......................................................................................................................................... 120
Q.87) .......................................................................................................................................... 121
Q.88) .......................................................................................................................................... 122
Q.89) .......................................................................................................................................... 125
Q.90) .......................................................................................................................................... 125
Q.91) . ......................................................................................................................................... 127
Q.92) .......................................................................................................................................... 128
Q.93) .......................................................................................................................................... 129
Q.94) .......................................................................................................................................... 130
Q.95) .......................................................................................................................................... 131
Q.96) .......................................................................................................................................... 132
Q.97) .......................................................................................................................................... 133
Q.98) . ......................................................................................................................................... 134
Q.99) .......................................................................................................................................... 136
Q.100) . ......................................................................................................................................... 138
Q.1) The Constitutional imperatives in terms of the Protection of Wildlife and environment can be traced
to which of the following provisions in the Indian Constitution?
1. Preamble
2. Fundamental Rights
3. Directive Principles
4. Fundamental Duties
Select the correct answer using the code given below.
(a) 2, 3 and 4 only
(b) 3 and 4 only
(c) 2 and 3 only
(d) 1, 2, 3 and 4
EXPLANATION:
The Constitution of India under part III guarantees fundamental rights which are
essential for the development of every individual and to which a person is inherently
entitled by being human alone. Right to the environment is also a right without which
the development of an individual and realization of his or her full potential shall not be
possible. Articles 21, 14, and 19 of this part have been used for environmental
protection. So, Statement 2 is correct.
The Directive principles under the Indian constitution are directed towards ideals of
building a welfare state. A healthy environment is also one of the elements of a welfare
state. Article 48 -A of the constitution says that “the state shall endeavor to protect and
improve the environment and to safeguard the forests and wildlife of the country”. So,
Statement 3 is correct.
The chapter on fundamental duties of the Indian Constitution imposes a duty on every
citizen to protect the environment. Article 51-A (g), says that “It shall be the duty of
every citizen of India to protect and improve the natural environment including forests,
lakes, rivers, and wildlife and to have compassion for living creatures.” So, Statement
4 is correct.
ADDITIONAL INFORMATION:
Constitutional Protection of Environment
Initial Stage When the Constitution of India was adopted in 1950, the framers
had not foreseen the importance of environmental preservation.
42nd In 1976, the 42nd amendment incorporated the protection of
Constitutional wildlife and forests in the Directive Principles. It also included
Amendment forests and protection of wild animals in the Concurrent List –
Seventh Schedule (Article 256) of the Constitution.
Provisions The protection of wildlife and the environment is now enshrined
in the following provisions of the Indian Constitution
 Fundamental Rights
 Directive Principles
 Fundamental Duties
Q.2) Consider the following statements:
1. National parks and Tiger Reserves are by law more strictly protected, allowing virtually no human
activity except that which is in the interest of wildlife conservation.
2. Grazing and private tenurial rights are disallowed in National Parks but can be allowed in Wildlife
Sanctuaries at the discretion of the Chief Wildlife Warden.
3. Community reserves and Conservation reserves provide a greater role for local communities,
stakeholders and civil society as well as the opportunity to protect many areas of conservation
value.
Which of the statements given above is/are correct?
(a) 1 only
(b) 1 and 2 only
(c) 2 and 3 only
(d) 1, 2 and 3
EXPLANATION:
The Wildlife (Protection) Act, 1972 (Last amended in 2006) provides for several categories
of Protected Areas/Reserves such as National Parks, Wildlife Sanctuaries, Tiger Reserves,
Conservation Reserves, and Community Reserves. National parks and Tiger Reserves are
by law more strictly protected, allowing virtually no human activity except that which is
in the interest of wildlife conservation. Grazing and private tenurial rights are disallowed
in National Parks but can be allowed in sanctuaries at the discretion of the Chief Wildlife
Warden. The amended WLPA does not allow for any commercial exploitation of forest
produce in both national parks and wildlife sanctuaries, and local communities can
collect forest produce only for their bona fide needs. So, Statements 1 and 2 are
correct.
Community reserves and conservation reserves are two new categories of protected areas
that have been included under the WLPA. These two categories provide a greater role for
local communities, stakeholders, and civil society as well as the opportunity to protect
many areas of conservation value that cannot be designated under strict categories such
as wildlife sanctuaries or national parks. So, Statement 3 is correct.

ADDITIONAL INFORMATION:
National Park  An area, whether within a sanctuary or not, can be notified
by the state government to be constituted as a National Park,
because of its ecological, faunal, floral, geomorphological, or
zoological association of importance, needed for protecting &
propagating or developing wildlife therein or its environment.
 No human activity is permitted inside the national park
except for the ones permitted by the Chief Wildlife Warden of
the state under the conditions given in chapter IV, WPA
1972.
Wildlife  Any area other than area comprised with any reserve forest
Sanctuary or the territorial waters can be notified by the State
Government to constitute as a sanctuary if such area is of
adequate ecological, faunal, floral, geomorphological, natural,
or zoological significance, to protect, propagate, or developing
wildlife or its environment.
 Some restricted human activities are allowed inside the
Sanctuary area details of which are given in chapter IV, WPA
1972.
Conservation and  Conservation reserves and community reserves in India are
Community terms denoting protected areas of India which typically act as
Reserves buffer zones to connectors and migration corridors between
established national parks, wildlife sanctuaries, and reserved
and protected forests of India.
 Such areas are designated as conservation areas if they are
uninhabited and completely owned by the Government of
India but used for subsistence by communities and
community areas if part of the lands is privately owned.
 These protected area categories were first introduced in the
Wildlife (Protection) Amendment Act of 2002 − the
amendment to the Wildlife Protection Act of 1972. These
categories were added because of reduced protection in and
around existing or proposed protected areas due to private
ownership of land, and land use.

Q.3) When the number of individuals in a population or species drops too low, then
1. its contributions to ecosystem functions and services become more important than before.
2. its genetic variability and resilience are reduced.
3. its contribution to human welfare may be lost.
Select the correct answer using the code given below.
(a) 1 only
(b) 1 and 2 only
(c) 2 and 3 only
(d) 1, 2 and 3
EXPLANATION:
When the number of individuals in a population or species drops too low, its
contributions to ecosystem functions and services become unimportant as there will be
another population or species that may enter and functions important role or change in
ecosystem occurs. So, Statement 1 is not correct.
In a small population, the erosion of genetic variability and resilience occurs, resulting
in restriction of the ability of a species to adapt to environmental change and survival
rate. So, Statement 2 is correct.
When the number of individuals in a population or species drops too low, then its
contribution to human welfare may be lost. For example, there will be more pandemics
if we continue destroying habitats and trading wildlife. So, Statement 3 is correct.
Q.4) Consider the following statements:
1. Any species growing exponentially under unlimited resource conditions can reach enormous
population densities in a short span of time.
2. A slow-growing animal like an elephant could not reach enormous numbers even in the absence of
checks due to slow growth in nature.
3. No population of any species in nature has at its disposal to unlimited resources to permit
exponential growth.
Which of the statements given above is/are correct?
(a) 1 only
(b) 1 and 2 only
(c) 2 and 3 only
(d) 1 and 3 only
EXPLANATION:
Any species growing exponentially under unlimited resource conditions can reach enormous
population densities in a short time. Darwin showed how even a slow-growing animal like an
elephant could reach enormous numbers in the absence of checks. So, Statement 1 is
correct and Statement 2 is not correct.
No population of any species in nature has at its disposal unlimited resources to permit
exponential growth. This leads to competition between individuals for limited resources.
Eventually, the ‘fittest’ individual will survive and reproduce. The governments of many
countries have also realized this fact and introduced various restraints to limit human
population growth. In nature, a given habitat has enough resources to support a maximum
possible number, beyond which no further growth is possible. So, Statement 3 is correct.

ADDITIONAL INFORMATION:
Q.5) Consider the following protected areas in India.
1. Rajaji national park in Uttarakhand
2. Hastinapur wildlife sanctuary in Uttar Pradesh
3. Vikramshilagangetic Dolphin Sanctuary in Bihar
Which one of the following statements is correct about the above given protected areas?
(a) All are the well-known protected areas of Gangetic dolphins
(b) All are the well-known protected areas for Royal Bengal Tigers
(c) All are newly declared Ramsar Sites in India
(d) All are the high biodiversity areas in the course of the Ganges River
EXPLANATION:
With celebrations of the Ganga Utsav - to mark the 12th anniversary of declaring the Ganga
as a national river - having come to an end, the Wildlife Institute of India (WII) in the second
phase of its survey of the entire main stem of the Ganga river (the main river without its
tributaries), has found that 49 percent of the river has high biodiversity and that biodiversity
sightings, including of the Gangetic Dolphin and otters, have increased in the river. Scientists
at the institute say that this indicates reducing pollution levels and a healthier state of the
river. Ten percent of the high biodiversity areas fall alongside national parks and sanctuaries
such as Rajaji national park in Uttarakhand, Hastinapur wildlife sanctuary in UP, and
Vikramshila Gangetic Dolphin Sanctuary in Bihar. So, Option (d) is correct.

ADDITIONAL INFORMATION:
Gangetic Plains – High Biodiversity areas
Freshwater Ecosystem Freshwater ecosystems account for 0.01% of the earth’s surface water
but 10% of species. According to the UN Environment Programme World
Conservation Monitoring Centre (UNEP-WCMC), decline in diversity of
freshwater species is the highest and surpasses losses in marine and
terrestrial species – globally 20% of all known freshwater fish, 44%
waterbirds, and 42% amphibian species are under threat of extinction.
Gangetic Plains The highest loss of freshwater biodiversity has been reported from the
Indian subcontinent, specifically the Gangetic plains. The reason why
the government has taken up the project of biodiversity conservation in
the region.
Distribution The Ganga and its tributaries flow through 11 states and cover 26.3
percent of the country’s total geographical area.
But its main stem flows through five states – Uttarakhand, UP, Bihar,
Jharkhand, and West Bengal.
WII Survey With celebrations of the Ganga Utsav – to mark the 12th anniversary of
declaring the Ganga as a national river – having come to an end, the
Wildlife Institute of India (WII) in the second phase of its survey of the
entire main stem of the Ganga river (the main river without its
tributaries), has found that 49 percent of the river has high biodiversity.
High Biodiversity The high biodiversity stretches have been divided into six zones –
Devprayag to Rishikesh (61 km), Makdumpur to Narora (147 km),
Bhitaura to Ghazipur (454 km), Chhapra to Kahalgaon (296 km),
Sahibganj to Rajmahal (34 km) and Baharampur to Barackpore (246
km).
Come Back Findings have shown that many species that used to be found in the
main stem and had disappeared, are now coming back. Found nesting
colonies of the Indian Skimmer. Seibold’s, a species of water snake,
disappeared 80 years ago and has now resurfaced and also found new
distributions of the pufferfish. Many other species have started traveling
back from tributaries to the main stem of the river, indicating improving
water quality.

Q.6) Consider the following statements:


1. Nagpur in Maharashtra is called as the Tiger capital of India.
2. Ladakh is known as the Snow Leopard capital of India.
3. Guwahati in Assam is called as Dolphin city as it become the first city to have its own city animal.
Which of the statements given above is/are correct?
(a) 1 only
(b) 1 and 2 only
(c) 2 and 3 only
(d) 1, 2 and 3
EXPLANATION:
Nagpur is also called, ‘the tiger capital or tiger getaway’ of India as it connects many Tiger Reserves
in India to the world. Nagpur lies precisely at the center of the country with the Zero Mile Marker
indicating the geographical center of India. So, Statement 1 is correct.
In India, Ladakh is the snow leopard’s main habitat, followed by the Lahul-Spiti districts of
Himachal Pradesh. The leopard is the region’s apex predator and is listed as Vulnerable on the
IUCN Red List. Experts put the population of snow leopards to be between 200-300 individuals in
Ladakh alone and speak about the fragile relationship of their healthy population to the overall
natural health of the mountains. Ladakh is internationally known as the snow leopard capital of
the world. A lot of attention is given to conservation in the region because of this species. So,
Statement 2 is correct.
Guwahati, the Assam state capital become the first city in the country to have its city animal with
the Kamrup Metropolitan district administration declaring the Gangetic River dolphin as its
mascot. So, Statement 3 is correct.

ADDITIONAL INFORMATION:
Snow Leopards
About Snow leopards have evolved to live in some of the harshest conditions on
Earth. Their thick white-gray coat spotted with large black rosettes
blends in perfectly with Asia’s steep and rocky, high mountains. Because
of their incredible natural camouflage, rendering them almost invisible in
their surroundings, snow leopards are often referred to as the “ghost of
the mountains.”
Distribution The snow leopard’s habitat range extends across the mountainous
regions of 12 countries across Asia: Afghanistan, Bhutan, China, India,
Kazakhstan, Kyrgyz Republic, Mongolia, Nepal, Pakistan, Russia,
Tajikistan, and Uzbekistan. The total range covers an area of close to
772,204 square miles, with 60% of the habitat found in China.
Protection Snow leopard, whose numbers are dwindling worldwide, has been
categorized as “vulnerable” in the International Union for Conservation of
Nature Red List. It is listed under Schedule I of the Wildlife (Protection)
Act, 1972.
Threats These majestic and elusive cats face many threats including habitat loss
and degradation from climate change and human encroachment,
retaliatory killings resulting from human-wildlife conflict, reduced prey,
and poaching.

Ganges River Dolphin


About The Ganges river dolphin can only live in freshwater and is essentially
blind. They hunt by emitting ultrasonic sounds, which bounce off of fish
and other prey, enabling them to “see” an image in their mind. They are
frequently found alone or in small groups, and generally a mother and
calf travel together.
Distribution Found throughout the Ganges-Brahmaputra-Meghna and Karnaphuli-
Sangu river systems of Nepal, India, and Bangladesh, the Ganges River
Dolphin is a global priority and is also an indicator of healthy aquatic
systems.
Protection The GRDs have been designated the National Aquatic Animal of India
since 2010 and are listed as ‘Endangered’ under IUCN Red List
Assessments, Schedule I of the Indian Wildlife (Protection) Act (1972),
Appendix I of the Convention on International Trade in Endangered
Species of Wild Fauna and Flora (CITES).
Threats Dwindling populations can be attributed to wide-scale habitat
degradation from pollution, hydroelectric and development projects, and
industrial run-off, as well as accidental deaths via entanglement in
fishing nets or by villagers from curiosity, opportunistic poaching for
meat and oil in certain pockets of the country.

Q.7) Consider the following statements about the Vermin Species in India:
1. These are wild animals that are believed to be harmful to crops, farm animals, or carry diseases.
2. The request for the declaration of any wild animal as vermin is put forth by the concerned state
government before the Central government.
3. Once notified by the Central Government, a species will be considered vermin across all the states
of the country.
Which of the statements given above is/are correct?
(a) 1 only
(b) 1 and 2 only
(c) 2 and 3 only
(d) 1, 2 and 3
EXPLANATION:
 Vermin are pests or nuisance animals that spread diseases or destroy crops or livestock. Since
the term is defined about human activities, which species are included vary by region and
enterprise.
 Schedule V of Wildlife Protection act 1972, includes vermins i.e the animals which may be
hunted. e.g Common Crows, Fruit Bats, Nilgais, etc.
So, Statement 1 is correct.
 As per Section 62 of the Wildlife Protection Act, 1972, States can send a list of wild animals to
the Centre requesting it to declare them vermin for selective slaughter.
 The Central Government may by notification, declare any wild animal other than those
specified in Schedule I and part 11 of Schedule II of the law to be vermin for any area for a
given period of time. As long as the notification is in force such wild animals shall be included
in Schedule V of the law, depriving them of any protection under that law.
 Any animal which poses a threat to humans and their livelihood especially farming can be
declared Vermin under Schedule V of Wildlife Protection act 1972.
 Vermin species can be hunted by anyone within the specified territories of the states in which
they have been declared as vermin.
 Those who kill vermin animals will not be subject to the jail terms and fines that hunting these
animals typically invites.
So, Statement 2 is correct and Statement 3 is not correct.
ADDITIONAL INFORMATION:
SCHEDULES OF WILDLIFE PROTECTION ACT 1972
 Wildlife laws divide species into schedules ranked from I to VI.
 Species listed in Schedule I and part II of Schedule II of WPA are provided absolute protection
and offenses under these are prescribed the highest penalties. e.g Gangetic Dolphin, Clouded
Leopard, Sloth bear, etc.
 Species listed in Schedule III and Schedule IV are also protected, but the penalties are much
lower. e.g Chital, Indian porcupine, etc.
 Schedule V includes vermins i.e the animals which may be hunted. e.g Common Crows, Fruit
Bats, Nilgais, etc.
 The plants in Schedule VI are prohibited from cultivation and planting. e.g Pitcher plant, Red
Vanda, etc.

Q.8) The routes that migratory water birds traverse on an annual basis are known as ‘flyways’ and there
are nine major flyways around the world. In these, India comes under which one of the following?
1. East Asian Australian Flyway
2. Central Asian Flyway
3. West Asian East African Flyway
Select the correct answer using the code given below.
(a) 2 only
(b) 1 and 2 only
(c) 1 only
(d) 1, 2 and 3
EXPLANATION:
The East Asian - Australasian Flyway (EAAF) stretches from the Russian Far East and Alaska,
southwards through East Asia and South-east Asia, to Australia and New Zealand and
encompasses 22 countries. The EAAF is home to over 50 million migratory waterbirds from over
250 different populations, including 36 globally threatened species and 19 Near Threatened
species.
It includes eastern India and the Andaman and Nicobar Islands.
So, Statement 1 is correct.
The Central Asian Flyway (CAF) covers a large continental area of Eurasia between the Arctic and
Indian Oceans and the associated island chains. The Flyway comprises several important
migration routes of waterbirds, most of which extend from the northernmost breeding grounds in
the Russian Federation (Siberia) to the southernmost non-breeding (wintering) grounds in West
and South Asia, the Maldives, and British Indian Ocean Territory. The birds on their annual
migration across the borders of several countries. Geographically the flyway region covers 30
countries of North, Central, and South Asia and Trans-Caucasus.

It covers almost entire India.


So, Statement 2 is correct.
The West Asian–East African Flyway is a group of well-established routes by which many species of
birds migrate annually between mid-Palearctic breeding grounds in Asia and non-breeding sites in
eastern and southern Africa.
It covers parts of North-western India.
So, Statement 3 is correct.

ADDITIONAL INFORMATION:
Flyways

About The routes that migratory waterbirds traverse on an annual basis are
known as 'flyways.' There are nine major flyways around the world. During
migration, waterbirds rely on a system of highly productive wetlands to
rest and feed, building up sufficient energy to fuel the next phase of their
journey. International cooperation across their migratory range is therefore
essential to conserve and protect migratory waterbirds and the habitats on
which they depend.
Major Flyways 1. East Atlantic Flyway
2. Black sea/ Mediterranean Flyway
3. Mississippi Americas Flyway
4. Atlantic Americas Flyway
5. Pacific Americas Flyway
6. West Pacific Flyway
7. East-Asian Australian Flyway
8. Central Asian Flyway
9. West Asian- East African Flyway

Q.9) Consider the following statements about the India State of Forest Report (ISFR) 2021:
1. Andhra Pradesh, Telangana, Odisha, Karnataka and Jharkhand contributed the most to the
national gain in forest cover.
2. Five states in the Northeast – Arunachal Pradesh, Manipur, Meghalaya, Mizoram and Nagaland
have shown a loss in forest cover.
3. As per this report, all the States and Union Territories has more than 33% of their area under
forest cover.
4. For the first time, forest cover in tiger reserves, tiger corridors and the Gir forest have been
assessed.
Which of the statements given above are correct?
(a) 1, 2 and 3 only
(b) 1, 2 and 4 only
(c) 1, 3 and 4 only
(d) 2, 3 and 4 only

EXPLANATION:
According to the ISFR, 2021, the top five states in terms of increase in forest cover are Andhra
Pradesh, Telangana, Odisha, Karnataka and Jharkhand. So, Statement 1 is correct.
States showing a loss in forest cover are Arunachal Pradesh, Manipur, Nagaland, Mizoram and
Meghalaya. However, these states from the Northeastern region of the country have the highest
percentage of forest cover with respect to the total geographical area of the state. So, Statement 2 is
correct.
India has set a target of bringing 33% of its geographical area under forest cover as envisaged in the
National Forest Policy, 1988. However, only a total of 17 states/union territories’ have above 33% of
the geographical area under forest cover. So Statement 3 is not correct.
ISFR 2021 assessed forest cover in the Tiger Reserves and Lion Conservation Area (Gir Forest) for the
first time. Of 52 tiger reserves, the Kawal, Bhadra and the Sundarbans reserves have shown the
maximum loss. So, Statement 4 is correct.

ADDITIONAL INFORMATION:
Why in  The Ministry of Environment, Forests and Climate Change (MoEFCC) has
News? released the India State of Forest Report (ISFR) 2021.
India State of  It is an assessment of India’s forest and tree cover.
Forest Report  It is published every 2 years by the Forest Survey of India under the MoEFCC.
 The first survey was published in 1987.
 Data is computed through wall-to-wall mapping of India’s forest cover through
remote sensing techniques.
 ISFR is used in the planning and formulation of policies in forest management
as well as forestry and agroforestry sectors
Key Findings  The Total Forest and Tree cover is 24.62% of the geographical area of the
country.
 As compared to ISFR 2019 the current assessment shows an increase of
0.28% of forest and tree cover put together, at the national level.
 There is a slight increase in the Recorded forest Area/Green Wash (RFA/GW)
as compared to the previous assessment of 2019.
 According to the ISFR, 2021, the top five states in terms of increase in forest
cover are Andhra Pradesh, Telangana, Odisha, Karnataka and Jharkhand.
 States showing a loss in forest cover are Arunachal Pradesh, Manipur,
Nagaland, Mizoram and Meghalaya.
 However, these states from the Northeastern region of the country have the
highest percentage of forest cover with respect to the total geographical area of
the state.
 The Mangrove cover in the country has increased by 0.34% as compared to
the previous assessment.
 The states that show significant gain in Mangrove cover are Odisha and
Maharashtra.
 There is a decrease of 10,594 sq km in the bamboo bearing area as compared
to the estimate of ISFR 2019.
What’s new  For the first time forest cover in tiger reserves, tiger corridors and the Gir
in ISFR forest which houses the Asiatic lion has been assessed.
2021?  Forest cover has increased in 20 tiger reserves in these 10 years and
decreased in 32.
 Buxa, Anamalai and Indravati reserves have shown an increase in forest cover
while the highest losses have been found in Kawal, Bhadra and the
Sunderbans reserves.
 Pakke Tiger Reserve in Arunachal Pradesh has the highest forest cover (nearly
97%).

 A new chapter on ‘Mapping of climate change hotspots in Indian forests’,


considering the role of forests in relation to climate change by reducing the
emission of greenhouse gases (GHGs).
 To identify climatic hotspots in Indian forests two computer-based models
were used i.e. RCP 8.5, where RCP 4.5 represents moderate emission
scenarios and RCP 8.5 represents the highest emission scenario or worst-case
scenario.
Shortcomings  Plantations – such as coffee, coconuts or mango are included under forest
in the Survey cover.
 Forest survey is carried out as an assessment of India’s biodiversity. Such an
overarching survey does not meet that objective, experts say.

Q.10) Of the endemic species in India, the largest number of species are located in the
(a) Himalayas
(b) Peninsular region
(c) Andaman Nicobar Islands
(d) Lakshadweep
EXPLANATION:
India contains two of the 25 biodiversities “hotspots”, the Western Ghats and Eastern
Himalayas, with thousands of endemic flora and fauna. The Indian flora and fauna represent
nearly 12 and 8 per cent of the global floral and faunal diversity respectively. The floristically
rich, India has about 141 endemic genera belonging to over 47 families of higher plants. As per
the Botanical Survey of India (BSI), India has 46,214 plants species. Of these, about 17,500
(7,000 species in the northeast region alone) represent flowering plants (7% of the world flora);
37 % of them are endemic. Of the endemic species (5725), the largest number (about 3471) of
species are located in the Himalayas followed by the peninsular region (2051 species) and
Andaman Nicobar Islands (239 species). So, Option (a) is correct.

ADDITIONAL INFORMATION:
Biodiversity
Biodiversity The variability among living organisms from all sources including, inter
alia, terrestrial, marine and other aquatic ecosystems and the ecological
complexes of which they are part; this includes diversity within species,
between species and of ecosystems
Species Diversity Species diversity refers to an enormous number of individuals belonging
to interacting groups or populations of distinct species or sub-species.
The number of species in a region - its ‘species richness’ is often used as
a measure of species diversity.
Genetic Diversity Genetic diversity refers to the variation in terms of genetic make-up
within species, which is transferred across generations. The number of
possible combinations of genes and the molecules making up the genes is
immense.
Ecosystem This refers to the variety of associations among organisms and with the
Diversity environment for utilising resources. Ecosystems differ in the species
composition of their communities (association of species), and also in
their physical structures.
Endemic Species Endemic species are plants and animals that exist only in one geographic
region. Species can be endemic to large or small areas of the earth.
Q.11) Consider the following statements about the Off-budget Borrowings in India.
1. Loans that public sector undertakings were supposed to take on behalf of the Government and
Deferred payments of bills and loans by the Centre are included in this.
2. Such borrowings are a way for the Centre to finance its expenditures while keeping the debt off the
books so that it is not counted in the calculation of fiscal deficit.
Which of the statements given above is/are correct?
(a) 1 only
(b) 2 only
(c) Both 1 and 2
(d) Neither 1 Nor 2
EXPLANATION:
According to the last Budget documents, in the current financial year, the Centre was set to
borrow Rs 5.36 lakh crore. However, this figure did not include the loans that public sector
undertakings were supposed to take on their behalf or the deferred payments of bills and
loans by the Centre. These items constitute the “off-budget borrowings” because these loans
and deferred payments are not part of the fiscal deficit calculation. So, Statement 1 is
correct.
Off-budget borrowings are a way for the Centre to finance its expenditures while keeping
the debt off the books — so that it is not counted in the calculation of fiscal deficit. Such
borrowings are used to fulfil the government’s expenditure needs. So, Statement 2 is
correct.

ADDITIONAL INFORMATION:
Fiscal Deficit It is essentially the gap between what the central government spends and
what it earns. In other words, it is the level of borrowings by the Union
government. This number is the most important metric to understand the
financial health of any government’s finances.
Importance of Low One of the most sought after details in any Union Budget is the level of
Fiscal Deficit fiscal deficit. As such, it is keenly watched by rating agencies — both
inside and outside the country. That is why most governments want to
restrict their fiscal deficit to a respectable number.
Off-budget Off-budget borrowings are a way for the Centre to finance its expenditures
borrowings while keeping the debt off the books - so that it is not counted in the
calculation of fiscal deficit.
Off-budget borrowings are loans that are taken not by the Centre directly,
but by another public institution that borrows in the directions of the
central government. Such borrowings are used to fulfil the government’s
expenditure needs.
Raising of Funds The government can ask an implementing agency to raise the required
funds from the market through loans or by issuing bonds. For example,
food subsidy is one of the major expenditures of the Centre. In the Budget
presentation for 2020-21, the government paid only half the amount
budgeted for the food subsidy bill to the Food Corporation of India. The
shortfall was met through a loan from the National Small Savings Fund.
This allowed the Centre to halve its food subsidy bill from Rs 1,51,000
crore to Rs 77,892 crore in 2020-21.
Institutes that  Other public sector undertakings have also been borrowed for the
borrowed for government. For instance, public sector oil marketing companies
Government were asked to pay for subsidised gas cylinders for Pradhan Mantri
Ujjwala Yojana beneficiaries in the past.
 Public sector banks are also used to fund off-budget expenses. For
example, loans from PSU banks were used to make up for the
shortfall in the release of fertiliser subsidies.

True Fiscal Deficit  If we consider the amount borrowed from the NSSF only for this
year, the fiscal deficit will go up by Rs 40,000 to Rs 50,000 crore in
absolute terms.
 In addition to the borrowings by PSUs, the actual liabilities of the
government would include loans taken for the recapitalisation of
banks and capital expenditures of the Ministries of Railways and
Power.
 Given the various sources of off-budget borrowing, the true debt is
difficult to calculate. For instance, it was widely reported that in
July 2019, just three days after the presentation of the Budget, the
CAG pegged the actual fiscal deficit for 2017-18 at 5.85% of GDP
instead of the government version of 3.46%.

Q.12) Consider the following statements about the Intercontinental Ballistic Missile (ICBM).
1. It is a land-based, nuclear-armed ballistic missile with a range of more than 5,600 km.
2. Agni-5 Missile is a nuclear-capable long-range Surface to Surface Intercontinental Ballistic Missile
(ICBM).
3. The possession of an ICBM missile itself is enough for a country to be declared as having the
capability of the Nuclear Triad.
Which of the statements given above is/are correct?
(a) 1 only
(b) 1 and 2 only
(c) 2 and 3 only
(d) 1, 2 and 3
EXPLANATION:
An intercontinental ballistic missile (ICBM) is a land-based missile with a minimum range
of 5,500 kilometres primarily designed for nuclear weapons delivery (delivering one or more
thermonuclear warheads). So, Statement 1 is correct.
Agni-5 is a solid fuelled and nuclear-capable intercontinental ballistic missile (ICBM)
developed by the Defence Research and Development Organisation. The missile is believed
to have a range of around 5,500 to 8,000 km. So, Statement 2 is correct.
A nuclear triad is a three-pronged military force structure that consists of land-launched
nuclear missiles, nuclear-missile-armed submarines, and strategic aircraft with nuclear
bombs and missiles. Specifically, these components are land-based intercontinental
ballistic missiles (ICBMs), submarine-launched ballistic missiles (SLBMs), and strategic
bombers. So, Statement 3 is not correct.
ADDITIONAL INFORMATION:
Recently, the Defence Research and Development Organisation (DRDO) successfully test-fired the new
generation nuclear-capable ballistic missile ‘Agni Prime’.
Nuclear Triad  Both the United States and Russia have had the strongest and longest-
living nuclear triads. In recent times, Countries including China, India,
and France have developed nuclear triads.
These triads include the following components:
 Bomber aircraft: Aircraft carrying nuclear bombs, or nuclear-armed cruise
missiles, for use against ground or sea targets.
 Land-based missiles (MRBMs or ICBMs): Delivery vehicles powered by a
liquid or solid-fueled rocket that primarily travel in a ballistic (free-fall)
trajectory.
 Ballistic missile submarines (SSBNs): Nuclear missiles launched from
ships or submarines. They are classified under the umbrella of vessels
and submarines that are capable of launching a ballistic missile.
Intercontinental  Intercontinental Ballistic Missiles (ICBMs) allow for a long-range strike
Ballistic Missiles launched from a controlled environment.
(ICBMs)  These missiles can also be launched, and reach targets, faster than
the other legs of the triad.
 Advantage: ICBMs are known as the most immediate leg of the triad. It
offers militaries the ability to launch a nuclear attack more quickly
than the other two options.
 Disadvantage: ICBMs do not contribute as much nuclear deterrence as
bombers or submarines because they cannot be forward-deployed in a
particular location.

India’s Policy  India's nuclear weapons policy is that of "no first use" and "minimum
credible deterrence," which means that the country will not use nuclear
weapons unless they are attacked first, but the country does have the
capability to induce the second strike.
India’s Arsenal  India's land-based arsenal includes the Prithvi-1 with a range of 150 to
600 kilometres, the Agni-1 with a range of 700 kilometres, the Agni-2 with
a range of 2,000 kilometres, the Agni-3 with a range of 3,000, the Agni-4
with a range of 3,500 kilometres, and the Agni-5 with a range of 5,000
kilometres.
 These are all intermediate-range ballistic missiles, but the Agni-5 stands
on the border between intermediate and intercontinental ballistic missiles.
 An intermediate-range ballistic missile has a range of 3,000 to 5,500
kilometres and the ability to travel farther than 5,500 kilometres.
 India completed its nuclear triad with the commissioning of INS Arihant in
August 2016, which was India's first submarine built indigenously.
 INS Arihant is a nuclear-powered ballistic missile submarine armed with
12 K-15 missiles with a range of 750 km, which will later be upgraded K-4
missiles with an extended range of 3500 km.
 The INS Arihant was the first SSBN to be completed under India's
program.
Q.13) China shares its 22,457-km land boundary with 14 countries. Which one of the following countries
has the maximum share in that part?
(a) Russia
(b) Mongolia
(c) India
(d) Myanmar
EXPLANATION:
Mongolia (4,677), Russia (3,645) and India (3,488) are the three largest border sharing
countries with china. While Afghanistan shares the smallest border with China,
Mongolia shares a border of 4677 km with China, the longest for both countries. So,
Option (b) is correct.
ADDITIONAL INFORMATION:

Q.14) Consider the following statements about the registration of Political parties in India.
1. In India, every party must be get registered with the Election Commission of India under the
Representation of the People Act, 1951.
2. The candidates of the registered political party will get preference in the matter of allotment of free
symbols over independent candidates.
3. The registered political parties can get recognition as a ‘state party’ or a ‘national party’ subject to
the fulfilment of the conditions prescribed in the Election Symbols (Reservation and Allotment)
Order, 1968
Which of the statements given above are correct?
(a) 1 and 2 only
(b) 1 and 3 only
(c) 2 and 3 only
(d) 1, 2 and 3
EXPLANATION:
It is not mandatory to register with the Election Commission but registering as a political
party with the EC has its advantage in terms of intending to avail itself of the provisions of
the Representation of the People Act, 1951, (relating to registration of political parties). So,
Statement 1 is not correct.
The candidates set up by a political party registered with the EC will get preference in the
matter of allotment of free symbols vis-à-vis purely independent candidates. More
importantly, these registered political parties, over course of time, can get recognition as a
‘state party’ or a ‘national party’ subject to the fulfilment of the conditions prescribed by
the Commission in the Election Symbols (Reservation and Allotment) Order, 1968. So,
Statements 2 and 3 are correct.

ADDITIONAL INFORMATION:
Registering a  According to the Election Commission, any party seeking
political party registration has to apply to the Commission within 30 days
following the date of its formation.
 The registration of all political parties is governed by the provisions
of Section 29A of the Representation of the People Act, 1951.
Process  As per existing guidelines, the applicant is asked to publish a
proposed party name in two national daily newspapers and two
local daily newspapers, and
 Provide two days for submitting objections, if any, with regard to
the proposed registration of the party before the Commission within
30 days from the publication.
 The notice for publication is also displayed on the website of the
Election Commission
 It also needs to have the latest electoral rolls in respect of at least
100 members of the party to show that they are registered, electors.
Why  It is not mandatory to register with the Election Commission.
registering  Registering as a political party with the EC has its advantage in
with the EC is terms of intending to avail itself of the provisions of the
important Representation of the People Act, 1951.
 The candidates set up by a political party registered with the EC
will get preference in the matter of allotment of free symbols vis-à-
vis purely independent candidates.
 More importantly, these registered political parties, over course of
time, can get recognition as a ‘state party’ or a ‘national party’
subject to the fulfilment of the conditions prescribed by the
Commission in the Election Symbols (Reservation and Allotment)
Order, 1968.
 Recognised ‘state’ and ‘national’ parties need only one proposer for
filing the nomination and are also entitled to two sets of electoral
rolls free of cost and broadcast/telecast facilities over state-owned
Akashvani/Doordarshan during the general elections.
How EC There are several conditions that the Commission follows to classify these
recognises a parties. Those are:
political party  A party should be engaged in political activity for a continuous
as a state or period of five years.
national party  At least one member to the house of the people (LS) for every
twenty-five members of that House or any fraction of that number
from that state; or
 At least one member to the Legislative Assembly of that state (SLA)
for every thirty members of that assembly or any fraction of that
number.
 The “total number of valid votes polled by all the contesting
candidates set up by such party at the last general election in the
state to the house of the people, or Legislative Assembly of the
state, is not less than six per cent of the total number of valid votes
polled by all the contesting candidates at such general election in
the state.
 For recognition of a ‘national party’ if a political party is treated as a
recognised political party in four or more states, only then will it be
recognised as a `national party’ throughout the whole of India, but
it will only be continued to be categorised as one as long as that
political party continues to fulfil such conditions again.
 If a political party is treated as a recognised political party in less
than four states, it will be a `state party’ in the state but only so
long as that political party continues to fulfil such conditions from
time to time again.

Q.15) Consider the following statements about theNarcotic Drugs and Psychotropic Substances Act,
1985.
1. It is designed to fulfil India's treaty obligations under the Single Convention on Narcotic Drugs,
Convention on Psychotropic Substances, and United Nations Convention Against Illicit Traffic in
Narcotic Drugs and Psychotropic Substances.
2. It extends to the whole of India and it applies also to all Indian citizens outside India and to all
persons on ships and aircraft registered in India.
3. It is a stringent law where the death penalty can be prescribed for repeat offenders.
Which of the statements given above is/are correct?
(a) 1 only
(b) 1 and 2 only
(c) 2 and 3 only
(d) 1, 2 and 3

EXPLANATION:
The NDPS Act, 1985 is designed to fulfil India's treaty obligations under the Single
Convention on Narcotic Drugs, Convention on Psychotropic Substances, and United
Nations Convention Against Illicit Traffic in Narcotic Drugs and Psychotropic Substances.
Also, the Narcotics Control Bureau (NCB) was set up under the act with effect from March
1986. So, Statement 1 is correct.
The NDPS Act has since been amended four times in 1988, 2001, 2014 and 2021. The Act
extends to the whole of India and it applies also to all Indian citizens outside India and to
all persons on ships and aircraft registered in India. So, Statement 2 is correct.
In NDPS Amendment Act, 2014 Section 31 and 31A were also amended with certain
modifications, including making “death sentence” optional. So, it is a stringent law where
the death penalty can be prescribed for repeat offenders. So, Statement 3 is correct.
ADDITIONAL INFORMATION:
The Narcotic Drugs and Psychotropic Substances (Amendment) Bill, 2021 was introduced in Lok Sabha on
December 6, 2021.
Narcotic  The Act regulates certain operations such as manufacture, transport
Drugs and and consumption related to narcotic drugs and psychotropic
Psychotropic substances.
Substances  It is designed to fulfil India's treaty obligations under the Single
Act, 1985 Convention on Narcotic Drugs, Convention on Psychotropic
Substances, and United Nations Convention Against Illicit Traffic in
Narcotic Drugs and Psychotropic Substances.
 The Act extends to the whole of India and it applies also to all Indian
citizens outside India and to all persons on ships and aircraft
registered in India.
Offences and  Under the Act, financing certain illicit activities such as cultivating
Punishments cannabis, manufacturing narcotic drugs or harbouring persons
engaged in them is an offence.
 Persons found guilty of this offence will be punished with rigorous
imprisonment of at least 10 years - extendable up to 20 years - and
a fine of at least Rs.1 lakh.
 It also provides for forfeiture of property derived from or used in,
illicit traffic in narcotic drugs and psychotropic substances.
 It also provides for the death penalty in some cases where a person
is a repeat offender.
Narcotics  The Narcotics Control Bureau (NCB) is the Indian central law
Control enforcement and intelligence agency under the Ministry of Home
Bureau Affairs, Government of India.
 The agency is tasked with combating drug trafficking and the use of
illegal substances under the provisions of the Narcotic Drugs and
Psychotropic Substances Act, 1985.
 The Narcotics Control Bureau's national headquarters is located in
Delhi.
 The Narcotics Control Bureau is also represented on the Economic
Intelligence Council.
 The NCB is outside the ambit of the Right to Information Act under
Section 24(1) of the RTI act 2005.
Narcotic  The Act would replace an ordinance promulgated earlier this year
Drugs and (2021) to correct a drafting error in a 2014 amendment to the Act.
Psychotropic  Before the 2014 amendment, clause (viii-a) of Section 2 contained
Substances sub-clauses (i) to (v), which defined the term “illicit traffic”.
(Amendment)  Concerns Related to the Act, the new provision is giving retrospective
Act, 2021 effect from 1st May 2014.
 That means a criminal provision is given, which will not hold in good
law.
 It also violates the fundamental rights in Article 20 because a person
can be punished for an offence for which there is a law in existence
at the time of the commission of the offence.
Q.16) Which one of the following statements is correct about the DeeporBeel Wildlife Sanctuary, recently
seen in the news?
(a) It is the only tiger reserve in the North-Eastern part of India
(b) It is Assam’s only Ramsar site besides being an Important Bird Area
(c) It is the new tiger reserve notified in the Western Ghats region
(d) It is Rajasthan’s only Important Bird Area as well as the Ramsar site
EXPLANATION:
 The Ministry of Environment, Forests, and Climate change has notified the Eco-sensitive region
of the DeeporBeel Wildlife Sanctuary on the southwestern edge of the Guwahati.
 Deepor Beel (Beel means wetland or large aquatic body in Assamese) located about 10 km
Southwest of Guwahati city is considered one of the large and important riverine wetlands in
the Brahmaputra Valley of lower Assam, India. And the state Deepor Beel is an open lake basin
connected with a set of inflow and outflow channels.
 Deepor Beel has both biological and environmental importance besides being the only major
storm-water storage basin for Guwahati city. It is considered one of the staging sites for
migratory birds in India; and some of the large congregations of aquatic birds in Assam during
winter.
 Because of the richness of avian fauna it enjoyed, Deepor Beel has been selected as one of the
Important Bird Area (IBA) sites by Birdlife International. Deepor Beel has also been designated
as a Ramsar Site in November 2002 and the state’s only Ramsar site.
So, option (b) is correct.

ADDITIONAL INFORMATION:
CONCERNS  Its area has been long-suffering from environmental degradation due to
REGARDING encroachment. Once spread over 4,000 hectares, it has now shrunk to
DEEPORBEEL 500 hectares. It is home to over 200 species of migratory birds.
 Environmentalists have often alleged that wastes dumped on the
wetland area have turned the lake’s water toxic.
 The lake is bearing the brunt of adverse human activities such as filling
wetlands for habitation, cutting sides of wetlands, fishing, the killing of
migratory birds, etc.
 The degradation of water quality, sedimentation in the lake’s surface,
and deforestation activities in and around the water body called for its
conservation.

Q.17) Consider the following statements:


1. Whenever new species in India were discovered, the vertebrates constitute the majority of
discovery.
2. Whenever new species among the invertebrates were discovered in India, the insects constitute the
major portion.
3. In case of new species discovery among the vertebrates, Pisces and reptiles dominate the discovery.
Which of the statements given above is/are correct?
(a) 1 only
(b) 2 only
(c) 2 and 3 only
(d) 1 and 3 only
EXPLANATION:
According to Animal Discoveries 2020, a report published recently by the Zoological Survey of India,
India has added 557 new species to its fauna. Of these 557 species, invertebrates constitute the
majority with 486 species, while 71 species belong to vertebrates. Among invertebrates, insects
dominated, with 344 species, whereas Pisces and reptiles dominated among vertebrates. So,
Statements 2 and 3 are correct.

ADDITIONAL INFORMATION:
What in News?  Recently Animal Discoveries 2020, a document published recently by the
Zoological Survey of India (ZSI) shows India has added 557 new species to its
fauna, which includes 407 new species and 150 new records.
 The number of faunal species in India has climbed to 1,02,718 species with
the discovery of the news species.
Animal  Prepared by the Zoological Survey of India (ZSI) and released by the
Discoveries Environment Ministry.
Report  It is a document on new species and new records of fauna.
 Animal Discoveries are the only authentic source of faunal discoveries of
India.
 Since 2009, It is published annually.
Important  Among the new species, some interesting species discovered in 2020 are-
Findings  Trimeresurus Salazar, is a new species of green pit viper discovered from
Arunachal Pradesh.
 Lycodon Deccanensis, the Deccan wolf snake discovered from Karnataka.
 Sphaerotheca Bengaluru, is a new species of burrowing frog named after the
city of Bengaluru.
 Xyrius Anjalai, is a new deepwater species of snake eel from Kerala.
 Glyptothorax Giudikyensis, a new species of catfish from Manipur and
 Clyster Galateansis, a new species of scarab beetles from the Great Nicobar
Biosphere.
State Wise  Among the States, the highest number of new species were discovered from
Records Karnataka (66 species), followed by Kerala (51 species).
 Also, 46 new species were discovered from Rajasthan and 30 from West
Bengal.
 In terms of new records or species recorded in the country for the first time,
Arunachal Pradesh had the highest (20 new records).
 In the Andaman & Nicobar Islands, 25 new species were discovered and 16
new records were documented.
Zoological  Set up by British zoologist Thomas Nelson Annandale in 1916, the ZSI is
Survey of India India’s apex organization on animal taxonomy.
 It is a subordinate organization of the Environment Ministry.
 Headquartered in Kolkata, the ZSI BSI has 16 regional circles in different
regions of the country.
 ZSI promotes survey, exploration and research leading to the advancement
of knowledge on many aspects of the exceptionally rich faunal diversity of
India.
 It has been declared as a designated repository for National Zoological
Collection under the National Biodiversity Act, 2002.
Q.18) Consider the following statements about the Leaded Petrol:
1. According to the United Nations Environment Program, only the developing countries in the world
are still using this leaded petrol.
2. It is still available in aviation fuel for small aeroplanes.
3. The combustion of leaded petrol causes the lead to be released into the air which causes heart
disease, stroke and cancer.
Which of the statements given above is/are correct?
(a) 1 only
(b) 2 and 3 only
(c) 3 only
(d) 2 only
EXPLANATION:
Recently, the United Nations Environment Programme (UNEP) announced that the use of leaded
petrol has been eradicated from the globe. In July 2021, Leaded Petrol was eradicated from Algeria,
which is the last country to use leaded petrol. So, Statement 1 is not correct.

Despite leaded gas being illegal for road use in cars, some aeroplanes with piston-based engines
uses this fuel for higher Octane number.
So, Statement 2 is correct.
Leaded petrol causes heart disease, stroke and cancer. It also affects the development of the human
brain, especially harming children. So, Statement 3 is correct.

ADDITIONAL INFORMATION:
Leaded  The main difference between leaded and unleaded fuel is the additive
Petrol tetraethyl lead.
 Lead is a heavy pollutant that does damage not only to the environment but
also to the people causing heart disease, stroke, and cancer etc.
Partnership  It was set up at the World Summit on Sustainable Development, 2002.
for Clean  UNEP hosted the Secretariat.
Fuels and  Its aim is to eliminate leaded petrol globally and provide support to many
Vehicles countries and regional initiatives.
(PCFV)  Many major economic powers had already stopped using the fuel, including
the United States, China and India before such a campaign was initiated.
Significance  On top of the climate crisis, such eradication of lead petrol is greater
of success towards achieving climate targets and SDG goals (Good health and
Eradication well-being (SDG3), Clean Water (SDG6), Clean Energy (SDG7), Sustainable
cities (SDG11), Climate Action (SDG13) and Life on land (SDG15).
United  The UNEP is a leading global environmental authority established on 5th
Nations June 1972 and headquartered in Nairobi, Kenya.
Environment  Promotes sustainable development within the United Nations system, and
Programme serves as an authoritative advocate for global environment protection.
 Major Reports: Emission Gap Report, Adaptation Gap Report, Global
Environment Outlook, Frontiers, Invest into Healthy Planet.
 Major Campaigns: Beat Pollution, UN75, World Environment Day, Wild for
Life.
Q.19) Consider the following statements about the Eco-Sensitive Zone (ESZ):
1. The concept of the ESZ zone is mentioned and clearly defined in the Environment Protection Act,
1986.
2. These are the areas notified around the Protected Areas, National Parks and Wildlife Sanctuaries.
3. It acts as a transition zone from areas of high protection to that of lesser protection.
4. An ESZ could go up to 10 km around a protected area as provided in the Wildlife Conservation
Strategy, 2002.
Which of the statements given above are correct?
(a) 1, 2 and 3 only
(b) 1, 2 and 4 only
(c) 1, 3 and 4 only
(d) 2, 3 and 4 only
EXPLANATION:
Eco-Sensitive Zones (ESZs) or Eco Fragile Areas (EFAs) are areas notified around the Protected Areas,
National Parks and Wildlife Sanctuaries by the MoEFCC under the Environment (Protection) Act, 1986.
But the word “Eco-Sensitive Zones” was not mentioned in this Act. So, Statement 1 is not correct and
Statement 2 is correct.
The purpose of declaring ESZs is to create some kind of "shock absorbers" to the protected areas by
regulating and managing the activities around such areas. They also act as a transition zone from areas
of high protection to areas involving lesser protection. So, Statement 3 is correct.
An ESZ could go up to 10 km around a protected area as provided in the Wildlife Conservation Strategy,
2002. In the case where sensitive corridors, connectivity and ecologically important patches are crucial
for landscape linkage are beyond 10 km width, they should be included in the ESZs. So, Statement 4
is correct.
ADDITIONAL INFORMATION:
What in News?  On August 17 2021, Government of India has recognised the reserve forest
area on the fringes of Nagarjunasagar Srisailam Tiger Reserve (NSTR) as
an Eco-Sensitive Zone (ESZs).
 On December 4 2021, Karnataka Chief Minister Basavaraj Bommai
informed the Centre that the state is opposed to the Kasturirangan
Committee report on Western Ghats.
Eco-Sensitive  Also known as Ecologically Fragile Areas (EFAs), ESZs are areas notified
Zones (ESZs) around the Protected Areas, National Parks and Wildlife Sanctuaries by
the MoEFCC under the Environment (Protection) Act, 1986.
 But the word “Eco-Sensitive Zones” was not mentioned in this Act.
 As per the National Board for Wildlife NBWL, the delineation of eco-
sensitive zones have to be site-specific, and the activities should be
regulative in nature and not prohibitive unless required.
 Rule 5(1) of the Environment (Protection) Act 1986 (EPA) states that the
MoEFCC can prohibit or restrict the location of industries and carry on
certain operations or processes on the basis of certain considerations.
Purpose  The purpose of declaring ESZs is to create some kind of "shock absorbers"
to the protected areas by regulating and managing the activities around
such areas.
 They also act as a transition zone from areas of high protection to areas
involving lesser protection.
Coverage  An ESZ could go up to 10 km around a protected area as provided in the
Wildlife Conservation Strategy, 2002.
 In the case where sensitive corridors, connectivity and ecologically
important patches are crucial for landscape linkage are beyond 10 km
width, they should be included in the ESZs.

Activities on  Permitted activities - Ongoing agricultural or horticultural practices,
Control rainwater harvesting, organic farming, use of renewable energy sources,
adoption of green technology for all activities.
 Regulated activities - Felling of trees, the establishment of hotels and
resorts, commercial use of natural water, erection of electrical cables,
drastic change of agriculture system, widening of roads.
 Prohibited activities - Commercial mining, sawmills, industries causing
pollution, the establishment of major hydroelectric projects (HEP). It also
prohibits commercial use of wood, Tourism activities, discharge of
effluents or any solid waste or production of hazardous substances.
Recommendations  The Kasturirangan committee report proposes 37 per cent of the total area
of the of Western Ghats, which is roughly 60,000 square kilometres, to be
Kasturirangan declared as eco-sensitive area (ESA).
Committee Report  The report recommended a blanket ban on mining, quarrying, setting up of
red category industries and thermal power projects.
 It also stated that the impact of infrastructural projects on the forest and
wildlife should be studied before permission is given for these activities.
 It also stated that the UNESCO Heritage tag is an opportunity to build
global and domestic recognition of the enormous natural wealth that exists
in the Western Ghats.
The Core Issue of  The Kasturirangan committee report has been prepared based on the
Non- satellite images, but the people of the region have adopted agriculture and
implementation of horticultural activities in an eco-friendly manner.
Report  However, non-implementation will also affect Western Ghats Ecosystem
and overall performance towards sustainability, climate change goals and
even the economy.

Q.20) Consider the following statements about the algae.


1. Algae are ubiquitous that they are inhabiting almost all the ecosystems across the world from the
Arctic to Antarctica.
2. The study of algae in any wetlands is essential for predicting and understanding the implications of
environmental variation, anthropogenic effects and climate changes.
3. Algal diversity is one of the highest in the Andaman and Nicobar Islands.
Which of the statements given above is/are correct?
(a) 1 only
(b) 1 and 2 only
(c) 2 and 3 only
(d) 1, 2 and 3
EXPLANATION:
Algae are polyphyletic photosynthetic eukaryotes ranging from microscopic unicellular to gigantic
kelps that inhabit the world’s oceans, freshwater bodies, soils, rocks and trees. They are
ubiquitous that inhabit almost all the ecosystems across the world from the Arctic to Antarctica.
So, Statement 1 is correct.
Algae occur in wetlands as both phytoplankton and periphyton. Periphyton is an integral part of
the microbiota in any wetlands. They are more ideal experimental tools to access any alterations
in the wetland ecosystem infrastructure due to their high sensitivity towards any changes in water
quality and nutrient availability. So, Statement 2 is correct.
The most important area along the coast of India where the marine algae occur in abundance is
Okha Port and Dwarka along the Gujarat Coast while among archipelagos around the Andaman
and Nicobar Islands in the east and the Lakshadweep in the west also support a number of
marine algae. So, Statement 3 is correct.

ADDITIONAL INFORMATION:
What in  Recently, a group of botanists has discovered an algal species with an
News? ‘umbrella head' from the Andaman and Nicobar Islands.
 It is the first species of the genus Acetabularia discovered in India.
 It is a bright green algae with a size as small as 20 to 40 mm
 However, like Coral Reefs, they also face the threat of sea levels rising due
to Global Warming and are highly prone to Ocean Acidification.

Algae  Algae are ubiquitous and inhabit almost all the ecosystems across the
world from the Arctic to Antarctica.
 A multitude of species ranging from microscopic unicellular to gigantic
kelps inhabits the world’s oceans, freshwater bodies, soils, rocks and trees.
 They are polyphyletic photosynthetic eukaryotes, which are microscopic as
well as macroscopic thallophytes lacking any sterile covering of cells
around reproductive organs and contain chlorophyll-a as a primary
photosynthetic pigment.
Significance  They are more ideal experimental tools to access any alterations in the
of Algae wetland ecosystem infrastructure due to their high sensitivity towards any
changes in water quality and nutrient availability.
 Essential for predicting and understanding the implications of
environmental variation, anthropogenic effects, and changes in climate as
well.
 Enormous economic implications not only as primary producers and
pollution indicators but also as a source of several natural products,
biofertiliser and chemicals.
Important  The most important area along the coast of India where the marine algae
Algal occur in abundance are:
Abundant  Okha Port and Dwarka along the Gujarat Coast
Areas Of  Karwar along the West coast
India  Chennai, Kovelonge, Mahabalipuram, Tuticorin and Cap comorine along
the Tamil Nadu coast
 The Islands of the Gulf of Mannar viz. Pamban and Rameshwaram and a
group of coral Islands. viz. Krushadai, Shingle and Pallivasal Island
 Visakhapatnam coast in Andhra Pradesh
 Sunderban of West Bengal.
 Among archipelagos around the Andaman and Nicobar Islands in the east
and the Lakshadweep in the west also support a number of marine algae.

Q.21) Which of the following statements about the National Clean Air Programme, is/are correct?
1. It aims to achieve a 20% to 30% reduction in Particulate Matter concentrations by 2024.
2. It is being implemented across India to reduce particulate matter levels in all the cities where the
population is above 1 lakhs.
3. It also aims to achieve a 20% reduction in all Green House Gas Emissions by 2030 as per the Paris
Climate Deal, 2015.
Select the correct answer using the code given below.
(a) 1 only
(b) 1 and 2 only
(c) 2 and 3 only
(d) 1and 3 only
EXPLANATION:
The Central Government launched National Clean Air Programme (NCAP) as a long-term,
time-bound, national-level strategy to tackle the air pollution problem across the country
in a comprehensive manner with targets to achieve 20% to 30% reduction in Particulate
Matter concentrations by 2024 keeping 2017 as the base year for the comparison of
concentration. So, Statement 1 is correct.

Under NCAP, 132 non-attainment cities (NA Cities) have been identified across the country
based on the Air Quality data from 2014-2018. The NA cities list is a heterogeneous mix of
cities of all sizes and types. Most major cities of Mumbai Metropolitan Region, Pune,
Nagpur, Nashik, Aurangabad, Kolhapur, etc figure in the NA cities list for poor air quality.
So, Statement 2 is not correct.

India’s headline Paris pledge was to reduce the emission intensity of its gross domestic
product (GHG emissions per unit GDP) by 33-35 percent over 2005 levels by 2030. This is
its commitment under Intended Nationally Determined Contribution (NDC) and not listed
in the aim of the National Clean Air Program. So, Statement 3 is not correct.
ADDITIONAL INFORMATION:
National Clean Air Program

Objective The Central Government launched National Clean Air


Programme (NCAP) as a long-term, time-bound, national-level
strategy to tackle the air pollution problem across the country
in a comprehensive manner with targets to achieve 20% to
30% reduction in Particulate Matter concentrations by 2024
keeping 2017 as the base year for the comparison of
concentration.
Targeted Cities Under NCAP, 122 non-attainment cities have been identified
across the country based on the Air Quality data from 2014-
2018. A list of Smart Cities under the Smart Cities Mission,
which was non-attainment cities under the National Clean Air
Program (NCAP).
Implementation The city-specific action plans have been prepared which, inter-
alia, include measures for strengthening the monitoring
network, reducing vehicular/industrial emissions, increasing
public awareness, etc. Implementation of the city-specific
action plans is regularly monitored by Committees at the
Central and State level namely Steering Committee,
Monitoring Committee, and Implementation Committee.
Monitoring The air quality of cities is monitored by State Pollution Control
Boards which publish their results from time to time. Some
Smart Cities have established Integrated Command and
Control Centres (ICCCs) which are also connected to Air
Quality Monitors (AQMs) for effective monitoring.
Q.22) Which of the following statements about the Plastic Waste Management Amendment Rules of 2021
is/are correct?
1. To stop littering due to lightweight plastic carry bags, the thickness of plastic carry bags has been
increased from fifty microns to seventy-five microns and one hundred and twenty microns.
2. The plastic packaging waste, which is not covered under the phase-out of identified single-use
plastic items, shall be collected and managed in an environmentally sustainable way through the
Extended Producer Responsibility of the Producer, importer and Brand owner (PIBO).
3. Earbuds with plastic sticks, plastic sticks for balloons, plastic flags, candy sticks, ice-cream sticks,
polystyrene [Thermocol] for decoration will be considered as single-use plastics in India.
Select the correct answer using the code given below.
(a) 1 only
(b) 1 and 2 only
(c) 2 and 3 only
(d) 1, 2 and 3
EXPLANATION:
To stop littering due to lightweight plastic carry bags, the thickness of plastic carry
bags has been increased from fifty microns to seventy-five microns and one
hundred and twenty microns. This will also allow the reuse of plastic carry bags
due to an increase in thickness. So, Statement 1 is correct.
The plastic packaging waste, which is not covered under the phase-out of identified
single-use plastic items, shall be collected and managed in an environmentally
sustainable way through the Extended Producer Responsibility of the Producer,
importer, and Brand owner (PIBO), as per Plastic Waste Management Rules, 2016.
For effective implementation of Extended Producer Responsibility, the Guidelines
for Extended Producer Responsibility being brought out have been given legal force
through Plastic Waste Management Amendment Rules, 2021. So, Statement 2 is
correct.
Under the new rules, the manufacture, sale, and use of some single-use goods
made with plastic, polystyrene, and expanded polystyrene, such as earbuds with
plastic sticks, plastic sticks for balloons, plastic flags, candy sticks, ice-cream
sticks, polystyrene [Thermocol] for decoration; plates, cups, glasses, cutlery such
as forks, spoons, knives, straw, trays, wrapping or packing films around sweet
boxes, invitation cards, and cigarette packets, plastic or PVC banners less than
100 microns, stirrers are prohibited. So, Statement 3 is correct.

ADDITIONAL INFORMATION:
Plastic Waste Management Amendment Rules of 2021
India’s Stand Pollution due to single-use plastic items has become an
important environmental challenge confronting all countries.
India is committed to taking action for the mitigation of pollution
caused by littered Single-Use Plastics. In the 4th United Nations
Environment Assembly held in 2019, India had piloted a
resolution on addressing single-use plastic products pollution,
recognizing the urgent need for the global community to focus
on this very important issue. The adoption of this resolution at
UNEA 4 was a significant step.
Objective Ministry of Environment, Forest and Climate Change,
Government of India, has notified the Plastic Waste Management
Amendment Rules, 2021, which prohibits identified single-use
plastic items which have low utility and high littering potential
by 2022.
Ban on SUP The manufacture, import, stocking, distribution, sale, and use of
the following single-use plastic, including polystyrene and
expanded polystyrene, commodities shall be prohibited with
effect from the 1st July 2022
 earbuds with plastic sticks, plastic sticks for balloons, plastic
flags, candy sticks, ice-cream sticks, polystyrene [Thermocol]
for decoration;
 plates, cups, glasses, cutlery such as forks, spoons, knives,
straw, trays, wrapping or packing films around sweet boxes,
invitation cards, and cigarette packets, plastic or PVC banners
less than 100 micron, stirrers.
Plastic Carry Bags To stop littering due to lightweight plastic carry bags, with effect
from 30th September 2021, the thickness of plastic carry bags
has been increased from fifty microns to seventy-five microns
and one hundred and twenty microns with effect from the 31st
December 2022. This will also allow the reuse of plastic carry
due to an increase in thickness.
Extended Producer The plastic packaging waste, which is not covered under the
Responsibility phase-out of identified single-use plastic items, shall be collected
and managed in an environmentally sustainable way through
the Extended Producer Responsibility of the Producer, importer,
and Brand owner (PIBO), as per Plastic Waste Management
Rules, 2016. For effective implementation of Extended Producer
Responsibility, the Guidelines for Extended Producer
Responsibility being brought out have been given legal force
through Plastic Waste Management Amendment Rules, 2021.
Other Initiatives The waste management infrastructure in the States/UTs is
being strengthened through the Swachh Bharat Mission. To
encourage innovation in the development of alternatives to
identified single-use plastic items and digital solutions to plastic
waste management, the India Plastic Challenge – Hackathon
2021, has been organized for students of Higher Educational
Institutions and startups recognized under Startup India
Initiative.
Q.23) Consider the following protected areas in India.
1. Thol Wildlife Sanctuary in Gujarat
2. Sultanpur National Park in Haryana
3. Bhindawas Wildlife Sanctuary in Haryana
4. Vadhavana Bird Sanctuary in Gujarat
By observing the above given protected areas, which one of the following statements can be concluded?
(a) New Ramsar sites in India
(b) New Important Bird Areas in India
(c) New Elephant corridors in India
(d) New Tiger reserves in India
EXPLANATION:
Four more Indian sites - two each from Haryana and Gujarat - have been
recognized as wetlands of international importance under the Ramsar Convention,
taking the number of such sites in the country to 46. For the first time, two
wetlands in Haryana - Sultanpur National Park in Gurgaon and Bhindawas Wildlife
Sanctuary in Jhajjar and Thol and Wadhwana from Gujarat have been included in
the Ramsar list. So, Option (a) is correct.

ADDITIONAL INFORMATION:
Ramsar Sites
Convention on The Convention on Wetlands is an intergovernmental treaty that
Wetlands provides the framework for national action and international
cooperation for the conservation and wise use of wetlands and
their resources. It is named after the Iranian city of Ramsar, on
the Caspian Sea, where the treaty was signed on February 2,
1971.
Wetlands Definition The Convention uses a broad definition of wetlands. It includes all
lakes and rivers, underground aquifers, swamps and marshes,
wet grasslands, peatlands, oases, estuaries, deltas, and tidal flats,
mangroves, and other coastal areas, coral reefs, and all human-
made sites such as fish ponds, rice paddies, reservoirs, and salt
pans.
Data of Wetlands  Number of Contracting Parties: 172
 Number of Wetlands of International Importance: 2,435
 The total surface of designated sites: 254,685,425 ha
Wetlands in India India currently has 47 sites designated as Wetlands of
International Importance (Ramsar Sites), with a surface area of
1,090,230 hectares.
Recent Places Bhindawas  Haryana’s Bhindawas Wildlife
Wildlife Sanctuary is a human-made freshwater
Sanctuary wetland. It is also the largest in
Haryana. Over 250 bird species use the
sanctuary throughout the year as a
resting and roosting site.
 The site supports more than 10 globally
threatened species including the
endangered Egyptian Vulture, Steppe
Eagle, Pallas’s Fish Eagle, and Black-
bellied Tern.
Sultanpur  The Sultanpur National Park in
National Haryana supports more than 220
Park species of resident, winter migratory
and local migratory waterbirds at
critical stages of their life cycles.
 More than 10 of these are globally
threatened, including the critically
endangered sociable lapwing, and the
endangered Egyptian Vulture, Saker
Falcon, Pallas’s Fish Eagle, and Black-
bellied Tern.
Thol Lake  The Thol Lake Wildlife Sanctuary in
Wildlife Gujarat lies on the Central Asian Flyway
Sanctuary and more than 320 bird species can be
found here.
 The wetland supports more than 30
threatened waterbird species, such as
the critically endangered White-rumped
Vulture and Sociable Lapwing, and the
vulnerable Sarus Crane, Common
Pochard, and Lesser White-fronted
Goose.
Wadhvana  The Wadhvana Wetland in Gujarat is
Wetland internationally important for its birdlife
as it provides wintering ground to
migratory waterbirds, including over 80
species that migrate on the Central
Asian Flyway.
 They include some threatened or near-
threatened species such as the
endangered Pallas’s fish-Eagle, the
vulnerable Common Pochard, and the
near-threatened Dalmatian Pelican,
Grey-headed Fish-eagle and
Ferruginous Duck.
Q.24) Consider the following statements about the Environment (Protection) Act of 1986:
1. It specifically mentions fuels or emission standards to be followed in India.
2. The Central Pollution Control Boardhas been conferred under this Act to monitor and control the
air pollution levels.
3. It authorizesthe central and state governments to regulate activities that can harm the
environment.
Which of the statements given above is/are correct?
(a) 3 only
(b) 1 and 2 only
(c) 1 and 3 only
(d) 1 only
EXPLANATION:
The Environment (Protection) Rules lay down general procedures for setting standards of emission or
discharge of environmental pollutants, but it does not particularly mention the fuels or emission
standards to be followed in India whereas various environmental bodies and the rules will lay down
the specific emission rules regularly. So, Statement 1 is not correct.
The Central Pollution Control Board (CPCB), the statutory organization, was constituted in September
1974 under the Water (Prevention and Control of Pollution) Act, 1974. Further, CPCB was entrusted
with the powers and functions under the Air (Prevention and Control of Pollution) Act, 1981.
Principal Functions of the CPCB, as spelled out in the Water (Prevention and Control of Pollution) Act,
1974, and the Air (Prevention and Control of Pollution) Act, 1981,
(i) to promote cleanliness of streams and wells in different areas of the States by prevention,
control, and abatement of water pollution, and
(ii) to improve the quality of air and to prevent, control, or abate air pollution in the country.
So, Statement 2 is not correct.
The Environment protection act is an umbrella legislation designed to provide a framework for the
coordination of central and state authorities established under the water and air prevention control
acts. The Central Government may, about its functions under this Act, from time to time, require any
person, officer, State Government or other authority to furnish to it or any prescribed authority or
officer any reports, returns, statistics, accounts, and other information and such person, officer, State
Government or other authority shall be bound to do so. So, Statement 3 is correct.

ADDITIONAL INFORMATION:
ENVIRONMENTAL PROTECTION ACT, 1986
 The Environment (Protection) Act, 1986 authorizes the central government to protect and
improve environmental quality, control and reduce pollution from all sources, and prohibit or
restrict the setting and /or operation of any industrial facility on environmental grounds.
 The Environment (Protection) Act was enacted in 1986 to provide for the protection and
improvement of the environment. It empowers the Central Government to establish authorities
charged with the mandate of preventing environmental pollution in all its forms and to tackle
specific environmental problems that are peculiar to different parts of the country. The Act was
last amended in 1991.
 The Environment (Protection) Rules lay down procedures for setting standards of emission or
discharge of environmental pollutants.
 The objective of Hazardous Waste (Management and Handling) Rules, 1989 is to control the
generation, collection, treatment, import, storage, and handling of hazardous waste.
 The Manufacture, Storage, and Import of Hazardous Rules define the terms used in this
context and set up an authority to inspect, once a year, the industrial activity connected with
hazardous chemicals and isolated storage facilities.
 The Manufacture, Use, Import, Export, and Storage of hazardous Micro-organisms/ Genetically
Engineered Organisms or Cells Rules,1989 were introduced to protect the environment, nature,
and health, in connection with the application of gene technology and micro-organisms.

Q.25) The world’s first conservation area for endangered sea cucumbers was recently set up at
(a) Andaman Islands
(b) Gulf of Mannar
(c) Sundarbans Forests
(d) Lakshadweep Islands
EXPLANATION:
In the face of recurring large-scale smuggling attempts targeting endangered sea cucumbers, the
Lakshadweep administration has created the world’s first conservation area for sea cucumbers.
Extending over a 239 square km radius of Cheriyapani reef, an uninhabited, independent, and
solitary atoll, it will be known as Dr. KK Mohammed Koya Sea Cucumber Conservation Reserve.
So, option (d) is correct.

ADDITIONAL INFORMATION:
Sea Cucumbers

s
About Sea cucumbers are part of a larger animal group called echinoderms,
which also contain starfish and sea urchins. Their body shape is similar
to a cucumber, but they have small tentacle-like tube feet that are used
for locomotion and feeding. Depending on the species, sea cucumbers
normally vary in size from less than an inch (2.5 centimeters) to over six
feet (1.8 meters).
Distribution Sea cucumbers are found in virtually all marine environments
throughout the world, from shallow to deep-sea environments. Sea
cucumbers are benthic, meaning they live on the ocean floor. However,
their larvae are planktonic, meaning they float in the ocean with the
currents.
Scavengers Sea cucumbers are scavengers that feed on small food items in the
benthic zone (seafloor), as well as plankton floating in the water column.
Algae, aquatic invertebrates, and waste particles make up their diet. They
eat with tube feet that surround their mouths.
In India There are around 1450 species of sea cucumbers found in the marine
system worldwide. Among them, 173 are found in the Indian seas. Out of
them, 16 species have been identified as economically very important.
Sea cucumber in India is treated as an endangered species listed under
schedule I of the Wildlife Protection Act of 1972.
Threat As endangered species are often vulnerable to poaching, sea cucumbers
have been the target of the seafood mafia operating in the Indian Ocean
region. The illegal transnational trade has been occurring in the
Lakshadweep region, despite the Wildlife Crime Control Bureau of India
launching enhanced special vigil involving coastguard, police, forest
officials, and fish workers in all the islands, especially those uninhabited
like Suhali.

Q.26) Consider the following statements about Green Hydrogen in the context of the Green Hydrogen
Energy Mission.
1. Green hydrogen is produced by splitting water into hydrogen and oxygen.
2. It has the highest energy content by weight and lowest energy content by volume.
3. It can be stored in cryo-compressed tanks in gaseous form apart from being kept in a liquefied and
solid state.
4. Currently, the production of green hydrogen is comparatively very cheap than other forms of
hydrogen such as Blue and Grey.
Which of the statements given above are correct?
(a) 1, 2 and 3 only
(b) 1, 2 and 4 only
(c) 1, 3 and 4 only
(d) 2, 3 and 4 only
EXPLANATION:
Green hydrogen is produced by splitting water into hydrogen and oxygen using an
electrolyzer powered by electricity from renewable energy sources such as wind and
solar. So, Statement 1 is correct.
Hydrogen is emerging as an important source of energy since it has zero carbon content
and is a non-polluting source of energy in contrast to hydrocarbons that have a net
carbon content in the range of 75–85 percent. Hydrogen energy is expected to reduce
carbon emissions that are set to jump by 1.5 billion tons in 2021. It has the highest
energy content by weight and lowest energy content by volume. So, Statement 2 is
correct.
Hydrogen can be stored physically as either a gas or a liquid. Storage of hydrogen as a
gas typically requires high-pressure tanks. Storage of hydrogen as a liquid requires
cryogenic temperatures because the boiling point of hydrogen at one-atmosphere
pressure is −252.8°C. Hydrogen can also be stored on the surfaces of solids (by
adsorption) or within solids (by absorption). So, Statement 3 is correct.
Currently, the production of green hydrogen is two or three times more expensive than
blue hydrogen, but with advancements in science and technology, it is expected to
become a cheap fuel in the future. So, Statement 4 is not correct.

ADDITIONAL INFORMATION:
National Hydrogen Energy Mission
Objective It aims to cut down carbon emissions and increase the use of
renewable sources of energy. The broad objective of the mission
is to scale up Green Hydrogen production and utilization and to
align India’s efforts with global best practices in technology,
policy, and regulation.
Fund Accordingly, the Government of India has allotted Rs 25 crore in
Allocation the Union Budget 2021–22 for the research and development in
hydrogen energy.
Hydrogen Hydrogen is emerging as an important source of energy since it
Energy has zero carbon content and is a non-polluting source of energy
in contrast to hydrocarbons that have a net carbon content in
the range of 75–85 percent.
Production The current global demand for hydrogen is 70 million metric tons
per year, more than 76 percent of which is being produced from
natural gas, 23 percent comes from coal and the remaining is
produced from electrolysis of water.
In India Hydrogen is at an early stage of entering the energy sector in
India. Government, as well as non-government funding agencies,
are engaged in R&D projects of hydrogen production, storage,
utilization, power generation, and transport applications.

Q.27) Arrange the following countries in the correct order from higher volume to lower volume in terms of
crude oil supply to India in the 2021 fiscal year.
1. Nigeria
2. Iraq
3. Saudi Arabia
4. United Arab Emirates
5. United States of America
Select the correct answer using the code given below.
(a) 2 – 3 – 4 – 5 – 1
(b) 3 – 2 – 4 – 5 – 1
(c) 4 – 2 – 3 – 5 – 1
(d) 1 – 2 – 3 – 4 – 5
EXPLANATION:
The country’s crude oil import during April-September 2021 stood at 100.9 million tonnes. The
top five suppliers were Iraq (24.7 million tonnes), Saudi Arabia (15.7 million tonnes), UAE (9.9
million tonnes), the US (eight million tonnes) and Nigeria (7.99 million tonnes). Iraq has been
topping the chart since 2017-18. So, Option (a) is correct.

ADDITIONAL INFORMATION:
India’s Oil Import
Trends in Import India's crude oil imports rose to a four-month high in August 2021,
recovering from the near one-year low in July 2021, with refiners buying
incremental barrels mainly from Latin America and the United States,
tanker data from trade sources showed.
The decline in  The two regions, the United States and Latin America, accounted for
West Asia Share about a fifth of India's August imports as refiners booked long-haul
crudes in end-May and June to gain from arbitrage. That led to a
decline in the share of West Asia and Africa oil, the data showed.
 The share of West Asian grades in India's August oil imports
declined to 61.8% from 64.7% in the previous month, while that of
African oil dropped to 14.1% from 16.7%.
 That dragged down the share of OPEC's oil in India's overall imports
to 67.7% from 77.6%. For April-August, the first five months of this
fiscal year, the group's share declined to the lowest.
Current Scenario  India's August imports were 23% higher than the previous month
in Market when refiners had shut units for maintenance. Indian refiners
curbed crude processing in August as they had a high inventory of
diesel and exports were not profitable.
 Domestic refiners have resumed exports of diesel as prices in
overseas markets improved due to higher demand from the West.
This would free up diesel storage and help in raising crude runs and
imports in the coming months.
Share of Top The country’s crude oil import during April-September 2021 stood at 100.9
countries million tonnes. The top five suppliers were Iraq (24.7 million tonnes), Saudi
Arabia (15.7 million tonnes), UAE (9.9 million tonnes), the US (eight million
tonnes) and Nigeria (7.99 million tonnes). Iraq has been topping the chart
since 2017-18.

Q.28) Consider the following statements about the Climate Vulnerability Index (CVI) of India.
1. Delhi-based Environmental think tank Council on Energy, Environment and Water (CEEW) has
carried such a first-of-its-kind district-level index in India.
2. As per this Index, Assam and Andhra Pradesh are the most vulnerable to extreme weather events,
and Kerala, Tripura and West Bengal are the least vulnerable.
Which of the statements given above is/are correct?
(a) 1 only
(b) 2 only
(c) Both 1 and 2
(d) Neither 1 Nor 2
EXPLANATION:
The Council on Energy, Environment, and Water (CEEW) is one of Asia’s leading not-for-profit
policy research institutions. The Council uses data, integrated analysis, and strategic outreach to
explain and change — the use, reuse, and misuse of resources. Environmental think tank Council
on Energy, Environment and Water has carried out a first-of-its-kind district-level climate
vulnerability assessment, or Climate Vulnerability Index (CVI), in which it has analyzed 640
districts in India to assess their vulnerability to extreme weather events such as cyclones, floods,
heatwaves, droughts, etc. So, Statement 1 is correct.

The CVI has ranked 20 states out of which Assam and Andhra Pradesh are the most vulnerable to
extreme weather events, and Kerala, Tripura, and West Bengal are the least vulnerable. The study
points out that the difference in the vulnerability of states ranked is marginal, making all states
vulnerable. So, Statement 2 is correct.

ADDITIONAL INFORMATION:
NEED FOR A  According to Germanwatch’s 2020 findings, India is the seventh-most
CLIMATE vulnerable country for climate extremes.
VULNERABILITY  Extreme weather events have been increasing in the country such as
supercyclone Amphan in the Bay of Bengal, which is now the
INDEX FOR
strongest cyclone to be recorded in the country.
INDIA
 Recent events such as the landslides and floods in
Uttarakhand and Kerala have also increased in the past decade.
 Another CEEW study has found that three out of four districts in
India are extreme event hotspots, with 40 percent of the districts
exhibiting a swapping trend, that is – traditionally flood-prone areas
are witnessing more frequent and intense droughts and vice-versa.
 Further, the IPCC states that every degree rise in temperature will
lead to a three percent increase in precipitation, causing the
increased intensification of cyclones and floods
KEY FINDINGS  While 27 Indian states and union territories are vulnerable to
extreme climate events, 463 districts out of 640 are vulnerable to
extreme weather events
 Dhemaji and Nagaon in Assam, Khammam in Telangana, Gajapati in
Odisha, Vizianagaram in Andhra Pradesh, Sangli in Maharashtra,
and Chennai in Tamil Nadu are among India’s most climate-
vulnerable districts
 More than 80 percent of Indians live in districts vulnerable to climate
risks – that is 17 of 20 people in India are vulnerable to climate risks,
out of which every five Indians live in extremely vulnerable areas
 More than 45 percent of these districts have undergone
“unsustainable landscape and infrastructure changes’’.
 183 hotspot districts are highly vulnerable to more than one extreme
climate events
 60% of Indian districts have medium to low adaptive capacity in
handling extreme weather events – these districts don’t have robust
plans in place to mitigate the impact
 North-eastern states are more vulnerable to floods
 South and central are most vulnerable to extreme droughts
 59 and 41 percent of the total districts in the eastern and western
states, respectively, are highly vulnerable to extreme cyclones.
MAJOR  Develop a high-resolution Climate Risk Atlas (CRA) to map critical
RECOMMENDATI vulnerabilities at the district level and better identify, assess, and
ONS project chronic and acute risks such as extreme climate events, heat
and water stress, crop loss, vector-borne diseases, and biodiversity
collapse. A CRA can also support coastal monitoring and forecasting,
which are indispensable given the rapid intensification of cyclones
and other extreme events.
 Establish a centralized climate-risk commission to coordinate the
environmental de-risking mission.
 Undertake climate-sensitivity-led landscape restoration focused on
rehabilitating, restoring, and reintegrating natural ecosystems as
part of the developmental process.
 Integrate climate risk profiling with infrastructure planning to
increase adaptive capacity.
 Provide for climate risk-interlinked adaptation financing by creating
innovative CVI-based financing instruments that integrate climate
risks for an effective risk transfer mechanism.

Q.29) Consider the following statements about the prevailing social status in India as per law.
1. Only a person, who professes a religion of Hinduism, shall be deemed to be a member of the
Scheduled Castes.
2. A person belonging to a Scheduled Castes (SC) or Scheduled Tribes (ST) will continue to be deemed
as such irrespective of his/her marriage to a non-SC/ST.
3. The rights of a person belonging to a Scheduled Tribe are independent of his/her religious faith.
4. No such religion-based bar operates for STs and Other Backward Classes (OBCs).
Which of the statements given above are correct?
(a) 1, 2 and 3 only
(b) 1, 2 and 4 only
(c) 1, 3 and 4 only
(d) 2, 3 and 4 only
EXPLANATION:
The Constitution (Scheduled Castes) Order, 1950 (as amended from time to time) states
that no person professing a religion different from Hindu or Sikh or Buddhist religion can
be deemed to be a member of a Scheduled Caste. If the person claims to be a Scheduled
Tribe, he may profess any religion. There is, however, no religious bar for being treated as
Scheduled Tribe or Other Backward Class.
A person who is a member of a Scheduled Caste or a Scheduled Tribe would continue to be
a member of that Scheduled Caste or Scheduled Tribe as the case may be, even after his or
her marriage with a person who does not belong to a Scheduled Caste or a Scheduled Tribe.
So, option (d) is correct.
ADDITIONAL INFORMATION:
IN CASE OF  Where a person migrates from the portion of the State in respect of which his
MIGRATION community is scheduled to another part of the same State in respect of
which his community is not scheduled, he will continue to be deemed to be a
member of the Scheduled Caste or the Scheduled Tribe, as the case may be,
concerning that State
 Where a person migrates from one State to another, he can claim to belong
to a Scheduled Caste or a Scheduled Tribe only concerning the State to
which he originally belonged and not in respect of the State to which he has
migrated.
IN CASE OF  Where a Scheduled Caste person gets converted to a religion other than
CONVERSION Hinduism or Sikhism and then reconverts himself back to Hinduism or
AND Sikhism, he will be deemed to have reverted to his original Scheduled Caste,
RECONVERSION if he is accepted by the members of that particular caste as one among them.
 In the case of a descendent of a Scheduled Caste convert, the mere fact of
conversion to Hinduism or Sikhism will not be sufficient to entitle him to be
regarded as a member of the Scheduled Caste to which his forefathers
belonged. It will have to be established that such a convert has been
accepted by members of the caste claimed as one among themselves and has
thus become a member of that caste.

Q.30) As per the Supreme Court verdict, popularly known as 2017 K.S. Puttaswamy judgment, any
invasion of privacy could only be justified in which of the following grounds in India?
1. The restriction must be made by the suitable law enacted by the competent authority
2. The restriction must promote a legitimate state interest such as National Security
3. It must be necessary only if other means are not available and proportionate only as much as
needed
4. The invasion could be justified under the pretexts of the global governance
Select the correct answer using the code given below.
(a) 1, 2 and 3 only
(b) 1, 2 and 4 only
(c) 1, 3 and 4 only
(d) 2, 3 and 4 only
EXPLANATION:
 Like other rights which form part of the fundamental freedoms protected by Part III, including
the right to life and personal liberty under Article 21, privacy is not an absolute right.
 A law that encroaches upon privacy will have to withstand the touchstone of permissible
restrictions on fundamental rights. In the context of Article 21, an invasion of privacy must be
justified based on a law that stipulates a procedure that is fair, just, and reasonable.
 The law must also be valid concerning the encroachment on life and personal liberty under
Article 21. An invasion of life or personal liberty must meet the threefold requirement of
o legality, which postulates the existence of law;
o need, defined in terms of a legitimate State aim; and
o proportionality which ensures a rational nexus between the objects and the means
adopted to achieve them.
So, option (a) is correct.
ADDITIONAL INFORMATION:
PUTTASWAMY CASE
ANALYSIS  A nine-judge bench ruled that the Right to Privacy is a fundamental right
for Indian citizens. Thus, no legislation passed by the government can
unduly violate it. Specifically, the court adopted the three-pronged test
required for the encroachment of any Article 21 right – legality-i.e. through
an existing law; necessity, in terms of a legitimate state objective and
proportionality, that ensures a rational nexus between the object of the
invasion and the means adopted to achieve that object.
 This clarification was crucial to prevent the dilution of the right in the
future on the whims and fancies of the government in power. The Court
adopted a liberal interpretation of the fundamental rights to meet the
challenges posed by an increasingly digital age. It held that individual
liberty must extend to digital spaces and individual autonomy and privacy
must be protected.
 This judgment settled that position of law and clarified that the Right to
Privacy could be infringed only when where there was a compelling state
interest for doing that. This position was the same as with the other
fundamental rights.
 This ruling by the Supreme Court paved the way for the decriminalization
of homosexuality in India on 6 September 2018, thus legalizing same-sex
sexual intercourse between two consenting adults in private. India is the
world's biggest democracy and with this ruling, it has joined the United
States, Canada, South Africa, the European Union, and the UK in
recognizing this fundamental right.

SUCCESSIVE  The judgment was interpreted as paving the way for the eventual
DEVELOPMENTS decriminalization of homosexuality in India in Navtej Singh Johar v. Union
of India (2018) and abolishing the provisions about the crime of Adultery
under the Indian Legal System in the case of Joseph Shine v. Union of
India (2018.)

Q.31) Consider the following statements about the Open Market Sale Scheme (OMSS).
1. It refers to the selling of food grains by Government agencies at predetermined prices in the open
market from time to time.
2. This scheme aims to enhance the supply of grains especially during the lean season and thereby to
moderate the general open market prices, especially in the deficit regions.
3. At present, the Food Corporation of India, on the instructions from the Government, sells all the
food crops in the open market from time to time.
Which of the statements given above are correct?
(a) 1 and 2 only
(b) 1 and 3 only
(c) 2 and 3 only
(d) 1, 2 and 3
EXPLANATION:
Food Corporation of India sells surplus stocks of only wheat and rice under Open Market
Sale Scheme (Domestic) at pre-determined prices under the guidelines of the government
through e-auction in the open market from time to time to enhance the supply of food
grains, especially wheat during the lean season and thereby moderate the open market
prices especially in the deficit regions. So, option (a) is correct.

ADDITIONAL INFORMATION:
DETAILS ABOUT  For transparency in operations, the Corporation has
OPEN MARKET switched over to e-auction for sale under Open Market Sale
SALE SCHEME Scheme (Domestic). The FCI conducts a weekly auction to
conduct this scheme in the open market using the platform
of commodity exchange NCDEX (National Commodity and
Derivatives Exchange Limited).
 The State Governments/ Union Territory Administrations are
also allowed to participate in the e-auction if they require
wheat and rice outside TPDS & OWS.
 The present form of OMSS comprises 3 schemes as under:
o Sale of wheat to bulk consumers/private traders
through e-auction.
o Sale of wheat to bulk consumers/private traders
through e-auction by dedicated movement.
o Sale of Raw Rice Grade ‘A’ to bulk consumers/private
traders through e-auction.

ABOUT FCI  FCI was set up in 1965 (under the Food Corporation Act,
1964) against the backdrop of a major shortage of grains,
especially wheat, in the country.
 Agricultural Prices Commission was also created in 1965 to
recommend remunerative prices to farmers, and FCI was
mandated with three basic objectives:
o to provide effective price support to farmers;
o to procure and supply grains to PDS for distributing
subsidized staples to economically vulnerable sections of
society; and
o keep a strategic reserve to stabilize markets for basic
foodgrains.

Q.32) Which one of the following statements about the Mullaperiyar dam is correct?
(a) It is located on the confluence of the Mullayar and Periyar rivers in Tamilnadu
(b) It is operated and maintained by the Kerala State Government
(c) It was built during the First five-year plan under the Harrod Domar model
(d) The Periyar Tiger Reserve and National Park are located around the dam's reservoir
EXPLANATION :

The dam is built at the confluence of Mullayar and Periyar rivers. The dam is located in Kerala on the
river Periyar, but is operated and maintained by the neighbouring state of Tamil Nadu.

The Dam was constructed and opened for use in the year 1895.

The Periyar National Park in Thekkady, a Protected area of Kerala, is located around the dam's reservoir,
Periyar lake.
So, Option (d) is correct

ADDITIONAL INFORMATION :
Mullaiperiyar Dam:
About  The dam is located in the upper reaches of the river Periyar, which flows into
Kerala after originating in Tamil Nadu.
 The reservoir is within the Periyar Tiger Reserve. The water diverted from the
reservoir is first used for power generation in lower Periyar (by Tamil Nadu) before
flowing into the Suruliyar, a tributary of Vaigai river, and then for irrigating nearly
2.08 lakh hectares in Theni and four other districts farther away.


 The 130-year old Mullaperiyar dam is located on the confluence of the Mullayar
and Periyar River in Kerala’s Idukki district.
 It was built by Pennycuick to create a harmonious atmosphere in the east coast by
taking the water there and have people cultivate crops.
 It is operated and maintained by the Tamil Nadu for meeting the drinking water
and irrigation requirements of 5 of its southern districts.
 The dam resulted in the formation of a big lake - the Periyar Lake. People call it the
Thekkady Wildlife Sanctuary.
Issues

 In 1979, a problem erupted over the safety of the dam which led to a tripartite
meeting that decided to bring the water level from the full reservoir level of 152 ft to
136 ft.
 By mid-1990, Tamil Nadu started demanding restoration of the water level in the
Mullaperiyar as it had completed the dam strengthening works.
 When no consensus was reached through negotiations, the Supreme Court was
approached.
 The Supreme Court in February, 2006 allowed Tamil Nadu to raise the water level
of the dam to 152ft.
 In response to that,Kerala government enacted Irrigation and Water Conservation
(Amendment) Act, 2006 and put the second schedule of Mullaperiyar Dam as
endangered and fixed its maximum water level to 136ft.
 In 2014, the apex court declared the Act unconstitutional and restrained Kerala
from interfering with the rights of Tamil Nadu in raising the water level in the dam
to 142 feet.
 The Kerala government has been at many times emphasising the urgent need for
the gradual release of water.
Kerala’s  Dam safety- The Mullaperiyar dam suffers from structural issues and the
Stance possibility of a dam-break cannot be ruled out.
 The dam is located in an earthquake-prone area and small-time earthquakes that
had happened in 1979 and 2011 caused some cracks in the dam.
 The leakage in the dam is another cause of concern.
 The technology which was put into use for constructing the dam 130 years ago was
obsolete compared to the sophisticated construction methods used now.
 According to the UN University report, 35 lakh people in Kerala will be directly hit
in the case of a dam-break.
 It will also have its impact on the National Periyar Park which hosts some of the
endangered species.
Tamilnadu’s  For Tamil Nadu the Mullaperiyar dam is like a lifeline for the people of Madras
Stance Presidency for irrigation and drinking
 The dam is also significant for the generation of the power in lower Periyar water
station as the region is is shadow and arid.
 So, the government of Tamil Nadu insists on raising the water level in the dam
pointing out the failure of crops.
 The government has also asserted that it has full right over the control of the dam.
 Tamil Nadu has challenged the Kerala’s proposal for decommissioning of the dam
and construction of a new dam.

Q.33) Consider the following statements:


1. The breeding areas for this bird are alpine meadows, lakeside and riverine marshes and river
valleys.
2. These birds, considered loyal couples, are only found in Ladakh’sChangthang region.
3. It is revered in Buddhist traditions and culturally protected across much of its range.
Which of the following bird is clearly indicated by the statements given above?
(a) Great hornbill
(b) Himalayan monal
(c) Hill myna
(d) Black-necked crane
EXPLANATION:
The black-necked crane, only found in the Ladakh region, was the State bird of J&K
before August 5, 2019. Recently, the snow leopard (Panther unica) and black-necked
crane (Grus nicricollis) became the State animal and State bird respectively of the UT of
Ladakh. Black-necked cranes, considered loyal couples, are only found in Ladakh’s
Changthang region. While the high altitude wetlands of the Tibetan plateau (Tibet,
Qinghai, Xinjiang, Gansu), Sichuan (China), and eastern Ladakh (India) are the main
breeding ground of the species, the birds spend winter at lower altitudes. The breeding
areas are alpine meadows, lakeside, riverine marshes, and river valleys. They also make
use of barley and wheat fields in these areas. Often associated with peace, the Black-
necked crane is central to Buddhist mythology and culture. According to a World Wide
Fund for Nature (WWF) blogpost, legend says “previous incarnations of the Dalai Lama
were carried from monastery to monastery on the backs of these holy birds.” So, Option
(d) is correct.

ADDITIONAL INFORMATION:
Black-necked Crane
About The upper long neck, head, primary and secondary flight
feathers, and tail are completely black and body plumage is
pale gray/whitish. A conspicuous red crown adorns the head.
The bill is greenish and the legs and feet are black.
Distribution The black-necked crane is a resident of Inner Asia—the
regions of Xinjiang, Tibet (and the Chinese provinces carved
out of it, namely Qinghai and Gansu) as well the Chinese
Proper provinces of Sichuan, Yunnan, and Guizhou. It also
occurs in Jammu and Kashmir (of which it is the state bird)
and Arunachal Pradesh as well as a few sites in Bhutan.
Protection It is listed in Schedule I of Wildlife (Protection) Act 1972. In
2020, a re-assessment of the crane’s population led its status
to change from Vulnerable to Near Threatened on the IUCN
Red List.
Threats The major threat to the successful breeding of black-necked
cranes is the damage to the eggs and chicks, caused by feral
dogs. These dogs are owned both by armed forces as well as
by the local nomads. Another threat to the bird is the loss of
habitat. The human pressure on the wetlands, the primary
habitat of cranes, has increased tremendously over the last
decade. The increased grazing pressure on the limited
pastures near the wetlands is also leading to the degradation
of the wetland habitat.

Q.34) Which of the following protected areas have been recommended for holding and conservation for
breeding of cheetah in India under the Action Plan for Introduction of Cheetah in India?
1. KunoPalpur National Park in Madhya Pradesh.
2. Nauradehi Wildlife Sanctuary in Madhya Pradesh
3. Gandhi Sagar Wildlife Sanctuary in Madhya Pradesh
4. Shahgarh bulge in Jaisalmer, Rajasthan
5. Mukundara Tiger Reserve in Rajasthan
Select the correct answer using the code given below.
(a) 1, 2 and 3 only
(b) 1, 4 and 5 only
(c) 1, 2, 4 and 5 only
(d) 1, 2, 3, 4 and 5
EXPLANATION:
 Amongst the 10 surveyed sites of the central Indian states, Kuno Palpur National Park (KNP) in
Madhya Pradesh has been rated the highest. The other sites recommended for holding and
conservation breeding of cheetah in India, in controlled wild conditions are:
 Nauradehi Wildlife Sanctuary (1,197 sq. km, habitat 5,500 sq. km), Madhya Pradesh
 Gandhi Sagar Wildlife Sanctuary – Bhainsrorgarh Wildlife Sanctuary complex (~2500 sq. km),
Madhya Pradesh
 Shahgarh bulge in Jaisalmer, Rajasthan (4,220 sq. km)
 Mukundara Tiger Reserve as fenced enclosure (~80 sq. km), Rajasthan
So, option (d) is correct.
ADDITIONAL INFORMATION:
CHEETAH
KEY  Cheetah happens to be the only large carnivore that got completely
FACTS wiped out from India, mainly due to over-hunting and habitat loss.
The Action Plan highlights the nation’s preparedness in bringing the
cheetah back. The conservation of Cheetahs has a very special
significance for the national conservation ethic and ethos.
 The very name 'Cheetah' (Acinonyx Jubatus Venaticus) originates from
Sanskrit and means 'the spotted one'.
 The locally extinct cheetah-subspecies of India are found in Iran and
are categorized as critically endangered.
 An important consideration during such conservation efforts is that
the sourcing of animals should not be detrimental to the survival of
the source population. Since it is not possible to source the critically
endangered Asiatic cheetah from Iran without affecting these sub-
species, India will source cheetahs from Southern Africa, which can
provide India with substantial numbers of a suitable cheetahs for
several years.
 Cheetahs from Southern Africa have the maximum observed genetic
diversity among extant cheetah lineages, an important attribute for a
founding population stock. Moreover, the Southern African cheetahs
are found to be ancestral to all the other cheetah lineages including
those found in Iran.
HABITAT  Besides conserving the big cat, the initiative in itself is a boon to the
ecosystem. Cheetahs live in open plains; their habitat is
predominantly where their preys live - grasslands, scrubs and open
forest systems, semi-arid environments, and temperatures that tend
to be hotter compared to cooler regimes.
 In saving cheetahs, one would have to save not only its prey-base
comprising certain threatened species, but also other endangered
species of the grasslands and open forest ecosystems, some of which
are on the brink of extinction.
 It is also observed that among large carnivores, conflict with human
interests is lowest for Cheetahs. They are not a threat to humans and
do not attack large livestock either.

Q.35) Consider the following statements about the National Tiger Conservation Authority (NTCA).
1. It has been constituted under the Wildlife (Protection) Act, 1972.
2. The Prime Minister of India is the ex officio chairman of this authority.
3. Project Tiger, a tiger conservation programme launched in 1973, is administered by this authority.
Which of the statements given above is/are correct?
(a) 1 only
(b) 1 and 2 only
(c) 2 and 3 only
(d) 1 and 3 only
EXPLANATION:
The National Tiger Conservation Authority (NTCA) has been constituted under section 38 L
(1) of the Wildlife (Protection) Act, 1972. Further, as per section 38 L, subsection 2 of the
said Act, the authority consists of the Minister in charge of the Ministry of Environment and
Forests ( as Chairperson), the Minister of State in the Ministry of Environment and Forests
(as Vice-Chairperson), three members of Parliament, Secretary, Ministry of Environment
and Forests and other members. So, Statement 1 is correct and Statement 2 is not
correct.
Project Tiger was administered by the National Tiger Conservation Authority. The overall
administration of the project is monitored by a steering committee, which is headed by a
director. A field director is appointed for each reserve, who is assisted by a group of field
and technical personnel. So, Statement 3 is correct.

ADDITIONAL INFORMATION:
PROJECT TIGER (1973)
ABOUT  Project Tiger is an ongoing Centrally Sponsored Scheme of the
PROJECT Ministry of Environment, Forests, and Climate Change providing
TIGER central assistance to the tiger States for tiger conservation in
designated tiger reserves.
OBJECTIVES Project Tiger's main aims are to:
 Reduce factors that lead to the depletion, of tiger habitats and
mitigate them by suitable management. The damages done to the
habitat shall be rectified to facilitate the recovery of the ecosystem
to the maximum possible extent.
 Ensure a viable tiger population for economic, scientific, cultural,
aesthetic, and ecological values.
 The monitoring system M-STrIPES was developed to assist patrol
and protect tiger habitats. It maps patrol routes and allows forest
guards to enter sightings, events and changes when patrolling. It
generates protocols based on these data, so that management
decision can be adapted

FRAMEWORK The various tiger reserves were created in the country based on the 'core-
buffer' strategy:
 Core area: the core areas are free of all human activities. It has the
legal status of a national park or wildlife sanctuary. It is kept free of
biotic disturbances and forestry operations like a collection of
minor forest produce, grazing, and other human disturbances are
not allowed within.
 Buffer areas: the buffer areas are subjected to 'conservation-
oriented land use'. They comprise forest and non-forest land. It is a
multi-purpose use area with twin objectives of providing habitat
supplement to spillover population of wild animals from core
conservation unit and to provide site-specific co-developmental
inputs to surrounding villages for relieving their impact on the core
area.
Q.36) Consider the following statements about the Critical elephant habitats:
1. Critical elephant habitats are notified as ‘elephant reserve’ for focus and synergy in elephant
conservation and to reduce conflict.
2. A total of 30 elephant reserves have been established in major elephant States in India.
3. Elephant reserves are pervasive in India including the North-Western part of the country.
Which of the statements given above is/are correct?
(a) 1 only
(b) 1 and 2 only
(c) 2 and 3 only
(d) 1, 2 and 3
EXPLANATION:
Critical elephant habitats had been notified as elephant reserves for focus and synergy in
elephant conservation and to reduce conflict. The notification was usually carried out with
the approval of the Steering Committee formed in the Environment ministry. So,
Statement 1 is correct.
Some 30 elephant reserves had been established so far in 14 major elephant states
to reduce human-elephant conflict and to avoid retaliatory killing of elephants.
Compensation was provided to local communities for the loss of their property and life
caused by wild elephants. The report submitted by a Karnataka Elephant Task Force in
2012 said human-elephant conflict mainly happened due to fragmentation of habitats,
habitat loss, change in cropping pattern, etc. So, Statement 2 is correct.
Old literature indicates that even during the Moghul period, elephants were found all over
India including many parts of Central India like Marwar, Chanderi, Satwas, Bijagarh, and
Panna. However the current distribution of wild elephants in India is confined to South
India; North East including North West Bengal; Central Indian states of Odisha, South WB,
and Jharkhand; and North India in Uttarakhand and UP. So, Statement 3 is not correct.

ADDITIONAL INFORMATION:
INDIAN ELEPHANTS
FACTS  Indian elephants may spend up to 19 hours a day feeding and they can
produce about 220 pounds of dung per day while wandering over an area that
can cover up to 125 square miles. This helps to disperse germinating seeds.
 They feed mainly on grasses, but large amounts of tree bark, roots, leaves,
and small stems are also eaten. Cultivated crops such as bananas, rice, and
sugarcane are favored foods as well. Since they need to drink at least once a
day, these elephants are always close to a source of freshwater.
 Elephant ( Elephas maximus) is the largest terrestrial mammal of India.
Elephants being wide-ranging animal requires large areas. The requirement
of food and water for elephants is very high and therefore their population
can be supported only by forests that are under optimal conditions.
 The status of elephants can be the best indicator of the status of the forests.
 More than 60 % of the population is held in India.
IUCN STATUS  Endangered
HABITATS  Subtropical broadleaf forest, tropical broadleaf moist forest, dry forest,
grassland
PROJECT  Project Elephant was launched in 1992 by the Government of India Ministry

ELEPHANT IN of Environment and Forests to provide financial and technical support

INDIA to wildlife management efforts by states for their free-ranging populations of

wild Asian Elephants.

 The project aims to ensure the long-term survival of the population of

elephants in their natural habitats by protecting them, their habitats,

and migration corridors.

 Other goals of Project Elephant are supporting the research of the ecology

and management of elephants, creating awareness of conservation among

local people, providing improved veterinary care for captive elephants

Q.37) Consider the following statements:

1. Madhya Pradesh has become the 'Leopard State' of India.

2. Gujarat is the only state with 100% of the Asiatic lion population in the world.

3. While Gujarat has a population of leopards, it has no single tiger population.

Which of the statements given above is/are correct?

(a) 1 only

(b) 1 and 2 only

(c) 2 and 3 only

(d) 1, 2 and 3

EXPLANATION:

Madhya Pradesh has become the 'Leopard State' of India. The population of the IUCN red-listed

vulnerable species - leopard has reached 3,421 in MP is the highest in the country. So, Statement 1

is correct.

After the centre planned for the relocation of the Asiatic lion to Madhya Pradesh, Kuno-Palpur Wildlife

Sanctuary, Gujarat state officials resisted the relocation, since it would make the Gir Wildlife

Sanctuary lose its status as the world's only home of the Asiatic lion. So, Statement 2 is correct.

Gujarat, which is the only home to Asiatic lions in India, had not seen the tiger for over three decades.

It is the only state in western India which does not have tigers until recently the Gujarat forest offcials

found the presence of tiger through camera traps. So, Statement 3 is not correct.
ADDITIONAL INFORMATION:
SPECIES CHARACTERISTICS AND RELATED INFORMATION
 IUCN Red List: Endangered.
Asiatic Lion  CITES: Appendix-I
 Indian Wildlife (Protection) Act, 1972: Schedule-I
 Restricted only to Gir National Park and the surrounding areas in the
Indian state of Gujarat.
 Habitats in Tropical and subtropical dry broadleaf forests, thorny forests
and savanna.
 In the surrounding Gir National Park, only Maldhari tribes have the right
to take their livestock for grazing.
 The Gir lion is similar in size to the Central African lion and smaller than
large African lions.
 Male Asiatic lions are solitary, or associate with up to three males, forming
a loose pride.
 Females associate with up to 12 females, forming a stronger pride together
with their cubs.
 The gestation period lasts between 100 – 119 days.
 IUCN Red List: Endangered.
 CITES: Appendix-I
Royal Bengal Tiger  Indian Wildlife (Protection) Act, 1972: Schedule-I
 Habitats in Tropical (Moist and Dry Deciduous forest), Subtropical and
Temperate forests. Native to the Indian subcontinent.
 According to the Tiger Census 2018, Madhya Pradesh holds the highest
population of tigers, overtaking Karnataka position.
 Nagarjunsagar Srisailam Tiger Reserve is the largest tiger reserve in India,
having the largest area under the ''Critical Tiger Habitat''.
 Adult animals congregate only temporarily when special conditions permit,
such as a plentiful supply of food. Otherwise, they lead solitary lives,
hunting individually.
 The gestation period is about 93 – 112 days.
 IUCN Red List: Vulnerable
Leopard  CITES: Appendix-I
 Indian Wildlife (Protection) Act, 1972: Schedule-I
 It is the smallest of the Big Cats (Of genus Panthera namely the Tiger,
Lion, Jaguar, Leopard, and Snow Leopard)
 Habitats in a wide range in sub-Saharan Africa, in small parts of Western
and Central Asia, on the Indian subcontinent to Southeast and East Asia.
 Melanism is commonly occurring in leopards and is often called black
panther or jaguar.
 As per a recent report ‘Status of leopards in India, 2018’ released by the
MoEF&CC- 60% increase in the population count of leopards in India from
2014 estimates’’.
 The largest number of leopards have been estimated in Madhya Pradesh
(3,421) followed by Karnataka (1,783) and Maharashtra (1,690).
 Highly vulnerable to Roadkill in North India.
Q.38) Consider the following statements about the Wildlife Crime Control Bureau (WCCB):
1. It is a statutory multi-disciplinary body established under the Wild Life (Protection) Act, 1972.
2. It is mandated to collect and collate intelligence related to organized wildlife crime activities.
3. It also assists and advises the Customs authorities in inspection of the consignments of flora and
fauna as per the provisions ofEXIM Policy.
Which of the statements given above is/are correct?
(a) 1 only
(b) 1 and 2 only
(c) 2 and 3 only
(d) 1, 2 and 3
EXPLANATION:
Wildlife Crime Control Bureau (WCCB) is a statutory multi-disciplinary body, established by the
Government of India under the Wild Life (Protection) Act, 1972 and functions under the Ministry of
Environment and Forests and Climate Change (MoEF&CC) to combat organized wildlife crime in the
country. So, Statement 1 is correct.
Under Section 38 (Z) of the Wild Life (Protection) Act, 1972, WCCB is mandated to collect and
collate intelligence related to organized wildlife crime, Establish a centralized wildlife crime data
bank, Assist foreign authorities and international organizations for wildlife crime control, Building
scientific and professional investigation, Assist State Governments and Advise the Government of
India on issues relating to wildlife crimes having national and international ramifications, relevant
policy and laws. So, Statement 2 is correct.
WCCB also assists and advises the customs authorities in inspection of the consignments of flora &
fauna as per the provisions of the Wild Life Protection Act, Convention on International Trade in
Endangered Species of Wild Fauna and Flora (CITES) and Export-Import (EXIM) Policy governing
such an item. So, Statement 3 is correct.
ADDITIONAL INFORMATION:
WCCB OBJECTIVE
OPERATION
Operation  The WCCB had launched operation “Save Kurma” from December 2016 to
Save Kurma January 2017 to focus on the poaching, transportation and illegal trade of
live turtles and tortoises.
Operation  Operation Turtshield- I and Operation Turtshield-II took up to tackle the
Turtshield illegal trade of live Turtles and Turtle Calipee.
Operation  Operation Lesknow, Lesknow-II and Operation Lesknow-III were conducted
Lesknow to gain the attention of enforcement agencies towards the illegal wildlife trade
in lesser-known species of wildlife.
Operation  Operation Clean Art was conducted to drag the attention of enforcement
Clean Art agencies towards illegal wildlife trade in Mongoose hair brushes.
Operation  Operation Softgold was conducted by WCCB to tackle Shahtoosh Shawl
Softgold (made from Chiru wool) illegal trade and to spread awareness among the
weavers and traders engaged in this trade.
Operation  WCCB conducted Operation Birbil to curb illegal trade in wild cat and wild
Birbil bird species.
Operation  Aimed to draw the attention of the enforcement agencies within the country
Wildnet to focus their attention on the ever-increasing illegal wildlife trade over the
internet using social media platforms.
Operation  Operation Freefly on the illegal trade of live birds.
Freefly
Operation  Operation Wetmark to ensure the prohibition of the sale of meat of wild
Wetmark animals in wet markets across the country.

Q.39) When the air quality for the particular region of India is indicated in yellow colour as per the
National Air Quality Index, then what does it mean?
(a) It is a moderately polluted category
(b) It is a very severe category
(c) It is in Satisfactory condition
(d) It is a poor category
EXPLANATION:

Values
Daily of
AQI Levels of Inde
Color Concern x Description of Air Quality

Green Good 0 to 50 Air quality is satisfactory, and


air pollution poses little or
no risk.

Yellow Moderate 51 to Air quality is acceptable.


100 However, there may be a
risk for some people,
particularly those who are
unusually sensitive to air
pollution.

Orange Unhealthy for 101 to Members of sensitive groups


Sensitive 150 may experience health
Groups effects. The general public is
less likely to be affected.

Red Unhealthy 151 to Some members of the general


200 public may experience
health effects; members of
sensitive groups may
experience more serious
health effects.

Purple Very 201 to Health alert: The risk of health


Unhealthy 300 effects is increased for
everyone.
Maroon Hazardous 301 Health warning of emergency
and conditions: everyone is more
high likely to be affected.
er

So, Option (a) is correct.

ADDITIONAL INFORMATION:
National Air Quality Index
Objective Air Quality Index is a tool for effective communication of air
quality status to people in terms, which are easy to
understand. It transforms complex air quality data of
various pollutants into a single number (index value),
nomenclature, and color.
Categories There are six AQI categories, namely Good, Satisfactory,
Moderately polluted, Poor, Very Poor, and Severe. Each of
these categories is decided based on ambient
concentration values of air pollutants and their likely
health impacts (known as health breakpoints).
Pollutants AQ sub-index and health breakpoints are evolved for eight
pollutants (PM10, PM2.5, NO2, SO2, CO, O3, NH3, and
Pb) for which short-term (up to 24-hours) National
Ambient Air Quality Standards are prescribed.

Q.40) Arrange the following states of India in the order of High to low in terms of tiger population based
on recent data.
1. Karnataka
2. Madhya Pradesh
3. Maharashtra
4. Uttarakhand
Select the correct answer using the code given below.
(a) 1 – 2 – 3 – 4
(b) 2 – 1 – 4 – 3
(c) 3 – 2 – 1 – 4
(d) 4 – 1 – 2 – 4
EXPLANATION:
According to the Tiger Census 2018, Madhya Pradesh became the Tiger state of India with
526 Tigers whereas Karnataka slipped to the second position with 524 Royal Bengal Tiger
population and Uttrakhand is third with 442 Tigers Population. So, Option (b) is correct.
ADDITIONAL INFORMATION:
SUBJECT INFORMATION
Tiger Census 2018  The fourth cycle of the Tiger Census 2018, conducted in
2018-19 is the most comprehensive in terms of both resource
and data recorded.
 It counted 2976 tigers which are 75% of the global tiger
population.
 The census is done quadrennially (every four years) by the
National Tiger Conservation Authority (NTCA) with technical
help from the Wildlife Institute of India and with cooperation
from the state Forest Departments and partners.
Significance  India has fulfilled its resolution to double the Tiger numbers
made at the St. Petersburg Tiger Summit in 2010, before the
target year of 2022.
 The tiger numbers in India have increased from around 1500
in 2010 to 2976 in 2020 (doubled) mainly through efforts of
ongoing Project Tiger.
Project Tiger  It was launched in 1973 with 9 tiger reserves for conserving
our national animal, the tiger.
 It is an ongoing Centrally Sponsored Scheme of the Ministry
of Environment, Forests and Climate Change.
 Currently, the Project Tiger coverage has increased to 51
Tiger reserves, spread out in 18 tiger range states.
 The tiger reserves are constituted on a core/buffer strategy.
 The core areas have the legal status of a national park or a
sanctuary, whereas the buffer or peripheral areas are a mix
of forest and non-forest land, managed as a multiple-use
area.
National Tiger  The NTCA was launched in 2005, following the
Conservation recommendations of the Tiger Task Force.
Authority  It is a statutory body of the MoEF&CC and functions as
provided in the Wildlife (Protection) Act, 1972.
 Chairperson- The Minister for MoEF&CC.
 Vice Chairperson- Minister of State for MoEF&CC.
 The authority derives its power from WLPA, 1972 and
functions under the guidance of Chairperson, Vice-
Chairperson and members.
Tiger Conservation  A collective responsibility between the Centre and States,
Foundation warranting concerted, innovative and time-bound action.
 As per the Wildlife (Protection) Act, 1972, the State
Government shall establish a Tiger Conservation Foundation
(TCF) for tiger reserves within the State to facilitate and
support the tiger reserve management.
 The Central Government has framed ‘National Tiger
Conservation Authority (Tiger Conservation Foundation)
Guidelines, 2007 for the purpose of regulation of Tiger
Conservation Foundations (TCF).
 Out of the 51 tiger reserves, the TCFs have been constituted
for most of the tiger reserves except Kamlang, Orang, Pilibhit,
Rajaji and Srivilliputhur Megamalai Tiger Reserve.
Q.41) Which of the following are the best places to spot the Fishing cats naturally in India?
1. Mangroves of Sundarbans
2. Bharatpur sanctuary of Rajasthan
3. High ranges of Western Ghats
4. Wetlands of Gujarat
Select the correct answer using the code given below.
(a) 1, 2 and 3 only
(b) 1, 2 and 4 only
(c) 1, 3 and 4 only
(d) 2, 3 and 4 only
EXPLANATION:
 The fishing cat lives foremost in the vicinity of wetlands, along rivers, streams, oxbow
lakes, swamps, and mangroves.
 In India, fishing cats are mainly found in the mangrove forests of the Sundarbans, on the
foothills of the Himalayas along the Ganga and Brahmaputra river valleys, and in the
Western Ghats,
 And wetland areas in Rajasthan, including the Bharatpur bird sanctuary, around the
Yamuna floodplains in Agra, in the different parts of the vast Terai landscape, protected
areas like the Dudhwa Tiger Reserve and Corbett National Park and the extensive lower
Gangetic floodplains and the Sundarbans in West Bengal.
 Head further north and east from here, the species has been reported in Nagaland,
Arunachal Pradesh, and Assam. Travel south from West Bengal and records emerge from
around the Chilika lake and Bhitarkanika National Park in Orissa, a single stray record of a
rescued individual in Jharkhand and then substantial populations in the Godavari and
most recently in Krishna river delta in Andhra Pradesh.
So, option (a) is correct.

ADDITIONAL INFORMATION:
FISHING CATS
IUCN STATUS  ENDANGERED
FACTS  The fishing cat is an adept swimmer and enters water frequently to prey
on fish as its name suggests. It is known to even dive to catch fish.
 The fishing cat is nocturnal and apart from fish also preys on frogs,
crustaceans, snakes, birds, and scavenges on carcasses of larger
animals.
 It is capable of breeding all year round but in India, its peak breeding
season is known to be between March and May.
 Litter size varies between 1-4 and the young are suckled to the age of
six months and become independent once they are ten months old. In
the wild, fishing cats live up to an age of 12 years on average.

THREATS  One of the major threats facing the fishing cat is the destruction of
wetlands, which is its preferred habitat. As a result of human
settlement, drainage for agriculture, pollution, and wood-cutting most of
the wetlands in India are under threat of destruction.
 Another threat to the fishing cat is the depletion of its main prey-fish
due to unsustainable fishing practices. It is also occasionally poached
for its skin.
CONSERVATION  The Fishing Cat Conservation Alliance (FCCA) is a team of
MEASURES conservationists, researchers, and fishing cat enthusiasts working to
achieve a single dream – a world with functioning floodplains and
coastal ecosystems that would ensure the survival of the fishing cat
and all species with which it shares a home.
 The Convention on International Trade in Endangered Species (CITES)
lists the fishing cat in Appendix II part of Article IV of CITES, which
governs international trade in this species.
 In India, the fishing cat is included in Schedule I of the Indian Wildlife
(Protection) Act, 1972, and thereby protected from hunting.

Q.42) The Global Hunger Index (GHI)is a tool that measures and tracks hunger globally as well as by
country. Which one of the following parameters is not appropriate for the calculation of this Index?
(a) Percentage of undernourished in the population
(b) Percentage of children who die before the age of five
(c) Percentage of children under five years who suffer from wasting and stunting
(d) Percentage of children under five years who suffer from obesity and hyperalimentation
EXPLANATION :

GHI scores are calculated using a three-step process that draws on available data from various sources
to capture the multidimensional nature of hunger.

First, for each country, values are determined for four indicators:

UNDERNOURISHMENT: the share of the population that is undernourished (that is, whose caloric
intake is insufficient);

CHILD WASTING: the share of children under the age of five who are wasted (that is, who have low
weight for their height, reflecting acute undernutrition);

CHILD STUNTING: the share of children under the age of five who are stunted (that is, who have low
height for their age, reflecting chronic undernutrition); and

CHILD MORTALITY: the mortality rate of children under the age of five (in part, a reflection of the fatal
mix of inadequate nutrition and unhealthy environments).

So, Option (d) is correct.

ADDITIONAL INFORMATION :
Global Hunger Index:
About  The GHI has been brought out every year by Welthungerhilfe (lately in
partnerships with Concern Worldwide) since 2000.
 A low score gets a country a higher ranking which implies a better performance.
 The reason for mapping hunger is to ensure that the world achieves “Zero Hunger
by 2030” - a Sustainable Development Goals of the UN.
 The Global Hunger Index ranked India at 101 position out of a total 116
countries. India is also among the 31 countries where hunger has been identified
as serious.
Dimensions
and
Indicators

Q.43) Which one of the following statements about the Green Day Ahead Market (GDAM), recently seen in
the news, is correct?
(a) It is the new marketplace for vegetable trade without any intermediary
(b) It is a marketplace for the trading of renewable power on a day-ahead basis
(c) It is the new mechanism for trading green bonds in the financial market
(d) It is the concept of mobilizing funds by Small Sector companies from the common people through
RBI.
EXPLANATION :

Green Day Ahead market (GDAM) is a marketplace for trading of renewable power on a day ahead
basis.
So, Option (b) is correct.

ADDITIONAL INFORMATION :
Green Day Ahead Market
DAM  Day-Ahead-Market (DAM) is a physical electricity trading market for deliveries for
any/some/all 15 minute time blocks in 24 hours of next day starting from
midnight.
GDAM  Green Day Ahead market (GDAM) is a marketplace for trading of renewable power
on a dayahead basis.
 As the buyer’s behavior is shifting from long term contracts to short term
contracts and also from fossil fuel to non-fossil fuel, this new initiative will help in
enabling this energy transition.
 The intent is to promote merchant green power plants and provide additional sale
avenues to existing renewable power plants that are
o Facing payment risk with the distribution companies (DISCOM) under the
existing Power Purchase Agreements (PPAs) or
o 2. Have surplus energy
Benefits  The launch of the Green Day-Ahead Market will,
o Deepen the green market and
o Provide competitive price signals,
o Offer an opportunity to the market participants to trade in green energy, in
a transparent, flexible, competitive & efficient manner.
o Buy & Sell - The distribution utilities would also be able to sell surplus
renewable power generated in their area.
o The obligated entities (distribution licensee, open access consumers and
captive power consumers) would also be able to meet the RPO target by
directly buying green power from the power exchange(s).
o The non-obligated entities will be able to buy power on voluntary basis and
help increasing the share of green power.
o Domino Effect - Introduction of GDAM is expected to create a domino effect
that will lead to a gradual shift from PPA based contract to market-based
models.
o These models will build and deepen the markets to the next level, paving
the way for India to meet its target of 450 GW green capacity by 2030.
o The other benefits of participating in the GDAM would be reduction of
curtailment of green power, unlocking untapped renewable energy
potential, ensuring instant payment to RE generators ie on the day of
delivery itself.
Nodal  National Load Despatch Centre (NLDC), POSOCO, as the Nodal Agency, has set up
Agency the latest technologies and communication infrastructure for facilitating the
Green Day Ahead Market.
Working  The Exchanges will offer the market participants to submit bids together for both
conventional & renewable energy through the separate bidding windows.
 The clearance will take place in a sequential manner - renewable energy bids will
be cleared first in accordance with the must run status of the renewables,
followed by conventional segment.
 This mechanism will allow renewable energy sellers to subsequently bid in the
conventional segment should their bids remain uncleared in the green market.
There will be separate price discoveries for the both the conventional and
renewables.

Q.44) Consider the following statements about the socialistic pattern of the economy:
1. In contrast to capitalism, socialists believe shared ownership of resources and central planning
offers a more equitable distribution of goods and services
2. A market-led socialistic pattern of the economy will result in a few becoming rich and a majority
suffering from sharp income inequality and rising poverty.
3. Market socialism, also called liberal socialism, is an economic system that represents a
compromise between socialist planning and free enterprise, in which enterprises are publicly
owned but production and consumption are guided by market forces rather than by government
planning.
4. While social democracy aims to achieve greater economic stability and equality through policy
measures such as taxes, subsidies and social welfare programs, market socialism aims to achieve
similar goals through changing patterns of enterprise ownership and management
Which of the statements given above are correct?
(a) 1, 2 and 3 only
(b) 2 and 4 only
(c) 1, 3 and 4 only
(d) 1, 2, 3 and 4
EXPLANATION :

Socialists contend that shared ownership of resources and central planning provide a more equal
distribution of goods and services and a more equitable society.
So, Statement 1 is correct.

A market-led socialistic pattern of economy will result in a few becoming rich and a majority
suffering from sharp income inequality and rising poverty.
So, Statement 2 is correct.
Market socialism, also called liberal socialism, economic system representing a compromise
between socialist planning and free enterprise, in which enterprises are publicly owned but
production and consumption are guided by market forces rather than by government planning.
A form of market socialism was adopted in Yugoslavia in the 1960s in distinction to the centrally
planned socialism of the Soviet Union.
A similar development occurred in Hungary during the late 1960s and early 1970s.

So, Statement 3 is correct


While social democracy aims to achieve greater economic stability and equality through policy
measures such as taxes, subsidies and social welfare programs, market socialism aims to achieve
similar goals through changing patterns of enterprise ownership and management

So, Statement 4 is correct

ADDITIONAL INFORMATION :
‘Common Prosperity’ Policy in China

Issue  With widening income inequality, rising debt levels and social disorder
caused by market-driven policies, China increasingly feels the need for a
policy shift.

About the  China has been adopting principles of market capitalism with tenets of
Policy Marxist-Leninist ideology.
 With these dual ideological principles and mechanism, China has emerged as
an economic powerhouse.
 But all such policies followed after Mao, especially during Deng Xiaoping’s
era, are not proving to be socially compatible for Chinese society.
 A market-led socialistic pattern of economy is resulting in a few becoming
extravagantly rich and a majority suffering from sharp income inequality and
rising poverty.
 The current leadership is highly dissatisfied with the policies that have been
driving the Chinese economy, society and politics for the last four decades.
 So, a societal correction has gained priority under Xi’s leadership.
 There is a conscious attempt to curb corruption, and change the policies to
restrict the rich from becoming richer at the cost of society.
 Efforts are on to revisit the state policies, which would focus exclusively on
‘common prosperity.’
 An initial sign is the government mounting unprecedented crackdowns on
Alibaba and other top corporate firms from various sectors.
 Mao propagated this concept in the policies and campaigns of hundred flower
movement and cultural revolution.
 However, Deng’s ascendancy to power sidelined such ideas, and laid
emphasis on ‘rich is glorious’.
 Deng believed that allowing some people and regions to get rich first would
speed up economic growth and help achieve the ultimate goal of economic
modernisation.
 But such policy emphasis of Deng has proved to be a societal disaster.
 The call for ‘common prosperity’ will soon be taken into account in the
decision-making process.

What will  Curbing tax evasion


be done  Reducing limits on the hours that the tech sector employees can work
 Bans on for-profit tutoring in core school subjects
 Strict limits on the time minors can spend playing video games, among many
others.

Q.45) Which one of the following is the main source of nitrogen pollution in India?
(a) Agriculture
(b) Sewage wastes
(c) Organic solid wastes
(d) Thermal power plants
EXPLANATION:
 The Indian Nitrogen Assessment (INA) released the Assessment’s publication, India
had become the third country/entity after the United States and the European
Union to have assessed the environmental impact of nitrogen on their respective
regions comprehensively.
 The INA shows that agriculture is the main source of nitrogen pollution in India.
Within agriculture, cereals pollute the most. Rice and wheat take up the maximum
cropped area in India at 36.95 million hectares (ha) and 26.69 million ha
respectively.
So, option (a) is correct.

ADDITIONAL INFORMATION:
ABOUT  Nitrogen is one of the five major chemical elements that are
NITROGEN necessary for life. While nitrogen is the most abundant of these,
more than 99% of it occurs as molecular nitrogen, or N 2, which
cannot be used by most organisms.
 This is because breaking the triple bond holding the two
nitrogen atoms together requires a large amount of energy,
which can be mustered only through high-temperature
processes or by a small number of nitrogen-fixing microbes.
 Most living organisms can only make use of reactive nitrogen,
which includes inorganic forms of nitrogen like ammonia,
ammonium, nitrogen oxide, nitric acid, nitrous oxide, and
nitrate, and organic compounds like urea, amines, proteins,
and nucleic acids. It includes any nitrogen compound that is
radiatively, chemically or biological activity.
INTERNATIONAL  The International Nitrogen Initiative (INI) is an international
NITROGEN program, set up in 2003 under the sponsorship of the Scientific
INITIATIVE Committee on Problems of the Environment (SCOPE) and from
the International Geosphere-Biosphere Program (IGBP). The key
aims of the INI are to:
o optimize nitrogen’s beneficial role in sustainable food
production, and
o minimize nitrogen’s negative effects on human health and the
environment resulting from food and energy production.
 The program is currently a sustained partner of Future Earth.

Q.46) The Spatially Explicit Capture Recapture (SECR) and Extract Compare, often seen in news, is used
for the
(a) Roaming details of elephants
(b) Population of Tigers
(c) The spatial range of Lions
(d) Relocation projects for cheetah

EXPLANATION:
The M-STrIPES, is a GPS-enabled mobile application and helps to capture data relating to
tiger sightings, deaths, wildlife crime and ecological observations while patrolling. The other
software used for tiger estimation was Spatially Explicit Capture-Recapture (SECR) and
Extract Compare, which helps differentiate between the stripes of tigers as each has a
different pattern. So, Option (b) is correct.
ADDITIONAL INFORMATION:
Tiger Census  Double sampling based on ground-based surveys and actual images captured
Methodology on camera traps.
 The double sampling method was introduced in 2006 after the “pugmark”
surveys were found to be inaccurate.
 In the 2018 census, 83% of the big cats censused were individually
photographed using camera traps.
 In Phases 1 & 2, ground-based surveys were carried out by Forest Department
officials to collect signs of tiger presence like scat and pugmarks and recorded
through remote-sensing and GIS (MSTrIPES) mobile applications.
 In phase 3, the information was plotted on the forest map prepared with
Camera trap based Spatially Explicit Capture-Mark-Recapture.
 In phase 4, data were extrapolated to areas where cameras could not be
deployed.
MSTrIPES  MSTrIPES: Monitoring system for Tigers – Intensive Protection and Ecological
Status.
 It was launched across Indian tiger reserves by NTCA and WII in 2010.
 MSTrIPES is a software-based monitoring system, designed to assist wildlife
protection.
 Under MSTrIPES, forest guards are expected to record their tracks using a
GPS, in addition to recording observations in site-specific datasheets.
The Spatially  SECR and Extract Compare is used in phase 3 of tiger census 2018 which
Explicit helps differentiate between the stripes of tigers as each has a different pattern.
Capture  This approach makes use of two samples –
Recapture  The first sample is collected by the forest staff of 18 tiger states and is
(SECR) and constituted by structured protocols that are easy and economical to generate
Extract information on the presence of tigers and relative abundance, along with
Compare information on prey, co-predators, habitat and human impact.
 The second sample is carried out by trained wildlife biologists who collect
information using camera traps on tiger, leopard and prey abundance.
 Individual tigers and leopards are identified using customized software that
uses the stripe and spot patterns (similar to human fingerprints) to identify
individuals.

Q.47) In India, the water quality of different rivers in the country is being regularly monitored by
1. Central Pollution Control Board
2. State Pollution Control Boards
3. Pollution Control Committees
4. Central Water Commission
Select the correct answer using the code given below.
(a) 1 and 2 only
(b) 3 and 4 only
(c) 1, 2 and 3 only
(d) 1, 2, 3 and 4
EXPLANATION:
One of the mandates of the Central Pollution Control Board (CPCB) is to collect, collate
and disseminate technical and statistical data relating to water pollution.
Hence, Water Quality Monitoring (WQM) and Surveillance are of utmost importance.
CPCB has established a network of monitoring stations on rivers across the country.
So, Statement 1 is correct.
The State Pollution Control Board is entrusted with several functions under the
provisions of the Water (PCP) Act 1974, Air (PCP) Act 1981, Water Cess Act 1977, and
different Rules and notifications issued under the Environment (Protection) Act 1986.
The goal is to improve the quality of air, promote the cleanliness of water bodies, and
prevent pollution. So, Statement 2 is correct.
Pollution Control Committee (PCC) has been constituted jointly for the Union
Territories of Daman & Diu and Dadra & Nagar Haveli. Some of the acts enforced by
PCC are the Water (Prevention & Control of Pollution) Act, 1974, Air (Prevention &
Control of Pollution) Act, 1981, Environment (Protection) Act, 1986, and so on. The
committee is committed to providing a pollution-free environment to the people of the
state. The Board has undertaken various studies of underground water, solid, and air
to take remedial steps to control pollution. So, Statement 3 is correct.
Central Water Commission (CWC) is monitoring the water quality of rivers since the
late 1950s and presently its water quality network is spread all over India. The water
quality network of CWC is incidental to the hydrological observation network. Central
Water Commission is monitoring water quality at 552 key locations (519 water quality
sites and 33 water quality sampling stations) covering all the major river basins of
India. So, Statement 4 is correct.

ADDITIONAL INFORMATION:
Central Pollution  The Central Pollution Control Board (CPCB) is responsible for the
Control Board restoration and maintaining the wholesomeness of aquatic
resources under the Water (Prevention & Control of Pollution) Act
1974 passed by the Indian Parliament.
 To ensure that the water quality is being maintained or restored
at the desired level it must be monitored on regular basis.
 Water quality monitoring helps in evaluating the nature and
extent of pollution control required, and the effectiveness of
pollution control measures already in existence.
 It also helps in drawing the water quality trends and prioritizing
pollution control efforts.
State Pollution  The State Pollution Control Board is entrusted with several
Control Board functions under the provisions of the Water (PCP) Act 1974, Air
(PCP) Act 1981, Water Cess Act 1977, and different Rules and
notifications issued under the Environment (Protection) Act
1986.
 The main functions of the State Pollution Control Boards are as
follows:
o To plan a comprehensive program for the prevention,
control, and abatement of air pollution and to secure
the execution thereof.
o To advise the State Government on any matter
concerning prevention, control, and abatement of air
pollution.
o To collect and disseminate information related to air
pollution.
o To collaborate with Central Pollution Control Board in a
program related to prevention, control, and abatement
of air pollution; and
o To inspect air pollution control areas, assess the quality
of air, and take steps for prevention, control, and
abatement of air pollution in such areas.
Pollution Control  The Pollution Control Committee has
Committee been established as a regulatory authority for implementing
various pollution control laws.
 The PCC has ambitious plans to prepare an industrial database
on industries and release the same on the internet with the hope
that it will create new opportunities for the industrialists to
market their products globally making use of E-commerce and
information technology.
 The legal hurdles posed in the way of dissemination of data in
the electronic media are being examined.
Central Water  Central Water Commission (CWC) is monitoring the water quality
Commission of rivers since the late 1950s and presently its water quality
network is spread all over India.
 CWC is also maintaining a three-tier laboratory system for
analysis of the parameters. The level-I laboratories are located
at 295 field water quality monitoring stations on major rivers of
India where physical parameters such as temperature, color,
odor electrical conductivity, total dissolved solids, pH, and
dissolved oxygen of river water are observed.
 There are 18 level –II laboratories located at selected division
offices to analyze 25 nos. physicochemical characteristics and
bacteriological parameters of river water. 5 level-III / II+
laboratories are functioning at Varanasi, New Delhi, Guwahati,
Hyderabad, and Coimbatore where 41
parameters including heavy metals / toxic parameters are being
analyzed.
Q.48) Consider the following statements about the Noise Pollution (Regulation and Control) Rules, 2000:
1. It authorises the Central government to declare ‘silent zones’ which includes areas that lie within
100 meters of the premises of schools, colleges, hospitals and courts.
2. In industrial areas, the permissible limit is 75 dB for daytime and 70 dB at night.
3. A loudspeaker or a public address system shall not be used except after obtaining written
permission from the authority.
4. No horn shall be used during nighttime in residential areas except during a public emergency.
Which of the statements given above is/are correct?
(a) 1 and 2 only
(b) 3 and 4 only
(c) 2, 3 and 4 only
(d) 1, 2, 3 and 4
EXPLANATION:
Under Noise Pollution (Regulation and Control) Rules, 2000, State governments have
declared ‘silent zones’ which include areas that lie within 100 meters of the premises of
schools, colleges, hospitals, and courts. The permissible noise limit in this zone is 50 dB
during the day and 40 dB during the night. So, Statement 1 is not correct.
The CPCB has laid down the permissible noise levels in India for different areas. Noise
pollution rules have defined the acceptable level of noise in different zones for both daytime
and nighttime. In industrial areas, the permissible limit is 75 dB for daytime and 70 dB at
night. In commercial areas, it is 65 dB and 55 dB, while in residential areas it is 55 dB and
45 dB during daytime and night respectively. So, Statement 2 is correct.
Rule 5 (Restrictions on the use of loudspeakers/public address system) of the Noise Pollution
Rule says a loudspeaker or a public address system shall not be used except on obtaining
written permission from the authority concerned. It also says that a loudspeaker or a public
address system shall not be used at night (between 10 p.m. and 6.00 a.m.) except in closed
premises for communication within, like auditoria, conference rooms, community halls, and
banquet halls or during a public emergency. No loudspeaker could be used without obtaining
a license. Permission could not be granted to use loudspeakers in silent zones and if granted
the noise levels should be within the prescribed levels. So, Statement 3 is correct.
No horn shall be used in silence zones or during nighttime in residential areas except during
a public emergency. Sound emitting firecrackers shall not be burst in silence zone or during
night time. Sound emitting construction equipment shall not be used or operated during
night time in residential areas and silence zones. So, Statement 4 is correct.
ADDITIONAL INFORMATION:
Noise Pollution in India
Noise Pollution  Any unwanted sound that causes annoyance, irritation, and pain
to the human ear is termed noise. It is measured in A-weighted
decibels (dB (A)) that indicate the loudness of the sound.
 Noise level refers to the decibel levels of noise produced by any
appliance or machine. In general, the human ear can tolerate
noise levels up to 85 dB. Anything beyond that can affect their
productivity and quality of life.
 The decibel levels of common sounds above 80 dB are considered
‘loud’, while the decibel levels of common sounds between 100-
125 dB are termed ‘uncomfortable’.
 All machines operating in an area should produce noise within
the acceptable level to maintain the well-being of people around.
Consequences of  Regular noise exposure can come out in the form of people being
Noise Pollution irritable, nervous, and facing difficulty in taking decisions.
 It has been shown to hinder the normal development of speech
and hearing in children, resulting in delayed developmental
milestones affecting their overall growth.
Problems in India  Noise has increasingly become a nuisance in India, especially in
urban areas and it, thus, becomes important for authorities to
make more efforts to control and regulate noise pollution to
protect millions of people. There are 70 noise monitoring stations
under the National Ambient Noise Monitoring Network established
in 2011 in seven cities: Bangalore, Chennai, Delhi, Hyderabad,
Kolkata, Lucknow, and Mumbai.
 The ambient noise level data of these seven cities showed about
90 percent of these stations found noise levels beyond accepted
limits both during day and night. At some stations, noise levels
recorded were nearly more than double the allowed norms.
Government  Earlier, noise pollution and its sources were addressed under the
Regulations Air (Prevention and Control of Pollution) Act, 1981.
 They are now, however, regulated separately under the Noise
Pollution (Regulation and Control) Rules, 2000.
 Additionally, noise standards for motor vehicles, air-conditioners,
refrigerators, diesel generators, and certain types of construction
equipment are prescribed under the Environment (Protection)
Rules, 1986.
 Noise emanating from industry is regulated by State Pollution
Control Boards / Pollution Control Committees (SPCBs / PCCs)
for states / Union territories under the Air (Prevention and
Control of Pollution) Act, 1981.
Permissible Noise  The CPCB has laid down the permissible noise levels in India for
Level in India different areas. Noise pollution rules have defined the acceptable
level of noise in different zones for both daytime and nighttime.
 In industrial areas, the permissible limit is 75 dB for daytime and
70 dB at night.
 In commercial areas, it is 65 dB and 55 dB, while in residential
areas it is 55 dB and 45 dB during daytime and night
respectively.
 Additionally, state governments have declared ‘silent zones’ which
include areas that lie within 100 meters of the premises of
schools, colleges, hospitals, and courts. The permissible noise
limit in this zone is 50 dB during the day and 40 dB during the
night.

Q.49) Consider the following statements about the radioactive pollution in water:
1. Radioactive contamination is more prevalent in surface water as compared to groundwater.
2. Thorium and Actinium are naturally occurring radioactive materials that contaminate water
resources.
3. Oceans and seas are the natural repositories of naturally occurring Uranium.
Which of the statements given above is/are correct?
(a) 1 and 2 only
(b) 2 and 3 only
(c) 1 only
(d) 1, 2 and 3
EXPLANATION:
Radioactive contamination is more prevalent in groundwater as compared to
surface water since it is much exposed to radioactive elements found in the rocks.
Sometimes, magma also releases radioactive gases into the environment.
Percolation of naturally occurring radioactive materials (NORM) from the soil
sediments to the aquifer causes groundwater contamination. So, Statement 1 is
not correct.
Several radionuclides are found in surface and subsurface waters, among
which 3H, 14C, 40K, 210Pb, 210Po, 222Rn, 226Ra, 228Ra, 232Th,
and 234,235,238U are common. Uranium, thorium, and actinium are three NORM
series that contaminate water resources. Mining activities of radioactive elements
like uranium and thorium also pollute surface and groundwater. So, Statement 2
is correct.
Oceans and seas are the natural repositories of naturally occurring uranium.
Where it is found in the form of uranyl carbonate ions. A significant concentration
of uranium is supposed to be found in the greater salinity of marine water. 40K is
also found in considerable concentration in the marine environment. So,
Statement 3 is correct.

ADDITIONAL INFORMATION:
Water Pollution
Pollutants  Water pollution is a severe concern for the current generation.
We are familiar with water contaminants like major ions, heavy
metals, dyes, and organic pollutants.
 However, radioactive pollution of water is newly emerging but is
of grave concern for water pollution and human health.
Recently, radioactive contamination and associated health
impacts have been reported in many parts of the globe.
Radioactivity  Radioactivity is the phenomenon of spontaneous emission of
particles or waves from the unstable nuclei of some elements.
There are three types of radioactive emissions: Alpha, Beta,
and Gamma.
 Alpha particles are positively charged He atoms, beta particles
are negatively charged electrons and gamma rays are neutral
electromagnetic radiations. Radioactive elements are naturally
found in the earth’s crust.
Radioactive  Percolation of naturally occurring radioactive materials (NORM)
Pollution from the soil sediments to the aquifer causes groundwater
contamination. Uranium, thorium, and actinium are three
NORM series that contaminate water resources.
 In anthropogenic sources of radioactive pollution, nuclear
weapon investigation, nuclear calamities, nuclear
powerhouses, and dumping of radioactive waste are major
sources of contamination, while application of radioisotopes in
industries and scientific laboratories are the minor sources.
 The application of radioactive elements in nuclear weapons, X-
rays, MRI, and other medical equipment causes their exposure
to human beings. Dumping of these radioactive wastes in
surface water bodies causes water pollution.
Natural Source of  Radioactive contamination is more prevalent in groundwater as
Contamination compared to surface water since it is much exposed to
radioactive elements found in the rocks. Sometimes, magma
also releases radioactive gases into the environment.
 The deposition of these radioactive gases in water bodies also
causes radioactive contamination. Atmospheric deposition
(both dry and wet) of cosmogenic radionuclides also add
radioactive nuclei to the surface water.
Guidelines  A small amount of radiation is found in all types of water but
the extended amount of radiation is harmful to human health.
Radioactivity in drinking water can be determined by a gross
alpha test.
 The World Health Organization set guidelines for drinking
water quality and a permissible limit of reference dose level of
0.1 micro-sieverts per year.
Health Hazards  Immediate, recoverable consequences distressing skin,
lungs, genitals, and causing hair to fall
 High doses of radiation can cause acute radiation
syndrome or cutaneous radiation injury.
 Exposure to radiation causes various disorders in
human physiology, including cancer, leukemia, genetic
mutations, osteonecrosis, cataracts,
and chromosomal disruption.
 Genetic effects ionizing radiation induces mutations in germ
cells (male sperm cells and female egg cells) or germ cells,
resulting in structural alteration in germ cell
DNA that is passed on to offsprings. Hereditary disorders can
lead to premature death and severe mental illness.
Prevention  Prevention and precaution measures can check the
anthropogenic sources of radioactive contamination in water
resources.
 Various treatment methods like aeration, reverse osmosis, ion
exchange, and granule carbon adsorption are effective remedial
measures for treating radioactively contaminated water.

Q.50) Consider the following statements about the ‘Awareness Programme on Environmental Hazards of
Electronic waste’:
1. It is an initiative of the Union Ministry of Environment, Forest and Climate Change.
2. It is implemented under the ‘Digital India’ initiative of the Government of India.
3. The programme covers all the States and Union Territories of India.
Which of the statements given above is/are correct?
(a) 1 and 2 only
(b) 2 and 3 only
(c) 1 only
(d) 1, 2 and 3
EXPLANATION:
The Ministry of Electronics and Information Technology (MeitY) has initiated the project
“Awareness Programme on Environmental Hazards of Electronic waste” on March 31,
2015. The project is expected to have a far-reaching and significant impact on the
growth of the country as it focuses on the reuse and recycling of e-waste, which has the
potential to conserve natural resources. So, Statement 1 is not correct.
Awareness Programme on Environmental Hazards of Electronic waste project is under
the ‘Digital India’ initiative of the Government of India. The project has three
components viz., Content Development, Inventory Assessment, and Awareness
Generation amongst different stakeholders. The project will help in the effective
implementation of E-waste Management, 2016. So, Statement 2 is correct.
In phase-II proposals are also invited for assessment of the quantum or Inventory of e-
waste in all the states and UT except for 10 states, namely Madhya Pradesh, Uttar
Pradesh, Jharkhand, Orissa, Goa, Bihar, Pondicherry, West Bengal, Assam, and
Manipur. (Inventory study in these states has already been undertaken during Phase-I).
So, Statement 3 is correct.

ADDITIONAL INFORMATION:
Awareness Programme on Environmental Hazards of Electronic waste
MeitY Ministry of Electronics and Information Technology (MeitY) had initiated
the “Awareness Program on Environmental Hazards of Electronic Waste
through Digital India Initiative” in 2015 to create awareness for hazards of
the recycling methods being used by the unorganized sector vis-à-vis best
practices available for environment-friendly recycling.
Objective  The primary focus of the project is to create awareness among
different stakeholders to reduce the adverse impact on the
environment and health due to improper disposal of e-waste. MietY
has played a key role in the dissemination of knowledge on e-waste
rules in the past and wishes to engage all key stakeholders during
this exercise.
Phase I  During the project duration of 5 years, a city in each of the 10
identified states viz. Madhya Pradesh, Uttar Pradesh, Jharkhand,
Orissa, Goa, Bihar, Pondicherry, West Bengal, Assam, and
Manipur will be covered.
 The activities will include organizing awareness workshops for
RWAs/Localities, Schools, Colleges, Bulk Consumers (including
corporate & Govt. sectors), Informal Sector, Dealers, Refurbishers,
Manufacturers, etc., to build capacities of the target groups to
channelize e-waste in a manner that the rules are effectively
implemented.
 A suitable course curriculum would also be framed for
schools/colleges. Efforts would be made to prepare the content in
the local language.
Phase II  In the second phase, a total of 20 states/UTs, namely, Andhra
Pradesh, Andaman, and Nicobar Island, Chhatisgarh, Daman and
Diu, Delhi, Gujarat, Haryana, Himachal Pradesh, Karnataka,
Kerala, Lakshadweep, Maharashtra, Meghalaya, Punjab,
Rajasthan, Sikkim, Tamil Nadu, Telangana, Tripura, and
Uttarakhand would be covered. The cities would, however, be
chosen during the evaluation and identification of the project
proposals.
 The project proposal(s) should cover e-waste awareness activities
and/or workshops related to the stakeholder(s) (schools, colleges,
RWAs and Localities, bulk consumers, dealers, refurbishers,
informal sector, and manufacturers) or e-waste inventorization in
the States/UTs indicated above.
Expected  Recycling recovers valuable materials from old electronics that can
Outcomes be used to make new products. As a result, we save energy, reduce
pollution, reduce greenhouse gas emissions, and save resources by
extracting fewer raw materials from the earth.
 Recycling will create jobs for professional recyclers and
refurbishers and create new markets for the valuable components
that are dismantled.
 The project will also take care of the responsibilities of the
producers that they must inculcate the Principle of EPR and follow
the mechanism for channelization of e-waste to registered
dismantlers or recyclers.

Q.51) The term ‘Physellaacuta’ sometimes seen in the news recently, is


(a) An invasive snail species
(b) Anew marine reptile species
(c) A wasp like flower flyspecies
(d) A wild Sun Rosespecies
EXPLANATION:
During the biodiversity impact assessment carried out for the Integrated Urban Regeneration
and Water Transport System (IURWTS) in Kochi, the team come across acute bladder
snail Physella acuta, globally branded as highly invasive. So, Option (a) is correct.
ADDITIONAL INFORMATION:
Physella acuta

About A tiny snail with a striking, pellucid golden-yellow shell. Small in size,
the snail can grow to 16 mm in height and 9 mm in width. The dead,
vacant shell is brownish-yellow while that of the live individuals is
translucent golden-yellow with a mottled appearance. Physella acuta is
easily identified by its sinistral (left-opening aperture) shell. Its good
looks make this snail a favorite of aquariums but special care should be
taken.
Recent Findings It was found in the Edappally canal in Kochi by the researchers of the
Department of Marine Biology, Microbiology, and Biochemistry of the
Cochin University of Science and Technology (CuUSAT) during a
biodiversity impact assessment study.
Harm to Humans It plays host to worms that can cause food-borne diseases and skin
itches in humans. Moreover, its rapid growth rate, air-breathing
capability, and tolerance to pollution makes the Physella acuta a
potential competitor to native fauna
Threat to The CUSAT study also draws attention to the threat posed by invasive
Economy species to global biodiversity; how their ability to quickly dominate new
environments endanger native species, even causing serious economic
loss.

Q.52) Consider the following statements about the Red Sanders:


1. It is an Indian endemic tree species, with a restricted geographical range in the Western Ghats.
2. It is classified as ‘near threatened’ in the International Union for Conservation of Nature’s Red List.
3. It is listed under Appendix II of the Convention on International Trade in Endangered Species.
Which of the statements given above is/are correct?
(a) 1 and 2 only
(b) 2 and 3 only
(c) 3 only
(d) 1, 2 and 3
EXPLANATION:
Red Sanders or Red Sandalwood (Pterocarpus santalinus) is an Indian endemic tree species, with
a restricted geographical range in the Eastern Ghats, a distinct tract of forests in Andhra
Pradesh and some parts of Tamil Nadu and Karnataka. So, Statement 1 is not correct.
Recently, In 2022, the IUCN’s Red List put Red Sanders back into the ‘endangered’ category from
Near-threatened and summarized its population saw a sharp decline of 50-80% over the last
three generations. So, Statement 2 is not correct.
It is listed under Appendix II of CITES and is banned from international trade. However, in 2019
Directorate General of Foreign Trade (DGFT), permitted the export of red sander, if it is obtained
from cultivated land, but despite this illegal trade continues. So, Statement 3 is correct.

ADDITIONAL INFORMATION:
Why in News?  The latest IUCN assessment stated that the ‘Red Sanders population
saw a sharp decline of 50-80% over the last three generations’.
 Red Sanders (Red Sandalwood) has fallen back into the ‘Endangered’
category in the International Union for Conservation of Nature’s
(IUCN) Red List.
Red Sanders  It is an Indian endemic tree species, with a restricted geographical
(Pterocarpus range in the Eastern Ghats, a distinct tract of forests in Andhra
santalinus) Pradesh and some parts of Tamil Nadu and Karnataka.
 This light-demanding species grows in the rocky, degraded and fallow
lands with Red Soil and a hot and dry climate.
Speciality  Red Sanders is known for its rich hue and therapeutic properties.
 They are high in demand across Asia, particularly in China and
Japan, for use in cosmetics and medicinal products as well as for
making furniture, woodcraft and musical instruments.
Protection  IUCN Red List-Endangered
Status  CITES-Appendix II (Controlled international trade)
 Wildlife Protection Act, 1972-Schedule II (Harvest of the tree is also
restricted at the state level).
India Trade  In 2019 Directorate General of Foreign Trade (DGFT), permitted the
Policy export of red sander if it is obtained from cultivated land, but despite
this illegal trade continues.
 In 2012, India got an export quota on Red Sanders from CITES, under
which the country could export 310 tonnes of red sanders obtained
from “artificially propagated” sources (grown on farms) and 11,806
tonnes of wood from seized sources.
The  It is an international agreement to which States and regional economic
Convention integration organizations adhere voluntarily.
on  Aims to ensure that international trade in specimens of wild animals
International and plants does not threaten their survival.
Trade in  Located in Geneva, Switzerland.
Endangered  It is legally binding on the Parties, However, it does not take the place
Species of of national laws.
Wild Fauna  It provides a framework to be respected by each Party, which has to
and Flora adopt its own domestic legislation to ensure that CITES is
(CITES) implemented at the national level.
Functions  CITES functions through Licensing system by including species in
Appendices I, II and III which affords the different levels or types of
protection from over-exploitation.
Appendix I  It lists species that are threatened with extinction among CITES-listed
animals and plants.
 Prohibits international trade in specimens of these species except
when the purpose of the import is not commercial, for instance for
scientific research.
 For example Gorillas, Tigers, Leopards, Asiatic lions and Asian
elephants.
Appendix II  It lists species that are not necessarily threatened with extinction but
that may become so unless trade is closely controlled.
 No import permit is necessary for these species under CITES (although
a permit is needed in some countries that have taken stricter
measures than CITES requires).
 For example African Lion and Red sanders tree.
Appendix III  It is a list of species included at the request of a Party that already
regulates trade in the species and that needs the cooperation of other
countries to prevent unsustainable or illegal exploitation.
 For example Map turtles and Walruses.
Q.53) Consider the following statements about the Particulate Matter (PM) pollution
1. Both PM2.5 and PM10 are capable of penetrating deep into the lungs.
2. Particulate Matter is primarily generated by fuel combustion.
3. Particulate Matter is not still classified as carcinogenic by World Health Organization
Which of the statements given above is/are correct?
(a) 1 and 2 only
(b) 1 and 3 only
(c) 2 only
(d) 1, 2 and 3
EXPLANATION:
Both PM2. 5 and PM10 are capable of penetrating deep into the lungs but PM2. 5 can even
enter the bloodstream, primarily resulting in cardiovascular and respiratory impacts, and
also affecting other organs. So, Statement 1 is correct.
Particulate Matter is primarily generated by fuel combustion in different sectors including
transport, energy, households, industry and agriculture. So, Statement 2 is correct.
In 2013, outdoor air pollution and particulate matter were classified as carcinogenic by
WHO’s International Agency for Research on Cancer (IARC). So, Statement 3 is not
correct.

ADDITIONAL INFORMATION:
Particulate  Particulate Matter (also called particle pollution) is a mixture of
Matter solid particles and liquid droplets found in the air with some
particles, such as dust, dirt, soot, or smoke.
Types of  PM10: Inhalable particles, with diameters that are generally 10
Particulate micrometres. For example Dust, Mold etc.
Matter  PM 2.5: fine inhalable particles, with diameters that are
generally 2.5 micrometres (more than 100 times thinner than
human hair) and remain suspended for longer. It is a primary
reason for the occurrence of smog.
Source  It is primarily generated by fuel combustion (anthropogenic) in
different sectors including transport, energy, households,
industry and agriculture.
 Other primary source includes natural phenomenon such as
windblown dust and forest fires.
 Secondary PM sources directly emit air contaminants into the
atmosphere. Hence, these pollutants are considered precursors
to PM formation.
 These secondary pollutants include SOx, NOx, VOCs, and
ammonia.
Important  Natural Causes – Volcanoes, Dust storms, Forest and grassland
Causes fires, living vegetation, Seaspray, Tornadoes and hurricanes.
 Man-Made Causes - Coal Combustion, Oil Combustion, Wood
combustion, Demolition, Road dust, Power plants, Industries,
Agriculture, Livestock, Deforestation, Poor condition of anti-
pollution technology and Tobacco smoke.
Effects  Both PM2.5 and PM10 are capable of penetrating deep into the
lungs but PM2.5 can even enter the bloodstream, primarily
resulting in cardiovascular and respiratory impacts, and also
affecting other organs.
 A recent study by IIT Delhi has tried to establish the association
between ambient PM 2.5 exposure and the occurrence of
anaemia among children under the age of 5 years of age in India.
 Other Effects on climate such as Weather, Drought, Rainfall
declines, Greenhouse and Global dimming
Recent Policy  The upper limit of annual PM2.5 was revised to five micrograms
Shift by WHO- per cubic metre as compared to 10 micrograms per cubic metre
Global Air (2005 standards, which is what countries now follow).
Quality  The 24-hour value for the PM2.5 ceiling used to be 25
Guidelines micrograms but has now dropped to 15.
2021  The PM10 upper limit is 20 micrograms and has now been
revised to 15 whereas the 24-hour value has been revised from
50 to 45 micrograms.
 However, the move doesn’t immediately impact India as the
National Ambient Air Quality Standards (NAAQS) don’t meet the
WHO’s existing standards. In India, the standards for ambient
air quality are set by the Central Pollution Control Board.
National Clean  It is a five-year action plan with a tentative target of 20-30%
Air Programme reduction in concentrations of PM10 and PM2.5 by 2024, with
2017 as the base year.
 Unlike, the Graded Response Action Plan (GRAP) which focused
on the Air Quality of Delhi-NCR, NCAP is a comprehensive pan-
India air pollution abatement scheme for 102 cities.
 Such cities were identified by Central Pollution Control Board
(CPCB) on the basis of their ambient air quality data between
2011 and 2015.
 The plan proposes a three-tier system, including real-time
physical data collection, data archiving, and an action trigger
system.
Loopholes of  The pollution reduction target in these cities is not ‘legally
NCAP binding’ on respective states.
 A reduction of merely 20-30% from the 2017 level by 2024 will
not be sufficient to bring the air quality to the desired level, i.e.
the targets under the action plan are less ambitious.
 Because of this, National Green Tribunal (NGT) has directed the
Ministry of Environment, Forest and Climate Change (MoEFCC)
to modify the NCAP by reducing the five-year target.

Q.54) With reference to the global air quality guidelines 2021, the World Health Organization provides
specific recommendations on air quality guideline levels for the
1. Particulate Matters
2. Ozone
3. Ammonia
4. Nitrogen dioxide
5. Sulfur dioxide
6. Lead
7. Carbon monoxide
Select the correct answer using the code given below.
(a) 1, 2, 4, 5 and 7 only
(b) 1, 2, 3 and 6 only
(c) 3, 4, 5 and 7 only
(d) 1, 2, 3, 4, 5, 6 and 7
EXPLANATION:
Recently, WHO updated its 2005 global air quality guidelines. The guidelines are not binding and
included two new air quality levels (Ozone (O3) and Carbon monoxide (CO)) along with existing four
major pollutants such as Particulate matter (PM 2.5), Particulate matter (PM10), Nitrogen dioxide(NO2)
and Sulfur dioxide (SO2). So, Option (a) is correct.

ADDITIONAL INFORMATION:
National  NAAQS was adopted in 1982 for the first time by the country.
Ambient Air  The Central Pollution Control Board (CPCB) under the Air (Prevention and
Quality Control of Pollution) Act empowers it to set standards for the quality of air.
Standards  The measurement of air quality was revised in 1994 and 2009 and is based
(NAAQS) on eight pollutants for which short-term (up to 24-hourly averaging period).
Pollutant  Particulate Matter (PM 2.5)
Covered  Particulate Matter (PM 10)
under NAAQS  Nitrogen dioxide
 Sulphur dioxide
 Carbon monoxide
 Ozone
 Benzene and
 Ammonia
International  Recently, WHO updated its 2005 global air quality guidelines.
Standards  The guidelines are not binding.
 Included two new air quality levels (Ozone (O3) and Carbon monoxide (CO))
along with existing four major pollutants such as Particulate matter (PM 2.5),
Particulate matter (PM10), Nitrogen dioxide(NO2) and Sulfur dioxide (SO2)
European  The European Union has also developed legislation on air quality standards.
Union  The pollutants for which the standards are specified are:
Standards  PM2.5
 Sulphur dioxide (SO2)
 Nitrogen dioxide (NO2)
 PM10
 Carbon monoxide (CO)
 Benzene
 Ozone
 Lead (Pb)
 Arsenic (As)
 Cadmium (Cd)
 Nickel (Ni)
 Polycyclic Aromatic Hydrocarbons
Ultra-Fine  Ultrafine particles (UFPs) are a particulate matter of nanoscale size (less than
Particles 0.1 μm or 100 nm in diameter).
(UFPs)  Regulations do not exist for this size class of ambient air pollution particles,
which are far smaller than the regulated PM10 and PM2.5 particle classes.
 It is believed to have several more aggressive health implications than those
classes of larger particulates.
 Natural sources: Hot volcanic lava, ocean spray, and smoke.
 Man-made Sources: Combustion reactions, or equipment such as printer
toner and automobile exhaust.
Q.55) ‘Kunming Declaration’ sometimes seen in the news recently, is related to the affairs of
(a) United Nations Framework Convention on Climate Change
(b) United Nations Convention on Biological Diversity
(c) United Nations Convention to Combat Desertification
(d) Convention on the Conservation of Migratory Species
EXPLANATION:
 The Kunming Declaration was adopted by over 100 countries on October 13, 2021,
at the first part of the ongoing virtual 15th meeting of the Conference of the Parties
to the United Nations Convention on Biological Diversity.
 It calls upon the parties to “mainstream” biodiversity protection in decision-making
and recognizes the importance of conservation in protecting human health.
 By adopting this, the nations have committed themselves to support the
development, adoption, and implementation of an effective post-2020
implementation plan, a capacity-building action plan for the Cartagena Protocol on
biosafety.
 The theme of the declaration is Ecological Civilization: Building a Shared Future for
All Life on Earth.
So, option (b) is correct.

ADDITIONAL INFORMATION:
UN CONVENTION ON BIOLOGICAL DIVERSITY
ABOUT  The convention was opened for signature at the Earth Summit in Rio de
Janeiro on 5 June 1992 and entered into force on 29 December 1993. It
has two supplementary agreements, the Cartagena Protocol and Nagoya
Protocol.
 The CBD’s governing body is the Conference of the Parties (COP). This
ultimate authority of all governments (or Parties) that have ratified the treaty
meets every two years to review progress, set priorities, and commit to work
plans.
 The Secretariat of the Convention on Biological Diversity (SCBD) is based in
Montreal, Canada. Its main function is to assist governments in the
implementation of the CBD and its programs of work, to organize meetings,
draft documents, coordinate with other international organizations and
collect and spread information. The Executive Secretary is the head of the
Secretariat.

GOALS  The Convention on Biological Diversity (CBD), known informally as


the Biodiversity Convention, is a multilateral treaty.
 The convention has three main goals:
o the conservation of biological diversity (or biodiversity);
o the sustainable use of its components; and
o the fair and equitable sharing of benefits arising from genetic resources.
 Its objective is to develop national strategies for the conservation and
sustainable use of biological diversity, and it is often seen as the key
document regarding sustainable development.
Q.56) Consider the following statements about the Model Code of Conduct (MCC).
1. It is a set of guidelines issued by the Election Commission of India for the conduct of political
parties and candidates during elections.
2. The applicability of this code during bye-election would be only to the concerned district(s)
comprising the Parliamentary/ Assembly Constituency going to such bye-election.
3. In case the constituency is comprised of State Capital/ Metropolitan Cities/ Municipal
Corporations, then this code will apply only to the particular constituency, not for the entire
district.
Which of the statements given above are correct?
(a) 1 and 2 only
(b) 1 and 3 only
(c) 2 and 3 only
(d) 1, 2 and 3
EXPLANATION:
The Model Code of Conduct for the guidance of political parties and candidates is a set
of norms that have been evolved with the consensus of political parties who have
consented to abide by the principles embodied in the said code and also binds them to
respect and observe it in its letter and spirit. So, Statement 1 is not correct.
During bye-elections, in case the constituency is comprised of State
Capital/Metropolitan Cities/Municipal Corporations, then MCC instructions would be
applicable in the area of concerned Constituency only. In all other cases, aforesaid
instructions would be enforced in the entire district(s) covering the Constituency going
for bye-election(s). The spirit of these instructions has been that developmental and
administrative works should continue without the implications of MCC and the
campaigning for the bye election should be restricted in the PC/AC concerned only. So,
Statements 2 and 3 are correct.

ADDITIONAL INFORMATION:
Model Code of Conduct
Model Code of The Model Code of Conduct for the guidance of political parties
Conduct and candidates is a set of norms that have been evolved with the
consensus of political parties who have consented to abide by the
principles embodied in the said code and also binds them to
respect and observe it in its letter and spirit.
Role of The Election Commission ensures its observance by political
Election party/parties in power, including ruling parties at the Centre and
Commission in the States and contesting candidates in the discharge of its
constitutional duties for conducting the free, fair, and peaceful
elections to the Parliament and the State Legislatures under
Article 324 of the Constitution of India. It is also ensured that
official machinery for electoral purposes is not misused.
Enforcement The Model Code of Conduct is enforced from the date of
announcement of election schedule by the Election Commission
and is operational till the process of elections is completed.
Applicability General During general elections to the House of
Elections People (Lok Sabha), the code is applicable
throughout the country. During general
elections to the Legislative Assembly
(Vidhan Sabha), the code is applicable in
the entire State.
Bye-Elections During bye-elections, in case the
constituency is comprised in State
Capital/Metropolitan Cities/Municipal
Corporations, then the code would be
applicable in the area of concerned
Constituency only. In all other cases, the
MCC would be enforced in the entire
district(s) covering the Constituency going
for bye-election(s).

Salient The salient features of the Model Code of Conduct lay down how
Features political parties, contesting candidates, and party(s) in power
should conduct themselves during the process of elections i.e. on
their general conduct during electioneering, holding meetings
and processions, poll day activities, and functioning of the party
in power, etc.

Q.57) ‘Raimona National Park’ sometimes seen in the news recently, is located in the state of
(a) Assam
(b) West Bengal
(c) Odisha
(d) Chhattisgarh
EXPLANATION:
Recently, the Assam government created the Raimona National Park in western Assam and it will be
administered by the Kachugaon Forest Division of Bodoland Territorial Council. It has 11 different
forest types and subtypes and is home to the golden langur, elephant, tiger, clouded leopard and
Indian gaur. So, Option (a) is correct.

ADDITIONAL INFORMATION:
What in News?  Recently, the Assam government created the Raimona National Park as the
sixth National Park and Dihing Patkai as the seventh National Park of the
state.
 With this, Assam now has the third most National Parks after Madhya
Pradesh (12) and the Andaman and Nicobar Islands (9).
Raimona  Raimona National Park is an integral part of the 2,837 sq. km Manas
National Park Biosphere Reserve and the Chirang-Ripu Elephant Reserve.
 It will be administered by the Kachugaon Forest Division of Bodoland
Territorial Council.
 The National Park has 11 different forest types and subtypes that are home
to the golden langur, elephant, tiger, clouded leopard and Indian gaur.
Boundaries and  Adjoins the Buxa Tiger Reserve in West Bengal to its West.
Rivers  Manas National Park to the East.
 Phipsoo Wildlife Sanctuary in Bhutan to its North.
 On the west by the Sonkosh river, on the east by the Saralbhanga river and
on the south by the Pekua river.
Dihing Patkai  Dehing is the name of the river that flows through this forest and Patkai is
National Park the hill at the foot of which the sanctuary lies.
 The Dihing Patkai is located in eastern Assam (a major elephant habitat).
 The forest village area diverted under Forest Conservation Act has been
excluded.
Boundaries and  It encompasses the erstwhile Dehing Patkai Wildlife Sanctuary, Jeypore
Rivers Reserve Forest and the western block of Upper Dihing Reserve Forest.
 Rivers flowing inside the park are Dehing (Buri Dihing) and Dirak.
Dehing Patkai  Dehing Patkai Wildlife Sanctuary is known as the Jeypore Rainforest.
Wildlife  It is the only sanctuary in India that is home to seven different species of
Sanctuary wild cats - tiger, leopard, clouded leopard, leopard cat, golden cat, jungle
cat and marbled cat.
 It is home to Chinese pangolin, flying fox, barking deer, serow, Malayan
giant squirrels, Assamese macaque and White Winged Wood Duck.
Other National  The five national parks that existed before the Raimona and Dihing Patkai
Parks of Assam National Park are Kaziranga, Manas, Nameri, Orang and Dibru-Saikhowa.

Q.58) Consider the following statements about AMRUT 2.0.


1. It targets 100% coverage of water supply by providing household tap connections in all statutory
towns in India.
2. It will provide 100% coverage of sewerage and septage in 500 AMRUT cities.
3. Rejuvenation of water bodies and urban aquifer management will be undertaken to augment
sustainable freshwater supply.
4. Under the Mission, freshwater bodies will be protected from getting polluted to make natural
resources sustainable.
5. The lakes and canal link plan has been formulated on the lines of the River Linking Plan under this
Mission.
Which of the statements given above are correct?
(a) 1, 2, 3 and 4 only
(b) 1, 2 and 4 only
(c) 2, 3 and 4 only
(d) 1, 2, 3, 4 and 5
EXPLANATION:
The Objective of AMRUT 2.0 is to provide 100% coverage of water supply to all households in
around 4,700 ULBs, by providing 2.68 crore urban household tap connections, thereby
benefitting around 10.7 crores people. So, Statement 1 is correct.
The Mission also envisages providing universal household coverage of sewerage/septage services
in 500 AMRUT cities through 2.64 crore new sewer connections/coverage with septage
management, thereby benefitting around 10.6 crores people. So, Statement 2 is correct.
Rejuvenation of water bodies and green spaces and parks are other components of the Mission.
The Aquifer Management Plan will focus on maintaining a positive groundwater balance in urban
aquifer systems. So, Statement 3 is correct.
The Mission ensures diverting the polluting drains to treatment plants, it also has the provision
of STP to treat inflow into a water body. So, Statement 4 is correct.
India’s river linking project itself is in the initial stages of implementation and no plan has been
made to link lakes and canals. AMRUT 2.0 mission doesn’t formulate any plan related to river
linking. So, Statement 5 is not correct.

ADDITIONAL INFORMATION:
AMRUT 2.0
Objective The Objective of AMRUT 2.0 is to provide100% coverage of water supply to
all households in around 4,700 ULBs, by providing 2.68 crore urban
household tap connections, thereby benefitting around 10.7 crores people.
It will provide 100% coverage of sewerage and septage in 500 AMRUT cities,
by providing 2.64 crore sewer connections/ septage connections, thereby
benefitting around 10.6 crores people.
Sustainable Rejuvenation of water bodies and urban aquifer management will be
Development undertaken to augment sustainable freshwater supply. Recycling and reuse
of treated wastewater are expected to cater to 20% of the total water needs
of the cities and 40% of industrial demand. Under the Mission, freshwater
bodies will be protected from getting polluted to make natural resources
sustainable.
Features  These include upscaling from 500 cities covered under AMRUT with
1 lakh+ population to all 4,372 cities, covering 100% urban India.
 It will promote the circular economy of water through the
formulation of the City Water Balance Plan for each city, focusing on
recycling/reuse of treated sewage, rejuvenation of water bodies, and
water conservation.
 The digital economy will be promoted through being a Paperless
Mission.
Pey Jal Pey Jal Survekshan will be conducted in cities to ascertain the equitable
Survekshan distribution of water, reuse of wastewater, and mapping of water bodies
concerning quantity and quality of water through a challenging process.
Technology Sub-Mission for water will leverage the latest global
technologies in the field of water.
Employment The Mission seeks to promote Aatma Nirbhar Bharat through encouraging
Opportunities Startups and Entrepreneurs. It will lead to the promotion of the GIG
economy and on-boarding of youth & women. Urban Water Information
System through NRSC will be developed, leading to Aquifer Management
system.
Capacity building Information, Education and Communication campaign will spread
program awareness among masses about conservation of water. A target-
based capacity building program will be conducted for all stakeholders
including contractors, plumbers, plant operators, students, women, and
other stakeholders.
Public-Private The AMRUT-2.0 Mission will promote Public-Private partnerships (PPP). It
Partnership has been mandated for cities with having a million-plus population to take
up PPP projects worth a minimum of 10 percent of their total project fund
allocation which could be on the Annuity/ Hybrid Annuity / BOT Model.

Q.59) Which of the following international treaty restricts the production of Persistent Organic
Pollutants?
(a) Stockholm Convention
(b) Bonn Convention
(c) Washington Convention
(d) Vienna Convention
EXPLANATION:
The Stockholm Convention is a global treaty to protect human health and the
environment from persistent organic pollutants (POPs). POPs are chemicals that remain
intact in the environment for long periods, become widely distributed geographically,
accumulate in the fatty tissue of living organisms, and are toxic to humans and wildlife.
POPs circulate globally and can cause damage wherever they travel. In implementing
the Convention, Governments will take measures to eliminate or reduce the release of
POPs into the environment. So, option (a) is correct.
The Convention on the Conservation of Migratory Species of Wild Animals, also known
as the Convention on Migratory Species (CMS) or the Bonn Convention, is an
international agreement that aims to conserve migratory species throughout their
ranges. The Agreement was signed under the auspices of the United Nations
Environment Programme and is concerned with the conservation of wildlife and habitats
on a global scale. So, option (b) is not correct.
CITES (the shorter name for the Convention on International Trade in Endangered
Species of Wild Fauna and Flora, also known as the Washington Convention) is
a multilateral treaty to protect endangered plants and animals. It was drafted as a
result of a resolution adopted in 1963 at a meeting of members of the International
Union for Conservation of Nature (IUCN). The convention was opened for signature in
1973 and CITES entered into force on 1 July 1975. It aims to ensure that international
trade in specimens of wild animals and plants does not threaten the survival of the
species in the wild, and it accords varying degrees of protection to more than
35,000 species of animals and plants. So, option (c) is not correct.
The Vienna Convention on the Law of Treaties (VCLT) is an international
agreement regulating treaties between states, Known as the "treaty on treaties", it
establishes comprehensive rules, procedures, and guidelines for how treaties are
defined, drafted, amended, interpreted, and generally operated. An international treaty
is a written agreement between international law subjects reflecting their consent to the
creation, alteration, or termination of their rights and obligations. The VCLT is
considered a codification of customary international law and state practice concerning
treaties. So, option (d) is not correct.
Q.60) Which of the following statements about Mount Manipur, is/are correct?
1. It is one of the highest peaks in the Andaman and Nicobar Islands.
2. It is the highest peak in the North East Himalayas.
3. It is predominantly known as the Mount Harriet National Park.
Select the correct answer using the code given below.
(a) 1 only
(b) 1 and 3 only
(c) 2 only
(d) 2 and 3 only
EXPLANATION:
Mount Manipur is the third highest peak in the Andaman and Nicobar Islands and
served as the summer headquarters of the Chief Commissioner during British Raj. So,
Statement 1 is correct.
Recently, the Union government rechristened Mount Harriet, a historical tourist spot in
the Andaman and Nicobar islands, as ‘Mount Manipur’ to befit tribute to the exiled
heroes of the Anglo-Manipur War. So, Statement 3 is correct.
ADDITIONAL INFORMATION:
Mount Manipur

Mount Harriet Mount Harriet is the third highest peak in the Andaman and Nicobar
Islands and served as the summer headquarters of the Chief
Commissioner during British Raj.
It is believed to be named after British artist and photographer, Harriet
Christina Tytler, who was the wife of Robert Christopher Tytler, a
soldier who served in the British Indian Army. Between 1862 and 1864,
Tytler was the superintendent of the penal colony at Port Blair.
Anglo-Manipur Anglo-Manipur War was fought between the kingdom of Manipur and
War the British for over a month in 1891. The battle was triggered by a coup
in the palace of Manipur, which had been marked by internal
factionalism in the years leading up to 1891. The British government
took advantage of the “internal dissension” among the princes of the
royal family. The war led to Manipur officially becoming a princely state
under the indirect rule of the British crown.
Manipur’s After the Anglo-Manipur War of 1891, several Manipuris who had
Connection to fought the British in the war, including Maharaja Kulachandra Dhwaja
Mount Harriet Singh, were exiled to the British penal colony in the Andaman Islands.
Since the cellular jail (Kalapani) was yet to be built, Kulachandra and
the prisoners were kept on Mount Harriet, a hillock in what is now the
Ferragunj tehsil of South Andaman district.
Mount Manipur Recently, the Union government rechristened Mount Harriet, in the
Andaman and Nicobar islands, as ‘Mount Manipur’ to befit tribute to
the exiled heroes of the Anglo-Manipur War. Close by is the Mount
Harriet National Park which is also renamed Mount Manipur National
Park and it is known for its wide variety of birds.
Q.61) Consider the following statements about the Deputy Speaker of Lok Sabha.
1. The Constitution says that the election for the office of deputy speaker must be held within six
months since the vacancy rise.
2. When this office is vacant, then the Lok Sabha cannot function as per the constitution.
3. Once elected, the Deputy Speaker usually continues in office until the dissolution of the House.
Which of the statements given above is/are correct?
(a) 1 only
(b) 1 and 2 only
(c) 2 and 3 only
(d) 3 only
EXPLANATION:
Article 93 says: “The House of the People shall, as soon as may be, choose two members of
the House to be respectively Speaker and Deputy Speaker thereof and, so often as the office
of Speaker or Deputy Speaker becomes vacant, the House shall choose another member to
be Speaker or Deputy Speaker, as the case may be.” The Constitution says the election
must be held as soon as possible. So, Statement 1 is not correct.
When the Speaker presides over the House, the Deputy Speaker is like any other ordinary
member of the House. He can speak in the House, participate in its proceedings and vote on
any question before the House. Therefore when the office of Deputy Speaker falls vacant the
functions of the Lok Sabha are not affected in the next sitting the Lok Sabha elects another
member to fill the vacancy. So, Statement 2 is not correct.
Once elected, the Deputy Speaker usually continues in office until the dissolution of the
House. Under Article 94 (Article 179 for state legislatures), the Speaker or Deputy Speaker
“shall vacate his office if he ceases to be a member of the House of the People”. So,
Statement 3 is correct.

ADDITIONAL INFORMATION:
Deputy Speaker
Constitutional  Article 93 says: “The House of the People shall, as soon as may be,
Position choose two members of the House to be respectively Speaker and
Deputy Speaker thereof and, so often as the office of Speaker or
Deputy Speaker becomes vacant, the House shall choose another
member to be Speaker or Deputy Speaker, as the case may be.”
 Article 178 contains the corresponding position for Speaker and
Deputy Speaker of the Legislative Assembly of a state.
Election  All that the Constitution says is the election must be held as soon
as possible. Generally speaking, the practice in both Lok Sabha
and the state Legislative Assemblies has been to elect the Speaker
during the (mostly short) first session of the new House — usually
on the third day after oath-taking and affirmations take place over
the first two days.
 The election of the Deputy Speaker usually takes place in the
second session, even though there is no bar on having this election
too in the first session of the new Lok Sabha/Assembly.
 But the election of Deputy Speaker is generally not delayed beyond
the second session without genuine and unavoidable constraints.
Duration of Office  Once elected, the Deputy Speaker usually continues in office until
the dissolution of the House.
 Under Article 94 (Article 179 for state legislatures), the Speaker or
Deputy Speaker “shall vacate his office if he ceases to be a member
of the House of the People”.
 They may also resign (to each other), or “may be removed from
office by a resolution of the House of the People passed by a
majority of all the then members of the House”.
Powers  Article 95(1) says: “While the office of Speaker is vacant, the duties
of the office shall be performed by the Deputy Speaker”.
 In general, the Deputy Speaker has the same powers as the
Speaker when presiding over a sitting of the House. All references
to the Speaker in the Rules are deemed to be references to the
Deputy Speaker when he presides.
 It has been repeatedly held that no appeal lies to the Speaker
against a ruling given by the Deputy Speaker or any person
presiding over a sitting of the House in the absence of the Speaker.
Anti-Defection Law  Tenth Schedule (commonly known as the anti-defection law) says
that a person who has been elected Speaker/ Deputy Speaker
shall not be disqualified if he, because of his election to that office,
voluntarily gives up the membership of the political party to which
he belonged immediately before such election — and does not, so
long as he continues to hold such office thereafter, rejoin that
political party or become a member of another political party.
 This exemption applies to the Rajya Sabha Deputy Chairman,
Chairman/ Deputy Chairman of a state Legislative Council, and
Speaker/ Deputy Speaker of a state Legislative Assembly as well.

Q.62) Consider the following statements about the clouded leopard of India.
1. It is the smallest of the large wild cats in India.
2. It is categorised as ‘Vulnerable’ under the IUCN Red List of Threatened Species.
3. It is largely known to inhabit the low elevation evergreen rainforests.
4. It is endemic only to the North East Himalayas.
Which of the statements given above are correct?
(a) 1, 2 and 3 only
(b) 1, 2 and 4 only
(c) 1, 3 and 4 only
(d) 2, 3 and 4 only
EXPLANATION:
The tree-climbing clouded leopard (Neofelis nebulosa), a medium-sized felid (wild cat), is the
smallest of the large wild cats and is categorised as ‘Vulnerable’ under the IUCN Red List of
Threatened Species. Since they are largely known to inhabit low elevation evergreen rainforests, the
sighting is significant.
The clouded leopard is found across Southeast Asia and the Himalayas in the following countries:
southern China, Bhutan, Nepal, northeast India, Burma, Thailand, Vietnam, Malaysia, Cambodia,
Laos, and Bangladesh. It is believed to be extinct in Taiwan, China. So, option (a) is correct.
ADDITIONAL INFORMATION:
FISHING CATS
FACTS  The clouded leopard is named after the distinctive 'clouds' on its
coat - ellipses partially edged in black, with the insides a darker
color than the background color of the pelt.
 The base of the fur is a pale yellow to rich brown, making the darker
cloud-like markings look even more distinctive.
 The limbs and underbelly are marked with large black ovals, and the
back of its neck is conspicuously marked with two thick black bars.
 The clouded leopard is a medium-sized cat, 60 to 110 cm long and
weighing between 11 and 20kg.
 It does, however, have an exceptionally long tail for balancing, which
can be as long as the body itself, thick with black ring markings.
 The clouded leopard has a stocky build and, proportionately, the
longest canine teeth of any living feline.
HABITATS  Throughout its range, the clouded leopard spends most of the time
in the tropical evergreen rainforests but can also be found in dry
tropical forests and mangrove swamps. It has been found at
relatively high altitudes in the Himalayas.
THREATS  Many of the remaining forest areas are too small to ensure the long-
term persistence of clouded leopard populations. They are
threatened by habitat loss following large–scale deforestation and
commercial poaching for the wildlife trade.
 Skins, claws, and teeth are offered for decoration and clothing,
bones and meat as a substitute for body parts of
the tiger in traditional Asian medicines and tonics, and live animals
for the pet trade.
 Few poaching incidents have been documented, but all range states
are thought to have some degree of commercial poaching. In recent
years, substantial domestic markets existed in Indonesia, Myanmar,
and Vietnam.

Q.63) The threat of desertification in Punjab is looming at a large level. What could be the reasons for
this issue?
1. Drawing more water than is being replenished.
2. Predominant cultivation of paddy.
3. Puddling process or tillage of paddy field in a flooded condition.
4. Use of tubewell or submersible pumps for extraction of groundwater.
Select the correct answer using the code given below.
(a) 1, 2 and 3 only
(b) 1, 3 and 4 only
(c) 3 and 4 only
(d) 1, 2, 3 and 4
EXPLANATION:
 According to Central Ground Water Board (CGWB) officials, the study was titled ‘The
State of the World Report, 1998′, published by Washington-based World Watch Institute
(WWI). It said Punjab’s aquifers could be depleted by 2025.
 They are drawing more water than is being replenished, as a result of which the water
table is going down. This is posing the threat of desertification for Punjab. The rate of
water extraction in Punjab is 1.66 times the rate of replenishment.
 Because of the adoption of a faulty cropping pattern. With the advent of the Green
Revolution in the state in 1966-67, paddy, historically never the main crop of Punjab,
was adopted on a large scale. The area under it increased from 2.93 lakh hectares (LH) in
1966-67 to 31.49 LS in 2020, which was the highest-ever area under rice cultivation in
the history of Punjab. This is an almost 11-time increase in rice area in five decades.
 They sacrificed this precious, natural resource to paddy crops, which also hampers water
recharging because of the puddling method used to prepare fields for transplanting.
Puddling has created a thick hard layer on the agricultural fields, disturbing the
recharging system and wasting rainwater in evaporation or creating flash floods.
 The water availability at 3 to 10 meters, which needs a centrifugal pump to extract water,
is the most desirable, but currently, water is available at 20 to 30 meters, or more than
30 meters down, in around 84 percent of Punjab. This needs tubewell or submersible
pumps for extraction, and farmers are spending huge amounts to get deeper and deeper
wells, leading to financial liabilities.
So, option (d) is correct.

Q.64) Consider the following statements about the Global Air Quality Guidelines 2021, recently seen in
the news.
1. It was recently released by the World Health Organization.
2. It recommends air quality levels for 6 pollutants - Particulate Matter (PM2.5 and PM10), Ozone,
Nitrogen dioxide, Sulphur dioxide and Carbon monoxide.
3. The upper limit of annual PM2.5, that is 10 micrograms per cubic metre, has now been revised to
five micrograms per cubic metre.
Which of the statements given above is/are correct?
(a) 1 only
(b) 1 and 2 only
(c) 2 and 3 only
(d) 1, 2 and 3
EXPLANATION:
New WHO Global Air Quality Guidelines (AQGs) provide clear evidence of the damage air pollution
inflicts on human health, at even lower concentrations than previously understood. The guidelines
recommend new air quality levels to protect the health of populations, by reducing levels of key air
pollutants, some of which also contribute to climate change. So, Statement 1 is correct.
WHO’s new guidelines recommend air quality levels for 6 pollutants, where evidence has advanced
the most on health effects from exposure. When action is taken on these so-called classical
pollutants – particulate matter (PM₁₀ and PM₂.₅), ozone (O₃), nitrogen dioxide (NO₂) sulfur dioxide
(SO₂), and carbon monoxide (CO), it also has an impact on other damaging pollutants. So,
Statement 2 is correct.
The recommendation for PM 2.5 is that the annual average should not exceed 5 micrograms per
cubic metre or 15 micrograms per cubic metre in a day. The earlier limit was 10 micrograms per
cubic metre annually and 25 micrograms per cubic metre in a day. So, Statement 3 is correct.

ADDITIONAL INFORMATION:
AIR POLLUTION  Around 7 million premature deaths are attributable to the joint
AND HEALTH effects of ambient and household air pollution and of these, more
than 500 000 happen in the WHO European Region.
 Air pollution is now recognized as the single biggest environmental
threat to human health, along with climate change.
 Air pollution affects:
o NCDs,
o cardiovascular and respiratory diseases,
o lower respiratory tract infections,
o preterm birth,
o and other causes of death in children and infants.
AIR POLLUTION  More than 90% of the global population in 2019 lived in areas where
IN THE WORLD PM ₂.₅ concentrations exceeded 2005 annual WHO AQG of 10 µg/m³.
 Many of the countries with the lowest national PM₂. ₅ exposure levels
were in WHO European Region.
 The patterns of ambient NO 2 concentrations had the highest
population-weighted concentrations in eastern Asia, the Middle East,
North America, and much of Europe, reflecting emissions from mobile
sources propelled by combustion engines.
 Air pollution leads to health-related economic impacts through
human health costs and lost labor productivity.

Q.65) Consider the following statements about the National Board for Wildlife:
1. It is headed by the Prime Minister and the minister of environment, forest and climate change
(environment minister) as vice-chairperson.
2. No alteration of boundaries of the protected areas is possible without the approval of this board.
3. The Standing Committee of the board is tasked with regulating land diversion within protected
areas and eco-sensitive zones.
Which of the statements given above are correct?
(a) 1 and 2 only
(b) 2 and 3 only
(c) 1 and 3 only
(d) 1, 2 and 3
EXPLANATION:
The National Board for Wildlife (NBWL) is a “statutory board” constituted under the Wild
Life (Protection) Act, 1972 (wildlife act). However, it is important to point out that the
wildlife act, as originally enacted in 1972, did not provide for the NBWL. It was only
through an amendment of the wildlife act in 2002 that the NBWL was constituted. So,
Statement 1 is correct.
The WLPA mandates that without the approval/recommendation of the NBWL,
construction of tourist lodges, alteration of the boundaries of PAs, destruction or
diversion of wildlife habitat, and de-notification of Tiger Reserves, cannot be done. So,
Statement 2 is correct.
Most of the proposals that come up before the Standing Committee are for dams,
highways, mines, power lines, and other projects that destroy wildlife or involve the
diversion of habitat. It is therefore mandatory for the Standing Committee to adjudicate
every such proposal strictly in conformity with Section 29 or sub-section (6) of Section
35. Every decision must be backed by clear reason(s), in writing, as to how the project
or the decision to approve it ensures improvement and better management of wildlife
therein. So, Statement 3 is correct.

ADDITIONAL INFORMATION:
National Board for Wild Life
Statutory Body The National Board for Wildlife (NBWL) is constituted by the
Central Government under Section 5 A of the Wildlife (Protection)
Act, 1972 (WLPA).
Members It is a 47-member committee, headed by the Prime Minister and
the minister of environment, forest, and climate change
(environment minister) as vice-chairperson.
Protection The NBWL is the apex body for the conservation of wildlife and
its habitat, and the NBWL’s role is of critical importance to
ensure the long-term protection of India’s biodiversity.
Concern Other larger issues of concern point to the fact that the NBWL
has become a “clearinghouse” for projects, where, irrespective of
its impact on wildlife, projects are approved and that the
decisions of the board are guided more by economic, strategic,
political, and other considerations and rarely in terms of wildlife
conservation.

Q.66) Which one of the following statements about the Living Root Bridges of India is correct?
(a) It has been granted the tagline of UNESCO World Heritage Site
(b) These are aerial bridges built by weaving and manipulating the roots of Bamboo trees
(c) It is predominantly found in all the North-Eastern states where bamboo trees are found.
(d) Ficuselastica is conducive to the growth of these bridges because of its very nature.
EXPLANATION:
The Zoological Survey of India (ZSI) has underlined some green rules for the living root
bridges of Meghalaya to get the UNESCO World Heritage Site tag and it has not yet been
granted. So, Option (a) is not correct.
The jing kieng jri or living root bridges – aerial bridges built by weaving and
manipulating the roots of the Indian rubber tree – have been serving as connectors for
generations in Meghalaya. So, Option (b) is not correct.
It is predominantly found in the state of Meghalaya and it uses traditional tribal
knowledge to train roots of the Indian rubber tree. So, Option (c) is not correct.
Ficus elastica is conducive to the growth of bridges because of its very nature. There are
three main properties: they are elastic, the roots easily combine and the plants grow in
rough, rocky soils. So, Option (d) is correct.
ADDITIONAL INFORMATION:
Living Root Bridges of India

Root Bridge A root bridge uses traditional tribal knowledge to train roots of
the Indian rubber tree, found in abundance in the area, to grow
laterally across a stream bed, resulting in a living bridge of roots.
Jing Kieng Jri The jing kieng jri or living root bridges – aerial bridges built by
weaving and manipulating the roots of the Indian rubber tree –
have been serving as connectors for generations in Meghalaya.
Spanning between 15 and 250 feet and built over centuries, the
bridges, primarily a means to cross streams and rivers, have also
become world-famous tourist attractions.
Process The process begins with the placing of young pliable aerial roots
growing from Ficus elastica (India rubber) trees in hollowed-out
Areca catechu or native bamboo trunks. These provide essential
nutrition and protection from the Sweather and also perform as
aerial root guidance systems. Over time, as the aerial roots
increase in strength and thickness, the Areca catechu or native
bamboo trunks are no longer required.
Potential of New research investigates the living root bridges structures and
Root Bridge proposes to integrate them in modern architecture around the
world, and potentially help make cities more environment-
friendly.

Q.67) Consider the following statements about Mass Extinctions of the Earth.
1. Planet Earth has witnessed five mass extinctions in the past, with the last one about 65 million
years wiping out the dinosaurs.
2. The present mass extinction is referred as Holocene extinction or Anthropocene extinction as a
result of nature induced climate change reactions.
3. According to the IUCN Red List data, molluscs have suffered a higher rate of extinction than birds
and mammals.
Which of the statements given above is/are correct?
(a) 1 only
(b) 1 and 2 only
(c) 2 and 3 only
(d) 1 and 3 only
EXPLANATION:
Mass extinction is when the Earth loses more than three-quarters of its species in a
geologically short period. So far, during the entire history of the Earth, there have been five
mass extinctions. 66 million years ago, at the end of the Cretaceous–Paleogene period, the
fifth mass extinction resulted in the extinction of dinosaurs. So, Statement 1 is correct.
The Holocene extinction otherwise referred to as the sixth mass extinction or Anthropocene
extinction, is an ongoing extinction event of species during the present Holocene epoch as a
result of human activity. So, Statement 2 is not correct.
According to the IUCN Red List data molluscs (land snails and slugs), the second-largest
phylum in numbers of known species has suffered a higher rate of extinction than birds
and mammals. However, among the invertebrates, only less than 2 per cent have been fully
evaluated and many remain in the ‘Data Deficient category. So Statement 3 is correct.

ADDITIONAL INFORMATION:
PERIOD MASS EXTINCTION IN DETAIL
Ordovician  485 million years to 440 million years ago.
Period and First  Many species still prevalent today evolved, such as primitive
Mass Extinction fish, corals, etc.
 The beginning of terrestrial lifeforms.
 By the end of the Ordovician, Gondwana had moved from the
equator to the South Pole.
 The glaciation of Gondwana resulted in a major drop in sea
level, killing off all life along its coast.
 Glaciation caused a snowball Earth, leading to the Ordovician-
Silurian extinction (First Mass Extinction).
Devonian Period  415 million years to 360 million years ago.
and Second Mass  Also known as the Age of the Fish.
Extinction  On land, the first trees and seeds evolved.
 Late Devonian extinction (Second Mass Extinction) The
extinction seems to have only affected marine life and no clear
cause of extinction.
Permian–Triassic  300 million to 250 million years ago.
Period and Third  At its beginning, all continents came together to form the super-
Mass Extinction continent Pangaea, surrounded by one ocean called
or Great Dying Panthalassa.
Event  It is the Earth’s most severe known extinction event, with up to
96% of all marine species and 70% of terrestrial vertebrate
species becoming extinct.
 It is the only known mass extinction of insects.
 Large meteor impact events, massive volcanism, runaway
greenhouse effect triggered by the sudden release of methane
from the seafloor due to methane-producing microbes known as
methanogens.
Triassic–Jurassic  Began 201 million years ago.
Period and  Occurred just before Pangaea started to break apart.
Fourth Mass  This event vacated terrestrial ecological niches, allowing the
Extinction dinosaurs to assume the dominant role.

 Gradual climate change, sea-level fluctuations, oceanic


acidification reached a tipping point.
 Massive volcanic eruptions might have caused intense global
warming (release of carbon dioxide or sulphur dioxide) or
intense global warming (release of aerosols).

Cretaceous–  Began 66 million years ago.


Paleogene period  At the end of the Cretaceous, the Deccan Traps and other
and Fifth Mass volcanic eruptions were poisoning the atmosphere.
Extinction or K-  A large meteor smashed into Earth, creating the Chicxulub
T Extinction Crater (Yucatan Peninsula Mexico) creating the event known as
the K-T Extinction.
 The age of the dinosaurs came to an end.
 Mammals in particular diversified in the Paleogene, evolving
new forms such as horses, whales, bats, and primates.
 Birds, fish, and perhaps lizards also radiated.
Holocene period  Began 11,700 years ago and lasts until the present day.
and Sixth Mass  All recorded history and “the history of the world” lie within the
Extinction boundaries of the Holocene epoch.
 Human activity is blamed for a mass extinction that began
roughly 10,000 years ago, though the species becoming extinct
have only been recorded since the Industrial Revolution.
 More than 322 species have become extinct due to human
activity since the Industrial Revolution.

Q.68) In the polar regions, why the very small-sized animals are rarely found?
1. They tend to lose body heat very fast when it is cold outside.
2. They have to expend much energy to generate body heat through metabolism.
3. There will be no sufficient foods for both herbivores and carnivores’ animals.
Select the correct answer using the code given below.
(a) 1 only
(b) 1 and 2 only
(c) 2 and 3 only
(d) 1, 2 and 3
EXPLANATION:
Small animals have a larger surface area relative to their volume, and they tend to lose
body heat fast when it is cold outside. They have to expend much energy to generate
body heat through metabolism. So, small animals are rarely found in polar regions. So,
Statements 1 and 2 are correct.
In polar regions, for both herbivores and carnivores’ animals, there will be available of
sufficient food eg: Fish and algae available all round the year and consumed by seabirds
and large animals like seal and polar bear. However, the main reason why very small
sized animals are rarely found in polar region is thermoregulation of heat loss with
respect to small animals large surface area. So, Statement 3 is not correct.

Q.69) Consider the following statements about the process of Homeostasis.


1. Homeostasis is the way animals maintain a stable internal balance in their body.
2. Most mammals use this process to regulate their body temperature similar to the ones that
humans use.
3. In this process, Plants are shedding their leaves to maintain their energy loss.
Which of the statements given above is/are correct?
(a) 1 only
(b) 1 and 2 only
(c) 2 and 3 only
(d) 1, 2 and 3
EXPLANATION:
Homeostasis is any self-regulating process by which an organism tends to maintain
stability while adjusting to conditions that are best for its survival. If homeostasis is
successful, life continues; if it's unsuccessful, it results in a disaster or death of the
organism. So, Statement 1 is correct.
Some organisms can maintain homeostasis by physiological (sometimes behavioral also)
means which ensures constant body temperature, constant osmotic concentration, etc.
All birds and mammals and a very few lower vertebrate and invertebrate species are
indeed capable of such regulation (thermoregulation and osmoregulation). The
mechanisms used by most mammals to regulate their body temperature are similar to
the ones that we humans use. Plants, on the other hand, do not have such mechanisms
to maintain internal temperatures. So, Statement 2 is correct and Statement 3 is
not correct.

Q.70) Consider the following statements about the carbon cycle.


1. Carbon constitutes 49 per cent of the dry weight of organisms and is next only to water.
2. 71 per cent carbon is found dissolved in the land surface which regulates the amount of carbon
dioxide in the atmosphere.
3. Carbon cycling occurs through the atmosphere, ocean and through living and dead organisms.
4. Decomposers also contribute substantially to the CO2 pool.
Which of the statements given above are correct?
(a) 1, 2 and 3 only
(b) 1, 2 and 4 only
(c) 1, 3 and 4 only
(d) 2, 3 and 4 only
EXPLANATION:
 When we study the composition of living organisms, carbon constitutes 49 per cent
of dry weight of organisms and is next only to water.
 If we look at the total quantity of global carbon, we find that 71 per cent of carbon is
found dissolved in oceans. This oceanic reservoir regulates the amount of carbon
dioxide in the atmosphere.
 Fossil fuel also represents a reservoir of carbon. Carbon cycling occurs through
atmosphere, ocean and through living and dead organisms. According to one
estimate, 4×1013 kg of carbon is fixed annually in the biosphere through
photosynthesis. A considerable amount of carbon returns to the atmosphere as CO2
through the respiratory activities of the producers and consumers.
 Decomposers also contribute substantially to the CO2 pool by their processing of
waste materials and dead organic matter of land or oceans. Some amount of the
fixed carbon is lost to sediments and removed from circulation. Burning of wood,
forest fire and combustion of organic matter, fossil fuel, volcanic activity are
additional sources for releasing CO2 into the atmosphere.
So, option (c) is correct.

ADDITIONAL INFORMATION:

Q.71) Consider the following statements:


1. The relationship and interaction between organisms and abiotic are highly complex.
2. Both biotic and abiotic factors of the environment are in flux and keep changing continuously.
Which of the statements given above is/are correct?
(a) 1 only
(b) 2 only
(c) Both 1 and 2
(d) Neither 1 nor 2
EXPLANATION:
The environment is not static. Both biotic and abiotic factors of the environment are in
flux and keep changing continuously. For example, consider the environment of a fish
in the pond. Its external environment consists of abiotic components such as light,
temperature etc., and biotic components such as aquatic plants, animals etc. Its
internal environment is enclosed by the outer body surface. So, Statement 2 is
correct.
ADDITIONAL INFORMATION:
Environment
Environment Every living organism is constantly interacting with its
environment comprised of air, light, water, land or substratum
and the various kinds of living organisms. The environment may
be defined as the surroundings or conditions in which an
organism lives or operates. The environment broadly includes
living and non-living components which are listed in the table
given below.
Components of Abiotic Biotic
the environment Light, climate (humidity and Living organisms include
temperature) atmospheric plants, animals,
gases, water, substrata (soil, microorganisms (bacteria,
river/sea bed). fungi, protozoa), and human
beings.

Q.72) The Genetic engineering appraisal committee (GEAC)


1. is the apex body for approval of activities involving large scale use of hazardous microorganisms
and recombinants.
2. will appraise proposals relating to the release of genetically engineered organisms and products
into the environment including experimental field trials.
3. has been established under the Food Safety and Standards Act, 2006.
4. is established under the Union Ministry of Environment, Forests and Climate Change.
Select the correct answer using the code given below.
(a) 1, 2 and 3 only
(b) 1, 2 and 4 only
(c) 1, 3 and 4 only
(d) 2, 3 and 4 only
EXPLANATION:
GEAC is chaired by the Special Secretary/Additional Secretary of MoEF & CC. Presently, it
has 24 members and meets every month to review the applications and appraisal of
activities involving large scale use of hazardous microorganisms and recombinants in
research and industrial production from the environmental angle. So, Statement 1 is
correct.
GEAC is also responsible for the appraisal of proposals relating to the release of genetically
engineered (GE) organisms and products into the environment including experimental field
trials. Presently, Bt cotton is the only commercially approved GM crop in India for
cultivation. However, confined field trials have been allowed for at least 20 GM crops. It
includes Bt brinjal and Bt mustard, which haven’t been approved yet. So, Statement 2 is
correct.
The Genetic Engineering Appraisal Committee (GEAC) was established under the
Environmental Protection Act, 1989 and functions under the Ministry of Environment,
Forest and Climate Change (MoEF&CC). So, Statement 3 is not correct and statement 4
is correct.
ADDITIONAL INFORMATION:
Why in news?  To boost domestic supply and arrest skyrocketing prices, the
government had in August 2021 permitted the import of 12 lakh tonnes
of GM soya meal after the environment ministry (GEAC) and Director
General of Foreign Trade (DGFT) cleared the air on the GM issue.
 This would be the first time that India would import GM soya meals in
view of the demand by the poultry industry.
GM issues in  It is important to notice that the Gm Soyameal falls under the non-living
India organism category, while India has only approved the Bt cotton crop as
under the living modified organism category.
 However, confined field trials have been allowed for at least 20 GM
crops. It includes Bt brinjal and Bt mustard, which haven’t been
approved yet.

The Genetic  Genetic Engineering Appraisal Committee (GEAC) is the apex body that
Engineering allows for the commercial release of GM crops.
Appraisal  It is established under the Environmental Protection Act, 1989 and
Committee functions under the Ministry of Environment, Forest and Climate
(GEAC) Change (MoEF&CC).
Members and  GEAC is chaired by the Special Secretary/Additional Secretary of MoEF
Functions of & CC and co-chaired by a representative from the Department of
GEAC Biotechnology (DBT).
 Presently, it has 24 members and meets every month to review the
applications and appraisal of activities involving large scale use of
hazardous microorganisms and recombinants in research and industrial
production from the environmental angle.
 It is also responsible for the appraisal of proposals relating to the
release of genetically engineered (GE) organisms and products into the
environment including experimental field trials.
Offence and  Use/Cultivation of the unapproved GM variant can attract a jail term of
Punishment 5 years and a fine of Rs.1 lakh under the Environmental Protection Act,
1989.
Q.73) In the context of population growth in ecology, which one of the following situations clearly
explains that the kind of interaction benefits only one species and the other species is neither
benefitted nor harmed?
(a) Both parasitism and predation
(b) Both Mutualism and competition
(c) Both commensalism and amensalism
(d) Only in commensalism
EXPLANATION:
The interaction where one species is benefitted and the other is neither benefitted nor
harmed is called commensalism. Examples of commensals include bird species, such as the
great egret (Ardea alba), that feed on insects turned up by grazing mammals or
on soil organisms stirred up by plowing. Various biting lice, fleas, and louse flies are
commensals in that they feed harmlessly on the feathers of birds and sloughed-off flakes
of skin from mammals. So, option (d) is correct.

ADDITIONAL INFORMATION:
 Both the species benefit in mutualism and both lose in competition in their
interactions with each other.
 In both parasitism and predation, only one species benefits (parasite and predator,
respectively) and the interaction is detrimental to the other species (host and prey,
respectively).
 The interaction where one species is benefitted and the other is neither benefitted
nor harmed is called commensalism.
 In amensalism on the other hand one species is harmed whereas the other is
unaffected. Predation, parasitism, and commensalism share a common
characteristic– the interacting species live closely together.
Q.74) Consider the following statements about the Net-Zero emissions and Carbon Neutral:
1. Carbon neutral refers to a policy of not increasing carbon emissions and of achieving carbon
reduction through offsets.
2. While net-zero carbon means making changes to reduce carbon emissions to the lowest amount
and offsetting as a last resort.
3. In both cases, carbon offsetting removes CO2 from the environment.
4. Both are the global mandates for the developed countries mentioned under the Paris Climate
Agreement.
Which of the statements given above are correct?
(a) 1, 2 and 3 only
(b) 1, 2 and 4 only
(c) 1, 3 and 4 only
(d) 2, 3 and 4 only

Q.75) Consider the following statements about the Border Security Force (BSF).
1. It is a Central Armed Police Force that functions under the Union Ministry of Home Affairs.
2. It is deployed in all the international borders of the country.
3. It is also deployed in areas affected by Left Wing Extremism (LWE).
4. It is routinely deployed for election and other law and order duties at the request of State
governments.
Which of the statements given above are correct?
(a) 1, 2 and 3 only
(b) 1, 2 and 4 only
(c) 1, 3 and 4 only
(d) 2, 3 and 4 only
EXPLANATION:
BSF is one of the five Central Armed Police Forces of the Union of India and is under the
administrative control of the Ministry of Home Affairs. It is known as the First Line of
Defence of Indian Territories. So, Statement 1 is correct.
It is deployed at various integrated check-posts of borders with Pakistan and Bangladesh,
but not all the international borders of India. Other paramilitary forces like Indo-Tibetian
Border Police is tasked with guarding the Indo-Tibetan border and the mountainous regions
of the India-China border. The Sashastra Seema Bal guards Indo-Nepal and Indo-Bhutan
Borders. So, Statement 2 is not correct.
The counter-insurgency operations in left-wing extreme areas are primarily handled by
Central Reserve Police Force. However, Some units of BSF are also deployed in Central
India to combat Naxal violence. Also, BSF and other Central Armed Police Forces are
routinely deployed for election and other law and order duties at the request of State
governments. So, Statements 3 and 4 are correct.

ADDITIONAL INFORMATION:
Why in  The recent notification of the Ministry of Home Affairs extending the
News? jurisdiction of BSF along the international borders in three states
has led to a fresh round of controversy involving the Centre and
states.
Border  Border Security Force (BSF) is a specialized centrally controlled
Security force to man the International Border with Pakistan.
Force  It is one of the five Central Armed Police Forces of the Union of
India and is under the administrative control of the Ministry of
Home Affairs.
 Based on the recommendations of the Committee of Secretaries in
the wake of the 1965 War, BSF came into existence on 01st Dec
1965.
 BSF has been termed as the First Line of Defence of Indian
Territories.
 BSF is deployed at various integrated check-posts of borders with
Pakistan and Bangladesh and is handling the security issues in the
famous Kartarpur Corridor.
Powers of  The violations against which the BSF carries out search and seizure
BSF include smuggling of narcotics, prohibited items, illegal entry of
foreigners and offences punishable under any other Central Act.
 The MHA has given powers to the BSF personnel in border areas
under the Customs Act, the Passport Act, the Narcotics Drugs and
Psychotropic Substances Act, the Code of Criminal Procedure
(CrPC), Registration of Foreigners Act, 1939, the Central Excises
and Salt Act, 1944, the Foreigners Act, 1946, and the Foreign
Exchange Regulation Act, 1947.
 The BSF does not have police powers but after apprehending a
suspect it can only conduct “preliminary questioning” and has to
hand over a seized consignment or the suspect to the local police
within 24 hours.
 It does not have the power to prosecute crime suspects.
 Police is a State subject under the Constitution.
Issues  Under the latest notification issued on October 11, 2021, the
Contented jurisdiction has been reduced from 80 km to 50 km in Gujarat.
Between  Also, in Assam, West Bengal and Punjab, the BSF jurisdiction has
Center and been extended from 15 km to 50 km which is seen as an assault on
States federalism.
 However, BSF informs that the recent amendment “establishes
uniformity in defining the area within which the BSF can operate”
and also improves its operational effectiveness in curbing trans-
border crimes.

Q.76) Consider the following statements about the Polyhouse technology, recently seen in the news.
1. It is used as a covering material under which crops can be grown in partially or fully controlled
climatic conditions.
2. It is covered with transparent material to permit the entry of natural light.
3. These are also helpful in reducing threats such as extreme heat and pest attacks in crops.
4. It is predominantly used for the cultivation of paddy and wheat.
Which of the statements given above are correct?
(a) 1, 2 and 3 only
(b) 1, 2 and 4 only
(c) 1, 3 and 4 only
(d) 2, 3 and 4 only
EXPLANATION:
A poly house is a specially constructed structure like a building where a specialised
polythene sheet is used as a covering material under which crops can be grown in partially
or fully controlled climatic conditions. So, Statement 1 is correct.
With rapidly rising temperatures due to mounting greenhouse gases in the atmosphere
from human activities, crops are increasingly facing threats such as extreme heat and pest
attacks. So, it is covered with transparent material to permit the entry of natural light while
protecting from heat and pests. So, Statements 2 and 3 are correct.
The crops that can be grown in poly house are floriculture crops, exotic Vegetables, and
nursery plants. So, Statement 4 is not correct.

ADDITIONAL INFORMATION:
Why in News?  Coffee-growing regions in Karnataka and Kerala are receiving
continuous rains which affect the harvest of ripened beans.
 It also led to the outbreak of fungal disease wherein the ripened
beans rot on the branches.
 Thus, Growers are raising demands to the Government to extend
the subsidy provided for setting up poly houses to coffee growers
also, who can utilise the infrastructure for drying the beans.
Polyhouse  A poly house is a specially constructed structure like a building
Technology where a specialised polythene sheet is used as a covering material
under which crops can be grown in partially or fully controlled
climatic conditions.
 It is covered with a transparent material as to permit the entry of
natural light.

Types of  Glass House


Polyhouse  Shade Net
Technology  Polythene Sheet House
Advantages  Protecting from extreme heat and rain or cold conditions.
 Preventing pest attacks and infections.
 Controlled manner of fertiliser and irrigation techniques available.
 Best suited for vegetable and ornamental crops.
Disadvantages  Wear and tear of polythene sheets and making crops vulnerable
during heavy storms.
 Construction and maintenance costs are high at the initial stages.
 Cladding of fertilisers and threat to labours on mishandling.
 Conventional poly houses have a stationary roof which sometimes
leads to excessive heat, insufficient light and are prone to
insufficient levels of carbon dioxide, transpiration and water
stress.

Q.77) The term ‘CaTRAT’ is sometimes seen in the news in the context of
(a) Monitoring of Tigers
(b) Captive breeding of Wild animals
(c) Security of National Highways
(d) Large scale mapping and Terrain modelling
EXPLANATION:
The 2018 tiger census was undertaken using the best available technology and
analytical tools such as The M-STrIPES app, CaTRAT software and the Extract Compare
programs. CaTRAT is an artificial intelligence software called Camera Trap data
Repository and Analysis Tool which uses a neural network and image recognition, to
classify wildlife into individual species while Extract and Compare allowed to
'fingerprint' tigers and identify individual tigers by their stripe patterns by comparing
images in the database. So, Option (a) is correct.
ADDITIONAL INFORMATION:

Q.78) Consider the following statements about Ecology:


1. A community is a group of organisms usually of the samespecies, occupying a defined area during
a specific time.
2. A community iseither fixed or rigid.
3. Tropical evergreen forest in North-Eastern part of India is an example of a Major Community.
Which of the statements given above is/are correct?
(a) 1 and 2 only
(b) 2 and 3 only
(c) 3 only
(d) 1, 2 and 3
EXPLANATION:
A population is a group of organisms usually of the same species, occupying a
defined area during a specific time.
A community is a group of organisms consisting of several different species that live
in an area and interact with each other. So, Statement 1 is not correct.
Members of a community also actively interact with their environment. In a
community, only those plants and animals survive which are adapted to a
particular environment. The climate determines the type of environment, hence, the
type of organisms in a community. Therefore a community is not fixed or rigid;
communities may be large or small. So, Statement 2 is not correct.
Major communities are large-sized, well organized and relatively independent. They
depend only on the sun’s energy from outside and are independent of the inputs
and outputs from adjacent communities. For example a pond, a forest, grassland or
lake. So, Tropical evergreen forest in North-East India is which is an independent
community is also an example of a Major Community. So, Statement 3 is correct.

ADDITIONAL INFORMATION:
Ecology
Ecology Ecology may be defined as the scientific study of the relationship
of living organisms with each other and with their environment.
The emphasis is on relationships between organisms and the
components of the environment namely abiotic (non-living) and
biotic (living).
Levels of Ecology not only deals with the study of the relationship of
Ecological individual organisms with their environment, but also with the
Organization study of populations, communities, ecosystems, biomes, and the
biosphere as a whole.

Individual An organism is an individual living being that can act or


function independently. It may be a plant, animal, bacterium,
fungi, etc.
Population Population’ is defined as a group of freely interbreeding
individuals of the same species present in a specific area at a
given time. For example, when we say that the population of a
city is 50,000, we mean that there are 50,000 humans in that
city.
Community In ecology, the term community, or more appropriately ‘biotic
community, refers to the populations of different kinds of
organisms living together and sharing the same habitat.
Ecosystem An ecosystem is defined as a structural and functional unit of
the biosphere consisting of a community of living beings and the
physical environment, both interacting and exchanging materials
between them.
Biome A large community unit, characterized by a major vegetation
type and associated fauna, found in a specific climatic region is
a biome Biomes refer basically to terrestrial areas. The aquatic
systems like the seas, rivers etc. are also divided into distinct life
zones on basis of salinity.
Biosphere The biosphere is made up of the parts of Earth where life exists.
The biosphere extends from the deepest root systems of trees to
the dark environment of ocean trenches, to lush rain forests and
high mountaintops.

Q.79) Consider the following statements:


1. A single species might show low diversity at the genetic level over its distributional range.
2. Communities with more species, generally, tend to be more stable than those with fewer species.
3. In terms of species diversity, the Western Ghats have a greater amphibian species diversity than
the Eastern Ghats.
4. The species diversity decreases as we move away from the equator towards the poles.
Which of the statements given above are correct?
(a) 1, 2 and 3 only
(b) 1, 2, 3 and 4 only
(c) 1, 3 and 4 only
(d) 2, 3 and 4 only
EXPLANATION:
A single species might show high diversity at the genetic level over its distributional range.
The genetic variation shown by the medicinal plant Rauwolfia vomitoria growing in different
Himalayan ranges might be in terms of the potency and concentration of the active
chemical (reserpine) that the plant produces. India has more than 50,000 genetically
different strains of rice and 1,000 varieties of mango. So, Statement 1 is not correct.
For many decades, ecologists believed that communities with more species, generally, tend
to be more stable than those with less species. A stable community should not show too
much variation in productivity from year to year; it must be either resistant or resilient to
occasional disturbances (natural or man-made), and it must also be resistant to invasions
by alien species. So, Statement 2 is correct.
The diversity at the species level, for example, the Western Ghats have a greater amphibian
species diversity than the Eastern Ghats. At the ecosystem level, India, for instance, with its
deserts, rain forests, mangroves, coral reefs, wetlands, estuaries, and alpine meadows has a
greater ecosystem diversity than a Scandinavian country like Norway. So, Statement 3 is
correct.
The diversity of plants and animals is not uniform throughout the world but shows a rather
uneven distribution. For many groups of animals or plants, there are interesting patterns in
diversity, the most well-known being the latitudinal gradient in diversity. In general, species
diversity decreases as we move away from the equator towards the poles. So, Statement 4
is correct.

ADDITIONAL INFORMATION:
BIODIVERSITY  Biodiversity conservation may be in situ as well as ex-situ.
CONSERVATION  In in situ conservation, the endangered species are protected in
their natural habitat so that the entire ecosystem is protected.
Recently, 34 ‘biodiversity hotspots in the world have been
proposed for intensive conservation efforts.
 Of these, three (Western Ghats-Sri Lanka, Himalaya, and Indo-
Burma) cover India’s rich biodiversity regions.
 Our country’s in situ conservation efforts are reflected in its 14
biosphere reserves, 90 national parks, > 450 wildlife sanctuaries,
and many sacred groves.
 Ex-situ conservation methods include protective maintenance of
threatened species in zoological parks and botanical gardens, in
vitro fertilization, tissue culture propagation, and
cryopreservation of gametes.

Q.80) Which of the following are the benefits of Agro-biodiversity?


1. Increase in productivity
2. Reduction in deforestation
3. Conservation of soil
4. Increase in nutritional values
5. Reduction in the use of agrochemicals
Select the correct answer using the code given below.
(a) 1, 3 and 5 only
(b) 1 and 3 only
(c) 2, 4 and 5 only
(d) 1, 2, 3, 4 and 5

EXPLANATION:
Knowledge of biodiversity and its conservation has multiple proven benefits for
agricultural productivity such as
 Increase productivity, yields and food security (and consequently economic
returns)
 Reduce the pressure of agriculture on fragile areas, forests and endangered
species (therefore restricting deforestation)
 Build stability, robustness, and sustainability of farming systems (restricting
desertification)
 Contribute to sound pest and disease management and sustainable
intensification (limiting the use of agrochemicals)
 Conserve soil and increase natural soil fertility and health (restricting soil
degradation)
 Diversify products and income opportunities from farms
 Reduce the spread of risks to individuals, communities, and nations
 Help maximise effective use of the resources and environment (restore
ecological health)
 Reduce dependency on external inputs
 Increase nutritional values, and provide sources of medicine and vitamins
So, Option (d) is correct.

ADDITIONAL INFORMATION:
Agro-Biodiversity
Biodiversity in Biodiversity has been fundamental to agriculture and food
Agriculture provisions for centuries. It has provided farming systems with
the capacity to evolve over the last 12,000 years. Food
production depends on a variety of managed agro-ecosystems
that benefit from natural resources both on farms and in
surrounding habitats such as forest, grassland and aquatic
ecosystems.
Agro-Biodiversity It encompasses- genetic resources that are essential living
material of plants and animals; edible plants/ crops, including
landraces varieties and hybrids; livestock and edible fishes/
aquatic organisms; soil organisms vital for soil fertility and
structure; naturally occurring insects, bacteria and fungi
which serve as predators of pests and diseases;
agroecosystems like polyculture/ monoculture, irrigated/
rainfed, small/ large that are valuable for nutrient cycle
stability and productivity; and the wild relatives of crop plants.
Species Diversity There are approximately 75,000 species of edible plants
worldwide; but over the course of human civilisation, only
about 7000 plant varieties have been used for food. Of these,
3000 domesticated species are being predominantly used to
provide various provisions required for human survival.
Major There is also remarkable diversity and abundance of insects,
Contributor fungi, and other organisms that are valuable to the
productivity of the agro-ecosystem. Arthropods are the most
abundant class of animals and contribute in a major way to
biomass and agro-ecosystem balance.
Medicinal Value Many rural communities, particularly tribals obtain their daily
food from several wild and non-conventional foods producing
species. In addition, 95% of prescriptions of traditional
systems of medicines are plant-based.

Q.81) Consider the following statements about the Hypersonic missiles.


1. Hypersonic vehicles travel at speeds that are 5 or more times the speed of sound.
2. Hypersonic systems primarily use the scramjet technology that needs to be able to handle high
temperatures.
3. Hypersonic vehicles are difficult to track because they don’t follow the fixed parabolic trajectory.
4. Only USA and China and not India possess this technology in the World.
Which of the statements given above are correct?
(a) 1, 2 and 3 only
(b) 1, 2 and 4 only
(c) 1, 3 and 4 only
(d) 2, 3 and 4 only
EXPLANATION:
A hypersonic missile travels at speeds of Mach 5 and higher – five times faster than the
speed of sound (3836 mph), which is around 1 mile per second. So, Statement 1 is
correct.
Most of the hypersonic vehicles primarily use scramjet technology which is a type of air-
breathing propulsion system, a complex technology used to handle high temperatures,
making the hypersonic systems very costly. So, Statement 2 is correct.
They can also maneuver in mid-flight, so these vehicles are difficult to track and intercept
because, unlike ballistic missiles, they don’t follow the fixed parabolic trajectory. So,
Statement 3 is correct.
The independent Congressional Research Service (CRS), in the latest report, said that
although the U.S., Russia, and China possess the most advanced hypersonic weapons
programs, several other countries, including Australia, India, France, Germany, and Japan,
are also developing hypersonic weapons technology. So, Statement 4 is not correct

ADDITIONAL INFORMATION:
HYPERSONIC TECHNOLOGY
TYPES There are two types of hypersonic missiles such as,
 Hypersonic cruise missile – these are the one that uses the jet
or rocket propellant through their flight and is considered the
fastest among all the cruise missiles.
 Hypersonic glider vehicle – these missiles first go up to an
atmosphere on their conventional rocket before being launched
towards their target.
SIGNIFICANCE  It will provide a nation to have better-striking capabilities.
 It can overcome any heavily defended target such as an aircraft
carrier.
DEVELOPMENTS  India has recently testfired the hypersonic test demonstrator
IN INDIA vehicle which is an unmanned demonstration scramjet vehicle
for hypersonic speed flight.
 During the tests, hypersonic air-breathing scramjet technology
was demonstrated.
 According to sources, India has developed its cryogenic engine
and has demonstrated it in a 23-second flight. There are plans
to make a hypersonic cruise missile using HSTDV.
 Hypersonic weapons systems with medium-to-long-range
capabilities are also expected to be available in the next four
years.

Q.82) Which one of the following is the northern most territory in the Ladakh region in India?
(a) Depsang Plains
(b) Karakoram Pass
(c) Galwan valley
(d) Pangong Tso Lake
EXPLANATION :

From the above map, arranging the given options from North to South,
1. Karakoram Pass
2. Depsang Plains
3. Galwan Valley
4. Pangong Tso lake

So, Option (b) is correct.


ADDITIONAL INFORMATION :
India China Standoff :
Context  Violent clashes took place between Indian and Chinese soldiers at the
Galwan valley in Ladakh along the Line of Actual Control (LAC), with
reported casualties on both sides.


 Until 1960, China used to have a company stationed at Kongka La and
Hot Springs which they increased to a regiment in the Hot Springs area
by 1962.
 In 1962, China attacked India’s position in Hot Springs and Galwan
Valley.
 In June 2020, 20 Indian and at least four Chinese soldiers died in
Galwan valley clashes.
 This is the first time after the 1962 War that soldiers have died in clashes
on the India-China border in Ladakh.
 Hot Springs is the last of the friction points that the Army came up last
year when China moved its troops across the LAC.
 Disengagement has been completed in other places like PP14 in Galwan
Valley, PP17A in Gogra Post, and the north and south banks of Pangong
Tso.
 Talks between senior military commanders of India and China to find a
solution to the 17-month long Ladakh standoff have run into rough
weather.
 The 13th round of talks held on the Chinese side of Line of Actual Control
(LAC) on Sunday were aimed at discussing disengagement of troops at
Hot Springs area of eastern Ladakh.
 However, the two sides blamed each other for the deadlock.
 India pointed out that the situation has been caused by unilateral
attempts of Chinese side to alter the status quo in violation of bilateral
agreements.
 New Delhi says that the Chinese delegation made no constructive
proposals while Beijing says that the Indian side made unreasonable
demands.

Q.83) Consider the following statements about the Commission for Air Quality Management in National
Capital Region and Adjoining Areas
1. It is a statutory body enacted by an act of Parliament.
2. The rulings by the commission on air pollution will override anything contained in any other law.
3. Orders of the commission can only be contested before the Supreme Court of India.
Which of the statements given above is/are correct?
(a) 1 and 2 only
(b) 1 and 3 only
(c) 2 only
(d) 1, 2 and 3
EXPLANATION:
Commission for Air Quality Management in National Capital Region and Adjoining
Areas was created by enacting “The Commission For Air Quality Management In
National Capital Region And Adjoining Areas Act, 2021”. So, Statement 1 is
correct.
The Commission is the most powerful air pollution monitoring body set up by the
Centre to date. The rulings by the commission on air pollution will override
anything contained in any other law. The powers of the commission will also
supersede that of any other body in matters of air pollution. Therefore, in cases
where conflict may arise between orders or directions issued by the other state
governments, state pollution control boards or even the Central Pollution Control
Board, the orders of the commission will prevail. So, Statement 2 is correct.
The Act further says that no civil court will have jurisdiction to entertain any suit,
proceeding or dispute about or arising out of the actions taken or directions issued
by the commission and that orders of the commission can only be contested before
the National Green Tribunal. So, Statement 3 is not correct.

ADDITIONAL INFORMATION:
Commission for Air Quality Management in National Capital Region and Adjoining Areas

Objective To create an overarching body to consolidate all monitoring


bodies, and to bring them on one platform so that air quality
management can be carried out in a more comprehensive,
efficient, and time-bound manner.
Replacing EPCA Apart from consolidating all agencies that monitored, investigated
and planned mitigation of air pollution in the region, the
Commission has replaced the Supreme Court-appointed
Environment Pollution (Prevention and Control) Authority (EPCA)
which had been running for 22 years. Over the years, the EPCA’s
powers had been waning. While dissolving the body, the Centre
felt that the EPCA had become redundant and had been
ineffective in addressing issues related to air pollution. The EPCA
also did not have penal provisions that the commission will now
have.
Powers  The Commission is the most powerful air pollution
monitoring body set up by the Centre to date. The rulings
by the commission on air pollution will override anything
contained in any other law.
 The Commission will have the power to take measures,
issue directions and entertain complaints “to protect and
improve the quality of air in the National Capital Region”.
 It will also coordinate action taken by states on air
pollution and will lay down parameters for air quality and
emission or discharge of environmental pollutants.
 It will also have powers to restrict industries in any area,
carry out random inspections of any premises including
factories and be able to close down an industry or cut its
power and water supply in case of non-compliance.
 It will also be monitoring the measures taken by the states
to prevent stubble burning.
Composition The Commission will be headed by a full-time chairperson and 20
other members. The commission will have at least three sub-
committees – monitoring and identification, safeguarding and
enforcement, and research and development.
Way of Appeal The Act further says that no civil court will have jurisdiction to
entertain any suit, proceeding or dispute about or arising out of
the actions taken or directions issued by the commission and that
orders of the commission can only be contested before the
National Green Tribunal.

Q.84) Consider the following statements about the Biodiversity Act of 2002:
1. The benefit-sharing guidelines are to be issued by the State Biodiversity Authority.
2. All matters relating to access by Indians for commercial purposes will be under the purview of the
National Biodiversity Authority.
3. There is no requirement under the legislation for seeking permission for carrying out research if it
is carried out in India by Indians.
Which of the statements given above is/are correct?
(a) 1 and 2 only
(b) 1 and 3 only
(c) 3 only
(d) 1, 2 and 3
EXPLANATION:
National Biodiversity Authority (NBA) is mandated to advise the central government,
regulate activities and issue guidelines for access to biological resources and fair and
equitable benefit sharing in accordance with the Biological Diversity Act, 2002. So,
Statement 1 is not correct.
All matters relating to access by Indians for commercial purposes under the purview of the
concerned State Biodiversity Boards, While all matters relating to requests by foreign
individuals, companies or institutions and the transfer of results of research to any
foreigner, approvals for intellectual property protection where biological resources and
associated knowledge are involved will be dealt with by NBA. So, Statement 2 is not
correct.
There is no requirement under the legislation for seeking permission for researching if it is
carried out in India by Indians, as well as under collaborative research projects that have
been drawn within the overall policy guidelines formulated by the Central Government. So,
Statement 3 is correct.

ADDITIONAL INFORMATION:
Convention on  The Convention on Biological Diversity (CBD) is a legally binding
Biological multilateral environmental agreement.
Diversity  It has 194 contracting Parties (Countries) as its members with
three objectives – conservation of biological diversity, sustainable
use of the diversity and ensuring fair and equitable sharing of
benefits of such use.
 It has entered into force on 29th December 1993.
Status of  India is one of the 17-mega biodiversity countries of the world.
India’s  With only 2.4% of the land area, India already accounts for 7-8%
Biodiversity of the recorded species of the world.
 Over 46,000 species of plants and 81,000 species of animals
have been recorded in the country so far by the Botanical Survey
of India and the Zoological Survey of India, respectively.
The  Primarily addresses issues of conservation, sustainable use of
Biodiversity Act biological resources in the country, issues related to access to
- 2002 genetic resources and associated knowledge and fair and
equitable sharing of benefits arising from the utilization of
biological resources to the country and its people.
 The Act also covers the protection of traditional knowledge and
equitable sharing of benefits arising out of the use of such
knowledge.
 A three-tiered structure has been established under the Act at
the national, state and local levels.
 All of these institutions are statutory, autonomous and
decentralised bodies established under the Biological Diversity
Act, 2002.
Three-Tiered  The National Biodiversity Authority: It is mandated to regulate
Structure the use of India’s biological resources; facilitates/ enable
conservation action and provides advice to Central and State
Governments on issues of conservation, sustainable use and
access and benefit-sharing.
 The NBA has its Headquarters in Chennai, Tamil Nadu, India.
 The State Biodiversity Authority: It was established by the State
Governments in accordance with Section 22 of the Act.
 All matters relating to access by Indians for commercial
purposes will be under the purview of the concerned State
Biodiversity Boards. However, the benefit-sharing guidelines are
to be issued by the NBA.
 Biodiversity Management Committees (BMCs): According to
Section 41 of the Biodiversity Act – 2002 Act, every local body
shall constitute the BMC within its area to promote
conservation, sustainable use and documentation of biological
diversity.
 The main function of the BMC is to prepare the People’s
Biodiversity Register in consultation with the local people.
Biodiversity  Under the Biological Diversity Act, 2002 the State Government in
Heritage Sites consultation with local bodies may notify the areas of
(BHS) biodiversity importance as Biodiversity Heritage Sites.
 The Biodiversity Heritage Sites are the well-defined areas that
are unique, ecologically fragile ecosystems - terrestrial, coastal
and inland waters and, marine having rich biodiversity
comprising of any one or more of the following components:

 The richness of wild as well as domesticated species or intra-


specific categories.
 High endemism.
 Presence of rare and threatened species.
 Keystone species.
 Species of evolutionary significance.
 Wild ancestors of domestic/cultivated species or their varieties
 Past pre-eminence of biological components represented by fossil
beds.
 Having significant cultural, ethical or aesthetic values.
 Important for the maintenance of cultural diversity (with or
without a long history of human association with them).

Q.85) Consider the following statements about the OPEC or Organization of the Petroleum Exporting
Countries.
1. All the petroleum exporting countries of the World are members of this organization.
2. They possess more than 80% of the world’s total crude oil reserves.
3. As the producers of crude oil, it does not influence the demand for petroleum.
4. It is to coordinate and unify petroleum policies among Member Countries.
5. To keep prices of the oil stable, OPEC countries usually maintain a quota of oil supply.
Which of the statements given above are correct?
(a) 1, 2, 3 and 4 only
(b) 2, 3, 4 and 5 only
(c) 1, 3, 4 and 5 only
(d) 1, 2, 4 and 5 only
EXPLANATION:
Currently, OPEC has 13 members (Algeria, Angola, Equatorial Guinea, Gabon, Iran, Iraq,
Kuwait, Libya, Nigeria, Republic of the Congo, Saudi Arabia, United Arab Emirates and
Venezuela). It is notable that some of the world’s largest oil producers, including Russia,
China, and the United States, are not members of OPEC. So, Statement 1 is not correct.
It is estimated that 44% of global oil production and 81.5% of the world's "proven" crude oil
reserves, giving OPEC a major influence on global oil prices. So, Statement 2 is correct.
OPEC as producers do not influence the demand for petroleum, they tend to change its
supply in order to influence the price by simply increasing or decreasing the supply of
crude oil. So, Statement 3 is correct.
The mission of OPEC is to coordinate and unify the petroleum policies of its Member
Countries and ensure the stabilization of oil markets in order to secure an efficient,
economic and regular supply of petroleum to consumers, a steady income to producers and
a fair return on capital for those investing in the petroleum industry. So, Statement 4 is
correct.
In order to keep prices of the oil stable, OPEC countries usually maintain a quota of oil
supply. OPEC countries together decide the amount of oil to be produced by them. while
keeping in mind the demand for oil in the market, OPEC decides the amount of crude to be
produced by member countries. So, Statement 5 is correct.

ADDITIONAL INFORMATION:
OPEC  It is a permanent, intergovernmental organization, initially
created at the Baghdad Conference in 1960, by Iran, Iraq,
Kuwait, Saudi Arabia, and Venezuela.
 Headquartered in Vienna, Austria.
Objective  OPEC’s objective is to coordinate and unify petroleum policies
among Member Countries, in order to secure fair and stable
prices for petroleum producers an efficient, economic and
regular supply of petroleum to consuming nations and a fair
return on capital to those investing in the industry.
OPEC Members  OPEC membership is open to any country that is a substantial
exporter of oil and which shares the ideals of the organization.
 As of now, OPEC has 13 members (Algeria, Angola, Equatorial
Guinea, Gabon, Iran, Iraq, Kuwait, Libya, Nigeria, Republic of
the Congo, Saudi Arabia, United Arab Emirates and Venezuela).
 Qatar terminated its membership on 1 January 2019.
 Ecuador withdraws its membership on 1 January 2020.
 It is notable that some of the world’s largest oil producers,
including Russia, China, and the United States, are not
members of OPEC.
OPEC+  The non-OPEC countries which export crude oil are termed as
Countries OPEC+ countries.
 It includes Russia, Azerbaijan, Bahrain, Brunei, Kazakhstan,
Malaysia, Mexico, Oman, South Sudan and Sudan.
 Note: USA and China are not allied with OPEC+ countries.

Q.86) A recent study has shown that the Indian Astronomical Observatory (IAO) at Hanle in Ladakh is
becoming one of the promising observatory sites globally. Which of the following will be the major
advantages for this?
1. More clear nights in a year
2. As dry as the Atacama Desert in Chile
3. Minimal light pollution
4. Extremely dry atmospheric condition
5. Uninterrupted by monsoon
Select the correct answer using the code given below.
(a) 1, 3, 4 and 5 only
(b) 1, 2 and 3 only
(c) 2, 3, 4 and 5 only
(d) 1, 2, 3, 4 and 5
EXPLANATION:
 The Indian Astronomical Observatory (IAO) located at Hanle near Leh in Ladakh is
becoming one of the promising observatory sites globally, said a recent study. This is
due to its advantages of more clear nights, minimal light pollution, background
aerosol concentration, extremely dry atmospheric condition, and uninterrupted by
monsoon.
 The study found that the Hanle site which is as dry as Atacama Desert in Chile and
much drier than Devasthal has around 270 clear nights in a year and is also one of
the emerging sites for infrared and sub-mm optical astronomy. This is because
water vapor absorbs electromagnetic signals and reduces their strength.
So, option (d) is correct.

ADDITIONAL INFORMATION:
ABOUT INDIAN  The Indian Astronomical Observatory (IAO), located
ASTRONOMICAL in Hanle near Leh in Ladakh, India, has one of the world's
OBSERVATORY highest located sites for optical, infrared, and gamma-ray
telescopes.
 It is operated by the Indian Institute of
Astrophysics, Bangalore. It is currently the tenth highest
optical telescope in the world, situated at an elevation of
4,500 meters (14,764 ft),
 The Observatory has two active telescopes. These are the 2.01
meter optical-infrared Himalayan Chandra Telescope (HCT)
and a High Altitude Gamma Ray Telescope (HAGAR). The HCT
is remotely operated from Bangalore from the Centre for
Research and Education in Science and Technology (CREST)
using a dedicated satellite link.
 The Indian Institute of Astrophysics is collaborating with the
McDonnell Center for the Space Sciences of Washington
University in St. Louis, the US to operate two 0.5
meters Cassegrain telescopes to monitor active galactic nuclei.
One of the observatories is to be established in Hanle. The
facilities 180 degrees apart in longitude are together to be
called the Antipodal Transient Observatory (ATO).

Q.87) Consider the following statements about the Small Finance Banks in India.
1. The minimum paid-up equity capital for small finance banks shall be Rs.100 crores.
2. The small finance bank shall primarily undertake basic banking activities of acceptance of deposits
and lending
3. The small finance banks will not be required to extend their Net Bank Credit to the sectors eligible
for classification as priority sector lending (PSL) by the Reserve Bank.
4. The small finance bank will be subject to the maintenance of Cash Reserve Ratio (CRR) and
Statutory Liquidity Ratio (SLR).
Which of the statements given above are correct?
(a) 1, 2 and 3 only
(b) 1, 2 and 4 only
(c) 1, 3 and 4 only
(d) 2, 3 and 4 only
EXPLANATION:
Small finance banks are a type of niche bank in India. Banks with a small finance bank
license can provide basic banking services of acceptance of deposits and lending. The aim
behind these is to provide financial inclusion to sections of the economy not being served by
other banks, such as small business units, small and marginal farmers, micro and small
industries, and unorganized sector entities. The minimum paid-up equity capital for small
finance banks shall be Rs. 100 crore. So, Statements 1 and 2 are correct.

 The small finance banks will be required to extend 75 percent of its Adjusted Net
Bank Credit (ANBC) to the sectors eligible for classification as priority sector lending
(PSL) by the Reserve Bank. At least 50 percent of its loan portfolio should constitute
loans and advances of up to Rs. 25 lakh.
 The small finance bank will be subject to all prudential norms and regulations of
RBI as applicable to existing commercial banks including the requirement of
maintenance of Cash Reserve Ratio (CRR) and Statutory Liquidity Ratio (SLR). No
forbearance would be provided for complying with the statutory provisions.
So, Statement 3 is not correct and statement 4 is correct.

ADDITIONAL INFORMATION:
SMALL FINANCE BANKS
OBJECTIVES  provision of savings vehicles, and
 supply of credit to small business units, small and marginal
farmers, micro and small industries, and other unorganized
sector entities, through high technology-low-cost operations.
SCOPE OF  The small finance bank shall primarily undertake basic banking
ACTIVITIES activities of acceptance of deposits and lending to unserved and
underserved sections including small business units, small and
marginal farmers, micro and small industries, and unorganized
sector entities.
 There will not be any restriction in the area of operations of small
finance banks.
ELIGIBILITY  Existing non-banking financial
CRITERIA AND companies (NBFC), microfinance institutions (MFI), and local area
REGULATIONS banks (LAB) can apply to become small finance banks.
 They can be promoted either by individuals, corporations, trusts,
or societies.
 They are established as public limited companies in the private
sector under the Companies Act, 2013.
 They are governed by the provisions of the Reserve Bank of India
Act, 1934, Banking Regulation Act, 1949, and other relevant
statutes.
 Resident individuals/professionals with 10 years of experience in
banking and finance;
 Companies and societies owned and controlled by residents will
be eligible to set up small finance banks.
 Existing Non-Banking Finance Companies (NBFCs), Micro
Finance Institutions (MFIs), and Local Area Banks (LABs) that are
owned and controlled by residents can also opt for conversion
into small finance banks.
 Promoter/promoter groups should be ‘fit and proper with a sound
track record of professional experience or of running their
businesses for at least five years to be eligible to promote small
finance banks.

OPERATIONAL  Ujjivan Small Finance Bank.


SFB’S IN INDIA  Janalakshmi Small Finance Bank.
 Equitas Small Finance Bank.
 A U Small Finance Bank.
 Capital Small Finance Bank, etc.

Q.88) The Indian Ports Bill 2021incorporates which of the following international regulatory
developments in India’s legislation?
1. International Ship and Port Facility Security (ISPS)
2. International Convention for the prevention of pollution from ships (MARPOL)
3. International Ballast Water Management (BWM) convention
4. Law of the Sea Convention
5. International Maritime Organization
Select the correct answer using the code given below.
(a) 1 and 2 only
(b) 1, 2 and 3 only
(c) 1, 2 and 4 only
(d) 1, 2, 3, 4 and 5
EXPLANATION:
The new Indian ports Bill incorporates international regulatory developments in the
national legislation namely the -
 the International Ship and Port Facility Security (ISPS) Code (2004), developed in the
aftermath of the 9-11 terror attacks, is a set of measures to enhance the security of
ships and port facilities.
 The International Convention for the Prevention of Pollution from Ships (MARPOL)
(1983, 2005) covers the prevention of pollution of the maritime environment by ships
due to operational or accidental causes. It mandates that port authorities must
provide adequate 'reception facilities’ to ships to dispose of their waste.
 The International Ballast Water Management (BWM) Convention (2017) aims at
preventing the spread of invasive aquatic species and potentially harmful pathogens
in ships' ballast water when it is released into a new location.
So, option (b) is correct.

ADDITIONAL INFORMATION:
INDIAN PORTS BILL 2021
SIGNIFICANT  The Indian Ports Bill 2021 seeks to replace more than a century
FEATURES old Indian Ports Act 1908.
 The Bill mandates port authorities to provide adequate “reception
facilities” to ships to dispose of their waste.
 It contains provisions for the safety and security of ports and the
prevention and containment of pollution at ports.
 These provisions will apply to all ports in India (major and non-
major ports).
 The ports are required to prepare a “Security Plan” and a waste
reception and handling plan.
 The Ports will also be subject to periodic audits by the Union
government to ensure compliance.

CONTENTIOUS Maritime State Development Council


PRINCIPLES  The Council will be established by the Centre with the Union
Minister of Ports, Shipping and Waterways as the Chairperson.
 The Ministers in charge of ports in the Maritime States will be the
members.
 It also includes the Secretary and Joint Secretaries of the Ministry
dealing with ports.
 But neither the State Maritime Boards nor expert members on
port administration and trade interests find a place in the Council.
 The Bill also proposes to make the Council a permanent body with
wide-ranging powers to -
o formulate a national plan for the development of existing
and new ports and revise the plan periodically
o monitor the development of major and non-major ports to
ensure their integrated development with the national plan
Powers
 The Union government can make a port non-operational if it does
not align with the national plan.
 Heavy penalties for administrative lapses on Port authorities and
officials ranging from Rs. 50,000 to Rs. 2 lakh or imprisonment up
to 6 months or both.
 The excessive authority and power concentrated in the Union
government relegate the maritime States into a secondary
position.
Q.89) Consider the following statements about the Indian Space Association (ISpA)
1. It is represented only by the leading home-grown companies and not by the global corporations
with advanced capabilities in space and satellite technologies.
2. It will carry out only the commercial launches for private companies and not for the strategic vision
of the Indian Government.
Which of the statements given above is/are correct?
(a) 1 only
(b) 2 only
(c) Both 1 and 2
(d) Neither 1 nor 2
EXPLANATION:
ISpA is represented by both leading homegrown and global corporations with advanced
capabilities in space and satellite technologies. So, Statement 1 is not correct.
ISpA said it would engage with stakeholders across the ecosystem for the formulation of an
enabling policy framework that fulfils the government vision of leading commercial space
exploration. So, Statement 2 is not correct.

ADDITIONAL INFORMATION:
FOUNDING  Its founding members include Larson & Toubro, Nelco (Tata
MEMBERS Group), OneWeb, Bharti Airtel, Mapmyindia, Walchandnagar
AND OTHER Industries, and Ananth Technology Limited.
CORE  Other core members include Godrej, Hughes India, Azista-BST
MEMBERS Aerospace Private Limited, BEL, Centum Electronics, Maxar India.
SIGNIFICANCE  ISpA aims to contribute to the Government of India’s vision of
OF THE making India Atmanirbhar a global leader in the space arena,
INDIAN SPACE which is fast emerging as the next growth frontier for mankind.
ASSOCIATION  The association will engage with stakeholders across the
ecosystem for the formulation of an enabling policy framework
that fulfills the Government’s vision.
 ISpA will work towards building global linkages for the Indian
space industry to bring in critical technology and investments into
the country to create more high-skill jobs.
 ISpA plans to work in very close coordination with IN-SPACe to
further the space vision of the Government and to expand its
capacity in the global space arena.

Q.90) Which of the following species found in India is/are classified as the‘critically endangered’ under
the Union for Conservation of Nature’s Red List of Threatened Species?
1. Kashmir stag
2. Malabar Civet
3. Lion tailed macaque
Select the correct answer using the code given below.
(a) 1 and 2 only
(b) 1 and 3 only
(c) 2 only
(d) 1, 2 and 3
EXPLANATION:
Kashmir Stag is restricted to the Dachigam National Park and as per the IUCN’s Red List, it
is classified as Critically Endangered. Malabar Civet is a viverrid, endemic to the Western
Ghats of India with IUCN Status as Critically Endangered. The lion-tailed macaque is
endemic to the Western Ghats in the states of Karnataka, Kerala and Tamil Nadu. However,
as per the IUCN’s Red List, it is classified as Endangered. So, Option (a) is correct.

ADDITIONAL INFORMATION:
SPECIES STATUS RELATED INFORMATION
Kashmir Stag  IUCN’s Red  The Kashmir stag or Hangul is the
List - state animal of Jammu and Kashmir
Critically and is the only surviving species of
Endangered. red deer in India.
 Listed under  Primarily restricted within the
the Schedule Dachigam Wildlife Sanctuary near
1 of Srinagar.
WPA,1972.  Project Hangul- In the 1970s, an
 Listed under initiative by the Government of
the CITES Jammu and Kashmir along with the
Appendix I. support of IUCN and World Wildlife
Fund (WWF) for the protection of the
habitat of Hangul.
 Hangul Conservation Project was
launched by Wildlife Conservation
Fund.
Malabar Civet  IUCN’s Red  Endemic to the Western Ghats of
List - India.
Critically  Species is nocturnal.
Endangered.  It has been hunted for the extraction
 Listed under of Civetone from scent glands used as
the Schedule medicine and aromatic compound.
1 of
WPA,1972.
 Listed under
the CITES
Appendix I.
Lion Tailed  IUCN Red  It is endemic to the Western Ghats in
Macaque list- the states of Karnataka, Kerala and
Endangered. Tamil Nadu.
 Listed under  Although the species has a relatively
the Schedule wide range, its area of occupancy is
1 of small and severely fragmented.
WPA,1972.  Protected areas:
 Listed under  Kudremukh National Park
the CITES (Karnataka)
Appendix I.  Periyar National Park (Kerala)
 Silent Valley National Park (Kerala)
 Threats: Deforestation, Agriculture
such as tea, eucalyptus and coffee.

Q.91) Consider the following statements about the status of geoheritage sites in India.
1. India is a signatory to the establishment of UNESCO Global Geoparks.
2. In India, geo heritage sites are protected under the Environment Conservation Act of 1986.
3. Geological Survey of India (GSI) has identified 32 sites as National Geological Monuments.
4. All the National Geological Monuments in India are recognised by the UNESCO
Which of the statements given above is/are correct?
(a) 1 and 2 only
(b) 2 and 3 only
(c) 1 and 3 only
(d)
2 and 4 only
EXPLANATION :

India does not have any legislation and policy for conservation of geoheritages.
So, Statement 2 is not correct .
None of the geo parks in India is recognised by the UNESCO so far.
So, Statement 4 is not correct .

ADDITIONAL INFORMATION :
India’s Geo Diversity :
Geoheritage  The geological heritage of our planet was first recognised in 1991 at First
sites International Symposium on the Conservation of our Geological
Heritage (Digne resolution).
 UNESCO facilitated efforts to create a formal programme to promote a global
network of geoheritage sites complementing the World Heritage Convention
and the UNESCO’s MAB.
 Countries like Vietnam and Thailand have implemented laws to conserve
their geological and natural heritage.
 The Global Geoparks Network was founded in 2004 as an international
partnership developed under the umbrella of UNESCO.
 Today, there are 169 Global Geoparks across 44 countries.

Status of  India is long known as the world’s natural laboratory for geo-scientific
India learning but we are inching towards the disappearance of most of our
geological heritage sites.
Reasons
 Lack of geological literacy - Indian classrooms view disciplines like
environmental science and geology inferior to other pure subjects like
physics, biology, and chemistry.
 The lack of awareness in geo-heritage parks makes decision-making on
climate change difficult.
 Developmental activities - The Anjar site in Kutch district which has high
concentration of iridium providing evidence for a massive meteoritic impact
that caused the extinction of dinosaurs about 65 million years ago was
destroyed by laying of a new rail track.
 A national geological monument exhibiting a unique rock called Nepheline
Syenite in Ajmer district of Rajasthan was destroyed in a road-widening
project.
 The Lonar impact crater in Buldhana district of Maharashtra is under the
threat of destruction.
 Unplanned and booming real estate business has destroyed many
geoheritage sites.
 Unregulated stone mining activities have also contributed to this
destruction.
Way ahead  The situation calls for immediate implementation of sustainable
conservation measures such as those formulated for protecting biodiversity.
 A progressive legal framework is needed to conserve geoheritage sites.
 There is a need for a national conservation policy under the direct
supervision of a national body committed to the protection of geo-heritage
sites.

Q.92) Which one of the following statements is not correct about the Wildlife Protection Act of 1972?
(a) It empowers the central and state governments to declare any area a wildlife sanctuary, national
park or closed area
(b) It completely prohibits the capture and transportation of wild animals
(c) It enforces and enables the state governments to constitute a Wildlife Advisory Board
(d) It prohibits hunting of any wild animal specified in Schedules I, II, III, and IV of this act

EXPLANATION:
The Wildlife Protection Act of 1972, prohibits the hunting of any wild animal specified in
Schedules I, II, III and IV of the act. However, there is an exception that a wild animal listed
under these schedules can be hunted/ killed only after getting permission from the Chief
Wildlife Warden (CWLW) of the state. So, Option (b) is not correct.

ADDITIONAL INFORMATION:
Wildlife  Enacted for the protection of plants and animal species. Currently,
(Protection) it extends to the whole of India except the State of Jammu and
Act, 1972 Kashmir.
 Before 1972, India had only five designated national parks.
 According to the National Wildlife Database, there are 101 existing
national parks in India covering an 1.23% of the geographical area
of the country.
Schedules of  It has six schedules that give varying degrees of protection.
WPA,1972.  Schedule I and Schedule II provide high protection and offences
under these are prescribed the highest penalties. Eg: Tiger and
Himalayan Black Bear.
 Species listed in Schedule III and Schedule IV are also protected,
but the penalties are much lower. Eg: Chital and Flamingo.
 However, a wild animal listed under the schedule I, II, III and IV (
can be hunted/ killed only after getting permission from the Chief
Wildlife Warden (CWLW) of the state.
 Animals under Schedule V, e.g. common crows, fruit bats, rats and
mice, are legally considered vermin and may be hunted freely.
 The specified endemic plants in Schedule VI are prohibited from
cultivation and planting but prior approval of the Chief Wildlife
Warden (CWLW) of the state is needed for such use. Eg. Pitcher
plant.
Bodies under  Bodies constituted under the Wildlife (Protection) Act, 1972 are:
WPA,1972. National Board for Wildlife (NBWL), Standing Committee of NBWL,
State Board for Wildlife (SBWL, Central Zoo Authority, National
Tiger Conservation Authority (NTCA) and Wildlife Crime Control
Bureau (WCCB)
National  It serves as an apex body for the review of all wildlife-related
Board for matters and for the approval of projects in and around national
Wildlife parks and sanctuaries.
(NBWL)  Chaired by the Prime Minister and is responsible for the promotion
of conservation and development of wildlife and forests.
 The Minister of Environment, Forest and Climate Change is the
Vice-Chairperson of the board.
 The board is ‘advisory’ in nature and can only advise the
Government on policymaking for the conservation of wildlife.
 Standing Committee of NBWL: The NBWL constitutes a Standing
Committee for the purpose of approving all the projects falling
within protected wildlife areas or within 10 km of them.
 The committee is chaired by the Minister of Environment, Forest
and Climate Change.
State Board  The state governments are responsible for the constitution of the
for Wildlife state board of wildlife.
(SBWL)  The Chief Minister of the state/UT is the chairperson of the board.
 The board advises the state government in:
 The selection and management of areas to be declared as protected
areas.
 The formulation of the policy for protection and conservation of the
wildlife and any matter relating to the amendment of any Schedule.

Q.93) Tiger is an “umbrella species” which ensures


1. Viable populations of other wild animals.
2. The water security of the region.
3. The climate security of the region.
Which of the statements given above is/are correct?
(a) 1 and 2 only
(b) 2 and 3 only
(c) 1 only
(d) 1, 2 and 3
EXPLANATION:
Tiger is an “umbrella species” which ensures viable populations of other wild animals (by
hunting down the prey and co-predator) and forest, thereby ensuring the ecological viability (the
capacity of an ecosystem to preserve its integrity and stability to host its original biodiversity) of
the entire area and habitat, which also ensures the water and climate security of the region. So,
Option (d) is correct.

Q.94) Which one of the following statements is correct about the Nairobi Convention?
(a) It aims to work towards a prosperous Western Indian Ocean region with healthy rivers,
coasts and oceans
(b) It is a new climate plan to slash greenhouse gas emissions by 2030
(c) It commits the countries to halt and reverse deforestation and land degradation by 2030
(d) It aims to work towards all sales of new cars and vans being zero emission globally
EXPLANATION:
The Nairobi Convention is a partnership between governments, civil society, and the private
sector, working towards a prosperous Western Indian Ocean Region with healthy rivers, coasts
and oceans. It pursues this vision by providing a mechanism for regional cooperation,
coordination, and collaborative actions; it enables the Contracting Parties to harness resources
and expertise from a wide range of stakeholders and interest groups, and in this way, it helps
solve inter-linked problems of the region’s coastal and marine environment. So, option (a) is
correct.

ADDITIONAL INFORMATION:
NAIROBI CONVENTION
HISTORY  The Nairobi Convention, which was first signed in 1985 and entered
into force in 1996, is part of UNEP’s Regional Seas Programme.
 The program aims to address the accelerating degradation of the
world’s oceans and coastal areas through the sustainable
management and use of the marine and coastal environment.
 It does this by engaging countries that share the western Indian
Ocean in actions to protect their shared marine environment.
 The contracting
parties (Comoros, France, Kenya, Madagascar, Mauritius, Mozambiq
ue, Seychelles, Somalia, Tanzania, and the Republic of South
Africa ) to the Convention are part of more than 143 countries that
participate in 18 Regional Seas initiatives
STAKEHOLDERS  The Secretariat works closely with collaborating partners such as
the “Consortium for Conservation of Coastal and Marine
Ecosystems in the Western Indian Ocean” referred to as the (“WIO-
C”) membership, other regional NGOs, and various national and
research institutions.
 Established in 2006 the “WIO-C” is a partnership between major
NGOs with the support of IGOs in the Western Indian Ocean. WIO-C
is anchored in the Nairobi Convention and is designed to improve
information exchange, synergy, and coordination between NGOs
working on coastal and marine environmental issues in the Western
Indian Ocean region, and to move towards a joint programmatic
approach in addressing these issues.

Q.95) Consider the following pairs:


(Marine Protected Areas) (State)
1.Chorao Island - Maharashtra
2. Khijadia - Gujarat
3. Lothian Island - West Bengal
Which of the pairs given above is/are correctly matched?
(a) 2 only
(b) 1 and 3 only
(c) 2 and 3 only
(d) 1, 2 and 3
EXPLANATION:
 Chorão, also known as Chodan, is an island along the Mandovi
River near Ilhas, Goa, India. It is the largest among the other 17 islands of Goa.
 Khijadiya Bird Sanctuary is a bird sanctuary located in the Jamnagar
district of Gujarat, India. About 300 species of migratory birds have been recorded
here. The sanctuary is unique having both freshwater lakes, salt, and freshwater
marshlands.
 Lothian Island Wildlife Sanctuary is situated in South 24 Parganas district, West
Bengal, India. The wildlife in this sanctuary includes estuarine crocodiles, olive
ridley sea turtles, spotted deer, jungle cats, and rhesus macaques. The tropical
wetland forest consists of mangrove vegetation that provides a dense cover along
with the habitat.
So, option (c) is correct.

ADDITIONAL INFORMATION:
MARINE  A marine protected area (MPA) is essentially a space in the ocean
PROTECTED where human activities are more strictly regulated than the
AREAS surrounding waters - similar to parks we have on land. These
places are given special protections for natural or historic marine
resources by local, state, territorial, native, regional, or national
authorities.
ITS  Establishing MPA networks is critical to maintaining climate
IMPORTANCE change resilience and rebuilding ecological and social resilience.
For example, MPAs that protect coastal habitats such as barrier
islands, coral reefs, mangroves, and wetlands reduce human
vulnerability in the face of climate change and provide the natural
infrastructure (e.g. storm protection) on which people rely.
 Strictly protected MPA networks in coastal carbon habitats
(mangroves, seagrasses, salt marshes) can ensure that no new
emissions arise from the loss and degradation of these areas.
 At the same time, they stimulate new carbon sequestration through
the restoration of degraded coastal habitats.
 MPAs, while not impervious to all climate change impacts, provide
areas of reduced stress, improving the ability of marine organisms
to adapt to climate change.
 Well-integrated MPA networks can increase species survival by
allowing them to move around and escape certain pressures. In
addition, MPAs, where stressors are controlled, can be used as
sentinel (research) sites to help track the effects of climate
change. This is consistent with the research and systematic
observation obligations of countries under the UN Framework
Convention on Climate Change (UNFCCC) and other international
agreements.

Q.96) Consider the following statements about the ‘Blue Flag’ certification:
1. It is awarded by the Foundation for Environmental Education.
2. It can also be obtained by a sustainable boating tourism operator.
3. Each coastal state of India has at least one beach under Blue Flag certification.
Which of the statements given above is/are correct?
(a) 1 and 2 only
(b) 1 and 3 only
(c) 2 only
(d) 1, 2 and 3
EXPLANATION:
The ‘Blue Flag’ is a certification that can be obtained by a beach, marina, or sustainable
boating tourism operator, and serves as an eco-label. The certification is awarded by the
Denmark-based non-profit Foundation for Environmental Education, which sets stringent
environmental, educational, safety-related, and access-related criteria that applicants must
meet and maintain. It is awarded annually to beaches and marinas in FEE member
countries. So, Statements 1 and 2 are correct.

The beaches selected for the certification are Kappad (Kerala), Shivrajpur (Gujarat),
Ghoghla (Diu), Kasarkod, and Padubidri (Karnataka), Rushikonda (Andhra Pradesh),
Golden (Odisha), and Radhanagar (Andaman & Nicobar Islands). Recently, two more Indian
beaches -- the Eden Beach in Puducherry and Kovalam Beach in Tamil Nadu – have been
awarded the title. All the coastal states in India have one blue flag certified beach except
Goa and West Bengal. So, Statement 3 is not correct.

ADDITIONAL INFORMATION:
BLUE FLAG CERTIFICATION
HISTORY  The Blue Flag program was started in France in 1985 and areas out
of Europe in 2001. The program promotes sustainable development
in freshwater and marine areas through four main criteria: water
quality, environmental management, environmental education, and
safety.
 Forty-seven countries currently participate in the program, and
4,573 beaches, marinas, and boats have this certification.

MISSION  promote and participate in environmental education programs


for the users of beaches, marinas, and boating operators.
 implement sound safety and environmental management
systems.
 monitor environmental conditions to reduce the impact of
human activity at the beaches, marinas, and boating operators.
 commit to partnerships and collaborative action to promote the
sustainable development of tourism.
SELECTING  coveted ‘Blue Flag’ certification by an eminent international jury,
BOARD which comprises members of the United Nations Environment
Programme (UNEP), United Nations World Tourism Organization
(UNWTO), Foundation for Environmental Education (FEE), and
International Union for Conservation of Nature (IUCN).

Q.97) The Central and State Pollution Control Boards have been assigned responsibilities under the
1. Bio-Medical Waste Management Rules, 2016
2. Public Liability Insurance Act, 1991
3. Plastic Waste Management Rules, 2016
4. Environment (Protection) Act, 1986
5. Solid Waste Management Rules, 2016
Select the correct answer using the code given below.
(a) 2 and 4 only
(b) 1, 3 and 5 only
(c) 1, 3, 4 and 5 only
(d) 1, 2, 3, 4 and 5

EXPLANATION:
The Central and various state pollution control boards are working based on the acts
passed by the government to maintain the same. And the acts and rules, assigned power to
them are,
 Bio-Medical Waste Management Rules, 2016
 Public Liability Insurance Act, 1991
 Water (Prevention and Control of Pollution) Act, 1974
 Air (Prevention and Control of Pollution) Act, 1981
 Plastic Waste Management Rules, 2016
 Environment (Protection) Act, 1986
 Solid Waste Management Rules, 2016
So, option (d) is correct.

ADDITIONAL INFORMATION:
FUNCTIONS  Advise the Central Government on any matter concerning prevention
OF CPCB and control of water and air pollution and improvement of the quality
of air.
 Plan and cause to be executed a nation-wide program for the
prevention, control, or abatement of water and air pollution;
 Coordinate the activities of the State Board and resolve disputes
among them;
 Provide technical assistance and guidance to the State Boards, carry
out and sponsor investigation and research relating to problems of
water and air pollution, and for their prevention, control or
abatement;
 Plan and organize training of persons engaged in the program on the
prevention, control, or abatement of water and air pollution;
 Organize through mass media, a comprehensive mass awareness
program on the prevention, control, or abatement of water and air
pollution;
 Collect, compile and publish technical and statistical data relating to
water and air pollution and the measures devised for their effective
prevention, control, or abatement;
 Prepare manuals, codes, and guidelines relating to treatment and
disposal of sewage and trade effluents as well as for stack gas
cleaning devices, stacks, and ducts;
 Disseminate information in respect of matters relating to water and
air pollution and their prevention and control;
 Lay down, modify or annul, in consultation with the State
Governments concerned, the standards for stream or well, and lay
down standards for the quality of air; and
 Perform such other functions as may be prescribed by the
Government of India.

Q.98) Consider the following statements about the Anti Defection Law in India.
1. The law does not provide a time frame within which the presiding officer has to decide a defection
case.
2. As per the Supreme Court verdict, the Speakers should decide on a defection petition within three
months.
3. As per the constitution, the anti-defectionlaw will not be applicable during the no-confidence
motions.
Which of the statements given above is/are correct?
(a) 1 only
(b) 1 and 2 only
(c) 2 and 3 only
(d) 1, 2 and 3
EXPLANATION :

The law does not provide a time-frame within which the presiding officer has to decide a defection
case.
There have been many instances where a Speaker has misused this in not determining the case of
a defecting MLA until the end of the legislature term.
So, Statement 1 is correct.
Supreme Court held that ideally Speakers should take a decision on a defection petition
within three months.
It also said that Parliament should set up an independent tribunal headed by a retired judge of the
higher judiciary to decide defection cases swiftly and impartially.
So, Statement 2 is correct .
There is no such provision in the constitution and in fact the Anti Defection law is most suitable
during the passage of no confidence motion.
So, Statement 3 is not correct .
ADDITIONAL INFORMATION :
Anti-Defection Law :
Context  Aaya Ram Gaya Ram was a phrase that became popular in Indian politics
after a Haryana MLA Gaya Lal changed his party thrice within the same
day in 1967.
 The anti-defection law sought to prevent such political defections which
may be due to reward of office or other similar considerations.
Enactment  The Tenth Schedule was inserted in the Constitution in 1985. It lays down
the process by which legislators may be disqualified on grounds of
defection by the Presiding Officer of a legislature based on a petition by any
other member of the House.
 A legislator is deemed to have defected if he either voluntarily gives up the
membership of his party or disobeys the directives of the party leadership on
a vote.
 This implies that a legislator defying (abstaining or voting against) the party
whip on any issue can lose his membership of the House.
 The law applies to both Parliament and state assemblies.
Exceptions  Legislators may change their party without the risk of disqualification in
certain circumstances.
 The law allows a party to merge with or into another party provided that at
least two-thirds of its legislators are in favour of the merger.
 In such a scenario, neither the members who decide to merge, nor the ones
who stay with the original party will face disqualification.
Recommen-  Various expert committees have recommended that rather than the
dations Presiding Officer, the decision to disqualify a member should be made by
the President (in case of MPs) or the Governor (in case of MLAs) on the
advice of the Election Commission.
 This would be similar to the process followed for disqualification in case the
person holds an office of profit (i.e. the person holds an office under the
central or state government which carries a remuneration, and has not
been excluded in a list made by the legislature).
Judicial  The law initially stated that the decision of the Presiding Officer is not
Review subject to judicial review.
 This condition was struck down by the Supreme Court in 1992, thereby
allowing appeals against the Presiding Officer’s decision in the High Court
and Supreme Court.
 However, it held that there may not be any judicial intervention until the
Presiding Officer gives his order.
Time Limit  The law does not specify a time-period for the Presiding Officer to decide on
a disqualification plea.
 Given that courts can intervene only after the Presiding Officer has decided
on the matter, the petitioner seeking disqualification has no option but to
wait for this decision to be made.
Performance  The anti-defection law seeks to provide a stable government by ensuring
and the legislators do not switch sides.
Suggestions  However, this law also restricts a legislator from voting in line with his
conscience, judgement and interests of his electorate.
 Such a situation impedes the oversight function of the legislature over the
government, by ensuring that members vote based on the decisions taken
by the party leadership, and not what their constituents would like them to
vote for.
 Political parties issue a direction to MPs on how to vote on most issues,
irrespective of the nature of the issue.
 Several experts have suggested that the law should be valid only for those
votes that determine the stability of the government (passage of the annual
budget or no-confidence motions)

Q.99) Consider the following statements about the All-India Quarterly Establishment-based Employment
Survey (QES) and Periodic Labour Force Survey (PLFS).
1. While the QES provides a demand-side picture, the Periodic Labour Force Survey (PLFS) gives the
supply side picture of the labour market.
2. While QES is released by the Labour Bureau of Ministry of Labour and Employment, the PLFS is
released by National Sample Survey Office (NSSO) under the Ministry of Statistics and Programme
Implementation.
Which of the statements given above is/are correct?
(a) 1 only
(b) 2 only
(c) Both 1 and 2
(d) Neither 1 nor 2
EXPLANATION :

While the QES provides a demand side picture, Periodic Labour Force Survey (PLFS) gives the
supply side picture of the labour market.
The Periodic Labour Force Survey (PLFS) gives estimates of Key employment and unemployment
Indicators like the Labour Force Participation Rates (LFPR), Worker Population Ratio (WPR),
Unemployment Rate (UR), etc.
The QES survey covers establishments employing 10 or more workers in the organised segment in
nine sectors (manufacturing, construction, trade, transport, education, health, accommodation
and restaurants, IT/BPO, and financial service activities).
So, Statement 1 is correct.

While QES is released by the Labour Bureau of Ministry of Labour and Employment, the PLFS is
released by National Sample Survey Office (NSSO) under Ministry of Statistics and Programme
Implementation.
So, Statement 2 is correct.

ADDITIONAL INFORMATION :
All-India Quarterly Establishment-based Employment Survey (QES)
AQEES  The AQEES will be prepared by the Labour Bureau.
 The survey covers establishments employing 10 or more workers in the
organised segment in nine sectors (manufacturing, construction, trade,
transport, education, health, accommodation and restaurants, IT/BPO,
and financial service activities).
 These sectors account for 85% of the total employment in establishments
employing 10 or more workers as per the Sixth Economic Census (EC),
which serves as the basis of the QES survey.
 The data for QES were collected either telephonically or through visits.

Need  The stated objective of the QES is to enable the government to frame a
“sound national policy on employment”.
 India ratified the International Labour Organization’s Employment Policy
Convention, 1964, which requires the ratifying countries to implement
“an active policy designed to promote full, productive and freely chosen
employment.”
 India does not have one till now.
Components  There are two components under AQEES, Quarterly Employment Survey
(QES) and Area Frame Establishment Survey (AFES).
o The Quarterly Employment Survey (QES) under AQEES would
provide the employment estimates for the establishments
employing 10 or more workers.

o The Area Frame Establishment Survey (AFES) would provide the


employment estimates for the establishments recruiting 9 or less
workers.
Highlights  Of the total employment estimated in the selected nine sectors,
Manufacturing accounted for nearly 39%, followed by Education with
22% and Health as well as IT/BPOs sectors both around 10%. Trade and
Transport sectors engaged 5.3% and 4.6% of the total estimated workers
respectively.
 Nearly 90% of the establishments have been estimated to work with less
than 100 workers
 Regular workers constitute 87% of the estimated workforce in the nine
selected sectors, with only 2% being casual workers.
 The over-all percentage of female workers stood at 32.1
 23.5% of all the establishments were registered under the Companies Act,
this percentage was as high as 82.8% in IT / BPO, 51.2% in
Construction, 42.8% in Manufacturing, 36.4% in Transport, 32.1% in
Trade and 23.8% in financial services.
 Looking at the educational qualifications of workers, it came out that
28.4% of those working in seven of the nine sectors (excluding Education
and Health)were matriculates/secondary or less educated, while
another 37.0% were graduates or had higher qualifications.
 It is somewhat encouraging to note that 16.8% of the establishments
provided formal skill development programmes, although mostly for their
own employees.

Periodic Labour Force Survey (PLFS)


 Considering the importance of availability of labour force data at more frequent time
intervals, National Statistical Office (NSO) launched Periodic Labour Force Survey
(PLFS) in April 2017.

 The objective of PLFS is primarily twofold:


o to estimate the key employment and unemployment indicators (viz. Worker
Population Ratio, Labour Force Participation Rate, Unemployment Rate) in
the short time interval of three months for the urban areas only in the
‘Current Weekly Status’ (CWS).
o to estimate employment and unemployment indicators in both ‘Usual Status’
(ps+ss) and CWS in both rural and urban areas annually.

Q.100) Consider the following statements about the role of Panchayat Raj Institutions (PRI) in disaster
management.
1. The PRIs performs both regulatory and welfare functions at the grass-root level which quickens the
pace of recovery while ensuring support to the vulnerable population.
2. Gram sabhas provide a platform to build consensus and make resolutions in the community’s
interest such as adherence to COVID norms.
3. When the traditional top-down disaster response system was compromised during the bad times of
disaster, then the PRIs will play remarkable role in the bottom-up approach.
Which of the statements given above are correct?
(a) 1 and 2 only
(b) 1 and 3 only
(c) 2 and 3 only
(d) 1, 2 and 3
EXPLANATION :

The PRIs performs both regulatory and welfare functions at the grass root level which quickens the
pace of recovery while ensuring support to the vulnerable population.
For eg, during COVID times, when there was nationwide lockdown, PRIs set up containment
zones, arranged transport and provisioned food for e incoming migrants, Effective implementation
of welfare schemes like MGNREGA and NRLM quickened the pace of recovery.
So, Statement 1 is correct .

Gram sabhas provide a platform to build consensus and make resolutions in the community’s
interest such as adherence to Covid norms.
For eg., they organised community-based surveillance systems involving village elders, youth and
SHGs to keep a strict vigil in quarantine centres and monitor symptoms in households.
So, Statement 2 is correct .
When the traditional top-down disaster response system was compromised during the bad times of
disaster, then the PRIs will play remarkable role in the bottom-up approach which helped reduce
Covid 19 risks, responded swiftly and thus helped people recover quickly.
The role of PRIs in mobilising citizens for COVID-19 vaccination is commendable.
It also provided essential leadership at the local level.
So, Statement 3 is correct .

ADDITIONAL INFORMATION :
Strengthening the PRIs for effective disaster management :

Why it is  With the expansion of Panchayati Raj systems and Gram sabhas becoming
needed more vibrant, it is imperative to make disaster resilience an inherent part of
community culture.
How can  It is crucial to include disaster management chapters in Panchayat Raj Acts
this be done to ensure citizen-centric mapping and planning of resources.
 Regular location-specific training programmes for the community can be
conducted.
 Platforms can be organised for sharing best practices that will strengthen
individual and institutional capacities.
 Since community is usually the first responder in case of a
disaster, community-based disaster management plans can be formulated
where traditional wisdom of local communities can complement modern
practices.
 Financial contributions from the community should be encouraged through
the establishment of community disaster funds in all gram panchayats.

You might also like